Download as pdf or txt
Download as pdf or txt
You are on page 1of 104

Kapitel 1: Cellulr neurobiologi

28. januar 2014


11:26

Opgaver side 1

1.1. Gliaceller:
30. januar 2014
21:14

1.1.1. Nvn de forskellige typer af gliaceller i det perifere og det centrale nervesystem, beskrev
deres morfologi og angiv de vigtigste funktioner for hver celletype.
Glia-celler er sttteceller til neuronerne. Overordnet er der tre typer:
Astrocytter: Astrocytter er sttteceller, der har mange cellulre udlbere. Der er mange af
dem og de udgr mellem 20% og 50% af cellevolumen i CNS. Astrocytterne er koblet sammen
via gab junctions og danner et netvrk, som har relation til bde neuroner, kapillrer, CSF og
andre strukturer.
En af de vigtigste opgaver, som astrocytten varetager, er reguleringen af EC-kalium. Desuden
regulerer astrocytten pH, vandbalancen og neurotransmitterkoncentration i CSF og desuden er
de med til at danne blod-hjerne-barrieren.
Oligodendrocytter (i CNS) og Schwannske celler (i PNS): Oligodendrocytter giver sttte til og
isolerer neuroner i CNS. Schwansske celler gr det samme, men i PNS. En enkelt
oligodendrocyt kan vre i kontakt med op til 50 axoner, mens en schwannsk celle kun kan
vre i kontakt med n.
Begge celletyper danner myelinskeder, ved at omslutte axonerne med lamellae, der dannes,
nr gliacellen vikler sig selv rundt om axonet. Myelinen er en i mellem afbrudt af Ranvierske
indsnrringer.
Microglia: Microglia er fagocyterende celler. De er meget sm og udgr en forholdsvis lille del
af gliacellerne. Microglia stammer fra monocytter, der vokser ind i nervevv og undergr
forandringer, men som beholder deres fagocyterende egenskaber. De er i stand til at fjerne
fremmed eller ddt materiale.
1.1.2. Nvn de celler, der danner myelin i det perifere og det centrale nervesystem og redegr for
forskellen i deres strukturelle relation til axonerne:
Myelinskeder dannes i PNS af Schwannske celler og i CNS af oligodendrocytter:
Oligodendrocytter kan danne myelinskeder omkring flere axoner p n gang, mens de schwansnske
celler kun kan danne myelinskeder omkring t axon.

Opgaver side 2

1.2. Neurotransmitterer:
30. januar 2014
21:14

1.2.1. Hvilke kriterier skal vre opfyldt, for at et kemisk stof kan siges at vre en
neurotransmitter?
For at vise, at et stof er en neurotransmitter, skal man vise, at der findes en tilsvarende receptor og
at transmitterstoffet findes i tilstrkkelig koncentration til at aktivere receptoren.
Desuden skal flgende kriterier opfyldes:

Stoffet skal produceres af neuroner.


Stoffet skal lagres i nerveterminalen.
Stoffet skal frigives ved depolarisering og frigivelsen skal ske via calcium-medieret exocytose.
Transmitterstoffet skal vre direkte rsag til postsynaptiske ndringer.
Der skal vre mekanismer tilstede, der kan inaktivere transmitterstoffet efter frigivelse.
1.2.2. Angiv seks lavmolekylre transmitterstoffer og for hver af dem en nervecelletype eller en
fiberprojektion, hvor de findes.
De mest kendte "klassiske", lavmolekylre stoffer er:
Glutamat: Findes overalt i CNS og i spinalganglier i PNS.
GABA: Findes overalt i CNS og i maven i PNS.
Acetylcholine: Findes i cortex cerebri og i prganglionre, autonome neuroner.
Norepinephrin: Findes i sympatiske ganglier.
Dopamin: Findes overalt i CNS, isr i basalganglierne.
Serotonin: Findes overalt i CNS.

1.2.3. Angiv de lavmolekylre transmittersubstanser i centralnervesystemet, opdelt efter kemisk


type.
Glutamat: Aminosyrer.
GABA (-Aminobutyric acid): Aminosyrer.
Glycin: Aminosyrer.
glutamin: Aminosyre
Histamin: Amin
Acetylcholine: Amin.
Norepinephrin: Amin.
Dopamin: Amin.

Opgaver side 3

Serotonin: Amin.
1.2.4. Hvad forsts ved neuroaktive peptider? Nvn to af disse og giv et eksempel p deres
cellulre forekomst.
Neuroaktive peptider er sm peptidmolekyler, som neuroner kan bruge til at kommunikere med
hinanden.
Eksempler kunne vre:
Vasoactive intestina polypeptide (VIP): Udskilles sammen med acetylcholin i spytkirtlerne. VIP
inducerer vasodilation og ger blodflowet til spytkirtlen, mens den secernerer spyt.
Substance P: Frigives fra primre, sensoriske neuroner sammen med glutamat og har en
indflydelse p opfattelsen af smerte.
1.2.5. Nvn to aminosyrer, som er neurotransmittere i CNS, og angiv deres typiske
receptorvirkning.
Glutamat og GABA er begge aminosyrer.
Glutamat virker hurtigt og langsomt excitatorisk p neuroner via sine receptorer AMPA (ionotrop,
koblet til natrium), NMDA (ionotrop, koblet til calcium) og mGluR-1-5 (metabotrop receptor, der
ogs kan virke inhibitorisk.).

GABA virker bde hurtigt og langsomt inhibitorisk p neuroner via sine receptor GABA(A), der er
ionotrop og koblet til chlorid, og GABA(B), der er metabotrop og koblet til kalium og calcium.
1.2.6. Hvilket kemisk stof er GABA forkortelsen for?
GABA str for -Aminobutyric acid, p dansk kendt som gamma-aminosmrsyre.
1.2.7. Hvordan virker neurotransmitteren GABA, og hvordan ophrer dens virkning p
receptorerne?
GABA er den dominerende inhibitoriske transmitter og findes nsten overalt i CNS. GABA bruges
primrt af interneuroner.
GABA virker via GABA(A), der er ionotrop og koblet til chlorid, eller via GABA(B), der er metabotrop
og koblet til kalium og/eller calcium. Ved binding af GABA til enten GABA(A) eller GABA(B)
hyperpolariseres membranen og dette virker inhibitorisk p neuronets evne til at danne et
aktionspotentiale.
GABA fjernes fra den synaptiske spalte via transporterer (GAT), der primrt findes i NEURONERNES
membraner (og alts ikke i s hj grad i glia-cellerne).
1.2.8. Redegr kort for aminerge nervetransmitteres modulerende effekt p aktiviteten af andre
neuroner.
Aminerge nervetransmitterer medierer ikke prcise informationer om tid og sted, de modulerer
funktionerne af specifikke glutamerge eller GABAerge systemer, f.eks. ved at forbedre signal-tonoise-forholdet.
Med andre ord ger de alts prcisionen af den informationsbearbejdning, som finder sted mange
steder i hjernen - f.eks. i hjernebarken.

Opgaver side 4

1.3. Kanaler:
30. januar 2014
21:15

1.3.1. Hvor finder man isr spndingsstyrede natriumkanaler p neuronet, og hvilken virkning
har det?
Isr i det initielle segment af axonet er der mange natriumkanaler, men der findes mange kanaler i
axonet i det hele taget. Det betyder, at signaler opfanget af dendritterne og propageret i retning af
axonet, vil have en bedre chance for at skabe basis for et aktionspotentiale, da de lettere vil kunne
depolarisere plasmamembranen og bne de spndingsafhngige natriumkanaler.

Hvis aktionspotentialet skal dannes et andet sted i neuronet skal depolariseringen vre strre, da
antallet af spndingsafhngige natriumkanaler er mindre end i det initiale segment.
1.3.2. Hvad forsts ved ligevgtspotentialet for kaliumioner?
Ligevgtspotentialet for kalium er det membranpotentiale, hvor der ikke er noget nettoflow af
kaliumioner over membranen. Ligevgtspotentialet er normalt omkring -75 mV.
1.3.3. Forklar, hvordan en transmitterstyret bning eller lukning af kaliumkanaler vil pvirke
nervecellens excitationstrskel.
En bning af kaliumkanaler vil medfre efflux af kalium ud i ECF pga. kaliums store
koncentrationsgradient. Dette vil hyperpolarisere cellen.
En lukning af kanaler vil medfre retention af kalium i ICF, hvilket vil medfrer depolarisering af
cellemembranen pga. den gede ophobning af kationer.
1.3.4. Hvad forsts ved ligevgtspotentialet for chloridioner?
Ligevgtspotentilet for chlorid er det membranpotentiale, hvor der ikke er noget nettoflow af
kaliumioner over membranen. Ligevgtspotentialet for chlorid er normalt omkring -65 mV.
1.3.5. Forklar, hvordan en transmitterstyret bning af chloridkanaler pvirker nervecellernes
excitabilitet.
bning af chloridkanaler vil ikke have den store effekt p nervecellens excitabilitet, da den kemiske
gradient vil drive chlorid ind i cellen med en lige s stor hastighed som den elektriske gradient driver
chlorid ud.
I nogle celler vil membranen dog hyperpolariseres, da chlorid drives IND. Dette ses f.eks. ved bning
af GABA(A)-receptorer.

1.3.6. Angiv en neurotransmitter, der bner chloridkanaler.


Ved binding af GABA-molekyler til GABA(A)-receptorer vil der bnes chloridkanaler. Dette vil virke
inhibitorisk p et neuron, idet neuronmembranen vil hyperpolariseres.
1.3.7. Forklar kort, hvordan bning af ionkanaler for chloridioner og lukning af ionkanaler for
kalium-ioner pvirker en nervecelles hvilepotentiale.
bning af chloridkanaler vil medfre influx af chlorid pga. chlorids meget store kemiske gradient.
Dette vil danne et IPSP og hyperpolarisere cellen. Det bliver svre at excitere cellen!
Lukning af kaliumokanaler vil medfre retention af kalium i cellen og depolarisering.

Opgaver side 5

1.4. Receptorer:
30. januar 2014
21:16

1.4.1. Receptorer for neurotransmitterer findes ofte i flere undertyper. Angiv to eksempler p
sdanne undertyper og deres funktionelle karakteristika.
Glutamatreceptorerne bruges som eksempel. Der er overordnet tre:
AMPA: Hurtig og ionotrop receptor, der virker excitatorisk ved at bne for natriumkanaler ved
binding af glutamat.

NMDA: Hurtig og ionotrop receptor, der virker excitatorisk ved at bne for calciumkanaler ved
binding af glutamat.
mGluR1-5: Langsomme og metabotrop receptorer, der virker enten excitatoriske eller
inhibitoriske.
1.4.2. Hvad er forskellen p ionotrope og metabotrope receptorer?
Der er overordnet to slags receptorer: Ionotrope og metabotrope. De fleste neurotransmitterer
virker p begge slags.
En ionotrop receptor er en del af ionkanalen og pvirker derfor direkte det funktionelle stadie af
kanalen (dvs. om den er bnet eller lukket). Transmitterstoffer, der virker via ionotrope receptorer,
virker ofte hurtigt og prcist. Et eksempel p sdan information kunne vre om "hvornr", "hvad"
og "hvor", hvis et sensorisk stimulus blev transmitteret.
En metabotrop receptor er koblet indirekte til en ionkanal via G-proteiner og intracellulre secondmessengers. Deres effekter er derfor langsommere om at starte, men varer til gengld lngere.
Metabotrope receptorer virker modulerende, fordi de justerer excitabiliteten af de postsynaptiske
neuroner - de danner alts ikke selv et aktionspotentiale, men gr cellen mere eller mindre flsom
overfor andre kanaler. Metabotrope celler har muligvis indflydelse p neuronernes vkst.
1.4.3. Angiv funktionen af to ionotrope glutamatreceptorer og virkningen af aktiveringen af
metabotrope glutamatreceptorer.
De to mest kendte ionotrope glutamatreceptorfamilier er AMPA og NMDA.

AMPA virer hurtigt og excitatorisk via bning af natriumkanaler. NMDA virker hurtigt og excitatorisk
via bning af calciumkanaler. Begge dele vil depolariserer cellen.
Den mest kendte metabotrope glutamatreceptor er MGluR1-5, der kan virke enten excitatoriske
eller inhibitoriske. De kan alts enten depolariserer eller hyperpolariserer cellen.
1.4.4. For neurotransmitteren glutamat kendes flere undertyper af receptorer, som f.eks. findes i
hippocampus. Nvn fire af disse receptortyper, og redegr kort for de forskellige receptortypers
virkemde.
Der findes overordnet tre slags glutamatreceptorer:
AMPA-familien: Ionotrop. Virker ved bning af natriumkanaler. Virker depolariserende.
NMDA-familien: Ionotrop. Virker ved bning af calciumkanaler. Virker depolariserende. Da
calcium har virkninger intracellulrt har NMDA en rolle i f.eks. long-term potentiation, hvor
synapserne ndres.
mGluR 1-5: Findes i tre undergrupper, der varierer mht. den intracellulre signalvej de
Opgaver side 6

mGluR 1-5: Findes i tre undergrupper, der varierer mht. den intracellulre signalvej de
benytter, der ultimativt enten giver langsomme EPSP'er eller langsomme IPSPer.
1.4.5. Angiv de mest kendte undertyper af glutamat- og GABA-receptorer.
De mest kendte undertyper af glutamatreceptorerne er:
AMPA-familien: Ionotrope receptorer, der bner natriumkanaler. Virker depolariserende.
NMDA-familien: Ionotrope celler, der virker ved bning af calciumkanaler. Virker
depolariserende.
mGluR 1-5: Findes i tre undergrupper, der varierer mht. den intracellulre signalvej. De giver
enten langsomme EPSP'er eller langsomme IPSP'er.
De meste kendte GABA-receptorer er:
GABA(A): Ionotrop receptor, der bner chloridkanaler. Virker hyperpolariserende.
GABA(B): Metabotrop receptor, der bner kalium- og/eller calciumkanaler. Virker
hyperpolariserende.
1.4..6 Angiv den postsynaptiske funktion af to GABA-receptorer.
De to vigtigste GABA-receptorer er:

GABA(A): bner hurtigt postsynaptiske chloridkanaler, hvilket hyperpolariserer membranen.


Der dannes alts et hurtigt IPSP.
GABA(B): bner langsomt postsynaptiske kalium- og/eller calciumkanaler via secondmessengers fra et G-protein. Dette medfrer et langsomt, modulerende IPSP.
1.4.7. Forklar funktionen af prsynaptiske transmitterreceptorer, f.eks. p prsynaptiske
acetylcholinreceptorer koblet til calciumkanaler.
Transmitterreceptorer der er placeret prsynaptisk kan modulerer transmitterfrigivelse, f.eks. via
autoreceptorer. Autoreceptorer inhiberer transmitterfrigivelse via negativ feedback.
Hvis f.eks. koncentratioen af acetylcholin i den synaptiske spalte bliver s hj, at acetylcholin
begynder at binde sig til den prsynaptiske autoreceptor, s vil exocytosen af acetylcholin-holdige
vesikler falde.
Heteroreceptorer er en anden slags prsynaptisk receptor. De er receptorer for transmitterstoffer,
der kommer fra andre neuroner - dvs. transmitterstoffer, som neuronet ikke selv kan producere.
Dette kan vre relevant for neuroner, der skal kommunikere med andre transmitterstoffer.

Opgaver side 7

1.5. Impulsledning:
30. januar 2014
21:16

1.5.1. Redegr kort for den betydning en myelinisering har p ledningen af nerveimpulser.
Myelinisering ger propageringshastigheden for nerveimpulset med en faktor 2-120.
Dette skyldes:
Nerven bliver bedre isoleret rent elektrisk.
Nervens interne resistans nedsttes med en myelinskede (pga. den gede diamater)
Aktionspotentialerne, der sendes igennem nerven, skal kun gendannes ved de Ranvierske
indsnringer. Aktionspotentialet kan sledes "springe" fra indsnring til indsnring.
1.5.2. Angiv den typiske cellulre fordeling af spndingsstyrede ion-kanaler p et neuron med
myeliniseret axon, hvilken ioner, det drejer sig om ved hver placering, samt hvilke funktioner disse
kanaler har.
Typisk er koncentrationen af spndingsstyrede natrium-kanaler strst ved overgangen fra soma til
axon (axon-hillock/initial segment). Det er disse kanaler, der er ansvarlige for dannelsen af
aktionspotentialet. Sdan kanaler vil depolarisere cellen. Der findes ogs mange excitatoriske
kanaler i dendritterne.
Der er mange hyperpolariserende kanaler omkring axon hillock. Disse kan fungere som en bremse
for aktionspotentialerne.
1.5.3. Beskriv, hvad der betinger en nervecelles refraktrperiode, og hvad der er dens funktion.
Nervecellens refraktrperiode har som forml at give nervecellen tid til at genoprette de
ndvendige gradienter for ionerne, sdan s der igen kan fyres et aktionspotentiale. Der er isr tale
gendannelse af natrium- og kaliumgradienterne.
Refraktrperioden skyldes for det frste, at de spndingsafhngige natriumkanaler inaktiveres, nr
aktionspotentialets amplitude peaker. Dette betyder, at der ikke kan komme mere natrium ind i
cellen og den derfor ikke kan depolariseres mere. Kanalerne er lukkede i et bestemt tidsrum, hvor
cellen alts IKKE kan fyre et aktionspotentiale, lige meget hvor stor en stimulus den modtager.
Dette er den absolutte refraktrperiode.
Refraktrperioden betinges ogs af fungerende, spndingsafhngige kaliumkanaler, der bner, nr
membranpotentialet nr en bestemt vrdi. bningen af kanalerne medfrer et efflux af kalium ud af
cellen, hvilket hyperpolariserer cellemembranen til en vrdi under hvilepotentialet, fordi
kaliumkanalerne er bne lngere tid end ndvendigt. Nr natriumkanalerne igen er bne, men
cellen stadig er hyperpolariseret, kan cellen godt fyre et AP - det krver dog en strre depolarisering
end normalt.
Dette er den relative refraktrperiode.
1.5.4. Gr rede for, hvorledes aktionspotentialet propageres i et axon, og angiv hvilke forhold, der
har betydning for propageringshastigheden.
Den vigtigste faktor, der afgr et axons propageringshastighed, er om axonet er myelineret. Et
myelineret axon leder impulser optil 120 gange hurtigere end et umyelineret.
Bde det myelinerede og det umyelinerede axon taber spnding under impulsledningen, hvilket
medfrer, at impulset dr ud, hvis ikke det gendannes. I et umyelineret axon gendannes impulset
kontinueret i en cyklus af depolariseringer og repolariseringer henover cellemembranen.
Opgaver side 8

kontinueret i en cyklus af depolariseringer og repolariseringer henover cellemembranen.


I et myelineret axon foregr gendannelsen kun i de ranvierske indsnringer. Impulsen kan derfor
"springe" fra indsnring til indsnring, hvilket ger propageringshastigheden.
1.5.5. Tegn et koordinatsystem med membranpotentialet (mV) som ordinat og tiden (msek) som
abscisse og illustrer flgende fnomener: Synaptisk inhibition, synaptisk excitation og synaptisk
temporal summation.
Billedet ses her:

Synaptisk inhibition ville betyde, at hvilemembranpotentialet hyperpolariseres, sdan s det bliver


svrer at depolariserer cellen til en threshold-vrdi og at dannelsen af et aktionspotentiale dermed
hindres.
Synaptisk excitation er det modsatte: En depolarisering af hvilemembranpotentialet, der gr det
nemmere for andre neuroner at depolariserer cellen og dermed skabe et aktionspotentiale.
Det er vigtigt at bemrke, at det er summen af alle synaptiske input til cellen, der bestemmer om
der dannes et aktionspotentiale - den skaldte synaptiske summation.
1.5.6. Angiv, hvad der forsts ved henholdsvis temporal og spatial summation af excitatoriske
og/eller inhibitoriske postsynaptiske potentialer.
Temporal summation er en summationsform, hvor en hj frekvens af aktionspotentialer til den
prsynaptiske membran faciliterer postsynaptiske potentialer, der overlapper eller forstrker
hinanden.
Spatial summation er summeringen af input overalt fra nervecellen (dendritter, soma, axoaxotiske
synapser osv.) og er en anden mde, hvorp man kan opn et aktionspotentiale ved summation.
Hvor den temporale summation alts har mere med hastigheden (som i impulser pr. tid) af
indkommende aktionspotentialer at gre, s har den spatiale summation mere med mngden af
impulser at gre.
1.5.7. Et neuron integrerer de til enhver tid indkommende eksitatoriske og inhibitoriske synaptiske
input, som i varierende grad kan resulterer i initieringen af et AP. Redegr kort for hvilke
ionstrmme over neuronets cellemembran og forandringer i membranpotentialet, der typisk er
afgrende for, om der initieres et aktionspotentiale eller ej.
Det er influx af natrium, der er srlig vigtig for dannelsen af et aktionspotentiale - srligt influx af
Opgaver side 9

Det er influx af natrium, der er srlig vigtig for dannelsen af et aktionspotentiale - srligt influx af
natrium gennem en spndingskoblet natriumkanal. Natriumkanalen skal depolariseres til en hvis
vrdi, fr den bner og tillader influx af natrium og en eksplosiv stigning i cellens
membranpotentiale og derfor muligheden for at danne et aktionspoteniale.
Hvis synaptiske input hyperpolariserer cellen (inhibitoriske input) vil det mindske sandsynligheden
for dannelsen af et aktionspotentiale, idet kanalen bringes lngere vk fra dens bne tilstand. Hvis
synaptiske input depolariserer cellen (eksitatoriske input) vil det bringe kanalen tttere p den bne
tilstand og derved vil cellen vre tttere p at ge sin natriuminflux og fyre et aktionspotentiale.
Det er summen af alle de eksitatoriske og inhibitoriske inputs, som cellen modtager, der bestemmer,
om de spndingsafhngige natriumkanaler bner og om cellen fyrer et AP.
1.5.8. Ift. sine forskellige afferente forbindelser beskrives nervecellen ofte som en integrator,
mens det efferente output (aktionspotentialet) er et "alt eller intet"-fnomen. Forklar kort, hvori
integrationen funktionelt set bestr.
Det er ndvendigt for et neuron et integrere synaptiske input fra andre neuroner for summere en
stor nr depolarisering til at fyre et aktionspotentiale. Jo strkere de eksitatoriske effekter er, desto
kortere tid vil der g fr cellen er depolariseret til en treshhold-vrdi og kan fyre et AP. Dette AP vil
dog altid have den samme amplitude - den er derfor alt eller intet. Enten skydes et AP, eller gr der
ikke. Der kan ikke skydes et "svagt" AP.
Det betyder, at frekvensen af aktionspotetialer er bestemt af "total synaptic input" til et neuron.
Total synaptic input er et udtryk for summen af bde excitatoriske og inhibitoriske signaler, som
neuronet har integreret.
1.5.9. Hvorfor har aktivering af en inhibitorisk synapske placeret p axonets initiale segment alt
andet lige strre effekt end en tilsvarende synapse placeret p en dendrit.
Fordi synapser placeret i det initiale segment er neuronets sidste chance for at hmme dannelsen af
et aktionspotentiale, hvis disse synapser virker hyperpolariserende.
Hvis forbindelsen til synapser i det initielle segment afbrydes og der dermed ikke kan sendes
inhibitoriske signaler til segmentet, s vil sandsynligheden for fyring af et AP alt andet lige vre
strre.

Opgaver side 10

1.6. Synaptisk transmission:


30. januar 2014
21:16

1.6.1. Hvordan udlser aktionspotentialet transmitterfristning i synapsen?


Aktionspotentialet depolariserer den prsynaptiske membran, hvilket leder til bning af
spndingsafhngige calciumkanaler.
bningen af kanalerne tillader et flow af calcium ind i boutonen. Den gede calciumkoncentration i
membranen medfrer exocytose af vesikler, der indeholder transmitter. Disse vesikler str klar i en
fri pulje, meget tt p den prsynaptiske membran.
1.6.2. Redegr kort for, hvad der forsts ved natrium, kalium og chlorids' ligevgtspotentialer
over neuroners cellemembran og hvordan man ud fra kendskab til de enkelte ioners
ligevgtspotentiale ift. et givet neurons hvilepotentiale, kan forudsige den excitatoriske eller
inhibitoriske virkning af bning og lukning af postsynaptiske ligand-styrede ionerkanaler for de tre
ioner.
Et ligevgtspotentiale for et ion er det membranpotentiale, hvor der ikke er noget nettoflow af
ionen over membranen. For natrium, kalium og chlorid er det:
Natrium: +55 mV
Kalium: -75 mV
Chlorid: -65 mV
Kendskab til de enkelte ioners ligevgtspotentiale er vrdifuldt, da man via ligevgtspotentialet for
ionen og hvilemembranspotentialet for cellen kan bestemme, om et ion vil bevge sig ind eller ud af
cellen, hvis cellemembranen bliver mere permeabel overfor ionen.
Hvis f.eks. cellemembranen blev mere permeable for kalium, ville det medfre efflux af kalium,
hvordi kalium ville bevge sig med sin kemiske gradient ud af cellen. Natrium ville derimod bevge
sig ind i cellen, pga. den store elektrokemiske gradient.
1.6.3 Redegr kort for hvilke naturlige mekanismer, der findes til at afslutte en transmitters
virkning efter frigivelse fra nerveterminalerne.
Neurotransmitterer skal inaktiveres. Dette sker p flere mder.

Diffusion: For det frste, vil transmitteren simpelthen diffundere vk fra synapsen.
Nedbrydning af enzymer: Nogle neurotransmitterer (f.eks. acetylcholin) nedbrydes af
specifikke enzymer (f.eks. acetylcholin-esterase).

Genoptag i glia-celler og neuroner: En stor del af transmitterstoffer bliver optaget i gliaceller


eller i neuroner. Specielle proteiner i cellemembranerne medierer dette genoptag. Der er to
familier af transporterer, der varierer efter, hvilke stof de genoptager (hhv. en familie, der
genoptager langt de fleste transmitterer og en familie, der hovedsagligt genoptager glutamat).
1.6.4. Redegr i rkkeflge kort for de mekanismer og processer, der er involverede fra
ankomsten af et AP til en glutamaterg nerveterminal og til udlsningen af et lokalt EPSP,
herunder, hvordan receptoren virker og fjernelsen af glutamat fra den synaptiske spalte.
Processen kan opdeles i flgende trin:
1) Ankomst af AP til nerveterminal.
2) Depolarisering af prsynaptisk membran.
3) bning af spndingsafhngige calciumkanaler.
Opgaver side 11

3)
4)
5)
6)
7)
8)
9)
10)

bning af spndingsafhngige calciumkanaler.


get influx af calcium og dermed stigende calciumkoncentration i boutonen.
Calciummedieret exocytose af vesikler indeholdende glutamat.
Glutamat frigivet til den synaptiske spalte fr kontakt til receptorer (f.eks. AMPA)
postsynaptisk.
bning af natriumkanaler via AMPA (og muligvis ogs bning af kanaler via metabotrope
receptorer).
Depolarisering af den postsynaptiske membran pga. get natriuminflux.
Dannelse af et excitatory postsynaptic potential (EPSP).
Fjernelse af glutamat fra den synaptiske spalte. Glutamat genoptages i glia-celler og neuroner
via en familie af transporterer, der udnytter koncentrationsgradienterne for natrium og/eller
kalium. Desuden diffunderer noget af glutamatet vk.

I gliacellerne omdannes glutamat til glutamin, der overfres til neuronet, hvor det igen omdannes til
glutamat, der kan genbruges som neurotransmitter.
1.6.5. Angiv mekanismen for fjernelse/neutralisering af glutamat fra synapsespalten.
Glutamat fjernes via diffusion vk fra den synaptiske spalte. Desuden genoptages glutamat i
neuroner og gliaceller via en familie af glutamattransporterer, der udnytter natriums og kaliums
koncentrationsgradienter til at transporterer glutamat tilbage i cellerne.
Glutamat der genoptages i neuroner bliver hurtigt pakket i vesikler igen, mens glutamat, der
genoptages i glia-celler omdannes til glutamin via glutamin-synthetase. Den dannede glutamin
transporteres til neuronets mitochondria, hvor omdannes til glutamat via glutaminase.

Alts: Glutamat -> glutamin-synthetase -> glutamin -> glutaminase -> glutamat
Glutamat-glutamin-kredslbet sikrer genbrug af transmitteren og desuden sikrer det, at neuronets
excitabilitet og helbred ikke pvirke, idet glutamin ikke er lige s giftigt som glutamat.
1.6.6. Redegr i rkkeflge for de mekanismer og processer, der er involverede fra ankomsten af
et aktionspotentiale til en GABAerg nerveterminal og til udlsningen af et lokalt IPSP, herunder
hvordan receptorer virker og ophr af GABAs virkning fra den synaptiske spalte.
Processen kan opdeles i flgende trin:
1)
2)
3)
4)
5)
6)
7)
8)
9)
10)

Ankomst af AP til nerveterminal.


Depolarisering af prsynaptisk membran.
bning af spndingsafhngige calciumkanaler.
get influx af calcium og dermed stigende calciumkoncentration i boutonen.
Calciummedieret exocytose af vesikler indeholdende GABA.
GABA frigivet til den synaptiske spalte fr kontakt til receptorer (f.eks. GABA(A)) postsynaptisk.
bning af chloridkanaler via GABA(A).
Hyperpolarisering af den postsynaptiske membran pga. get chloridinflux.
Dannelse af et inhibitory postsynaptic potential (IPSP).
Fjernelse af GABA fra den synaptiske spalte. GABA fjernes af specifikke transportproteiner
(GAT), der genoptager GABA til neuronet.

1.6.7. Angiv mekanismen for fjernelse/neutralisering af GABA fra synapsespalten.


GABA fjernes fra synapsespalten af specifikke transportproteiner (GAT), der genoptager GABA
primrt til neuronet.
P denne mde adskiller genoptagelsen af GABA sig fra genoptagelsen af glutamat, da glutamat
primrt optages til glia-celler.
1.6.8. Indenfor neurofysiologien opererer man med begrebet "pair pulse facilitation" svarende til,
Opgaver side 12

1.6.8. Indenfor neurofysiologien opererer man med begrebet "pair pulse facilitation" svarende til,
at det andet af to lige efter hinanden udlse (og i vrigt ens) aktionspotentialer genererer et
kraftigere postsynaptisk respons end det frste. Angiv den prsynaptiske rsag hertil.
Nr aktionspotentialer nr den samme nerveterminal med relativt korte intervaller vil amplituden af
det dannede postsynaptiske signal ofte ges. Dette kaldes facilitation.
Facilitation skyldes, at der frigives mere transmittersubstans for hver prsynaptisk potentiale. Den
postsynaptiske effekt stiger for hvert aktionspotentiale indtil steady state opns.
1.6.9. Angiv nogle muligheder for funktionel modulation svarende til den enkelte synapse.
Monoaminerge og cholinerge cellergrupper medierer ikke prcis temporal eller spatial information,
men modulerer derimod funktionerne i GABA- eller glutamatsystemer.
Dette skyldes blandt andet, at aminerne virker relativt uspecifkt (via volume transmission) og derfor
kan modulere flere synapser relativt diffust. Desuden virker aminerne primrt via metabotrope
receptorer, der udver langesomme, modulerende effekter p neuronerne og regulerer deres
excitabilitet.
1.6.10. Beskriv det ultrastrukturelle udseende af en synapse.
En prsynaptisk membran og en postsynaptisk membran er adskilt af en smal spalte, den skaldte
synaptiske klft.
Den prsynaptiske membran er besat med autoreceptorer og proteiner, der skal genoptage afgivet
transmitterstof.
Den postsynaptiske membran er besat med receptorproteiner, der skal binde transmitterstofferne,
sdan s de kan udve deres effekt.

1.6.11. Angiv mindst tre faktorer, som i en given synapse - med det samme teststimulus af den
prsynaptiske fiber - kan medfre en ndret synaptisk transmission.
Der kan vre flere faktorer:
Mngden af oplagret transmittersubstans i den prsynaptiske celle.
Mngden af transportproteiner prsynaptisk, der kan genoptage afgivet transmitterstof
(frre transporterer betyder mere transmitterstof i den synaptiske klft).
Mngden af receptorer for transmitterstoffet i den postsynaptiske membran.
Mngden af autoreceptorer, der virker p den prsynaptiske membran og dermed
mulighederne for feedback p neuronet.
Mngden/konfigurationen af kanaler, der kan virke postsynaptisk.
1.6.12. Angiv 3 principielle muligheder for farmakologisk/medicinsk pvirkning af den synaptiske
transmission.
Der kan vre flere:

Inhibering af postsynaptiske receptorer.


Hmning af transmitterfrigivelse fra den prsynaptiske celle.
Antagonister mod de enzymer eller proteiner, der skal inaktivere eller genoptage
transmittersubstans.
Lgemidler, der kan binde til de samme receptorer som neurotransmitterne.
Opgaver side 13

Lgemidler, der kan binde til de samme receptorer som neurotransmitterne.


1.6.13. Redegr kort for fnomenet "long term potentiaton" og den funktionelle betydning for
LTP.
Long-term potentation betyder, at den strukturelle opbygning af en synapse kan ndres i lngere
tid, hvis en synapse er aktiv samtidigt med andre synapser i nrheden. LTP afhnger mange steder
af aktivering af NMDA-receptorer.
Nr vi lrer sker der formentligt det, at flere synapser ndrer deres struktur og virkning inde i et
neuralt netvrk. Dette tillader os at genkalde minder.

Opgaver side 14

Kapitel 2: Hjernens hinder og kar


29. januar 2014
09:29

Opgaver side 15

2.1. Hjernehinder:
31. januar 2014
11:49

2.1.1. Angiv i rkkeflge, hvilke lag, rum og potentielle spalterum, man passerer fra kalvariets
knogle og ind til nervevvet i cortex cerebri.
Rkkeflgen vil vre:
1. Epiduralrummet.
2. Dura mater, der er forbundet til kalvariets periost.
3. Det subdurale rum (et potentielt spalterum, der kan blive et egentligt rum, hvis det fyldes med
blod eller vske)
4. Arachnoidea mater
5. Det subarachnoidelle rum (med CSF)
6. Pia mater
Herefter vil man vre ved nervevvet.
2.1.2. Angiv de typer af kar, der karakteristisk findes i hinderne eller rummene.
I pia mater findes der kapillrer, der brer blod til den externe del af hjernen (primrt cortex).
I ventriklerne findes der plexus choroideus, der er karslynger, der danner ultrafiltrat af blodet og
producerer CSF.
I relation til dura mater findes vense sinuser.
2.1.3. Hvis en kanyle stikkes igennem kalvariet i midtlinjnen svarende til sutura sagitalis og skrt
nedad og lateralt, angiv da, hvilke strukturer og potentielle og reelle hulrum, kanylen vil passere i
forlbet fra knogleoverfladen p kalvariets inderside og til den nr hjernevvet.
Flgende strukturer vil passeres:
1.
2.
3.
4.
5.
6.
7.
8.

Epiduralrummet.
Dura mater m. sinus sagitalis.
Det subdurale rum (et potentielt spalterum)
Arachnoidea mater
Det subarachnoidelle rum ( hulrum med CSF)
Pia mater
En lateral ventrikel (et hulrum med CSF)
Nervevv.

Herefter vil man ramme nervevv.


2.1.4. I hvilke strukturer er medulla spinalis ophngt i hvirvelsjlens canalis spinalis.
Medulla spinalis er ophngt i ca. 21 par ligg. denticulata og desuden ogs i spinalnerverne.

Opgaver side 16

2.2. Cerebrospinalvske:
31. januar 2014
11:50

2.2.1. Angiv i hvilke strukturer cerebrospinalvsken dannes, og hvilke rum og strukturer den
passerer fra lateralventriklerne til venesystemet.
Cerebrospinalvsken dannes i karfletninger kaldet plexus choroideus, der str i forbindelse til
hjernens ventrikler via tela choroidea. Den strste mngde vske dannes i lateralventriklerne.
Vsken produceret i de laterale ventrikler flyder til den tredje ventrikel i diencephalon via foramen
interventriculare.
Fra den tredje ventrikel flyder vsken via den cerebrale akvdukt til den fjerde ventrikel i pons og
medulla oblongata. Der tilsttes yderligere CSF i begge ventrikler.
I den fjerde ventrikel flyder vsken ud af ventrikelsystemet til cisterna magna i det subarachnoidale
rum via bninger i ventriklen (foramen Magendie og foramina Luschka) eller ned i rygmarven via
canalis centralis.
CSF drnes til de vense sinuser via arachoidea-villi, der danner arachnoidea granulationer, der
ligger i tt relation til isr sinus sagittalis superior.
Lateralventriklerne -> foramen interventriculare -> trejde ventrikel -> aquaductus cerebri -> fjerde
ventrikel -> forarmen Magendie og foramina Luschka -> subarachnoidalrummet.
2.2.2. Beskriv kort lokalisation, opbygning og funktion af plexus choroideus:
Plexus choroideus er karfletninger, der findes i alle ventriklerne. De strste mngde findes dog i de
laterale ventrikler.
Plexerne er dannet ved indvoksning af pia mater i neuralrret, hvorved der dannes villi, der
indeholder fenestrerede kapillrer.
Overfladen er dkket af kubisk epithel, som hnger sammen med ependymalceller p ventriklens
inderside.
Det hydrostatiske tryk i et plexus choroideus presser ultrafiltrat ud i villi. Filtratet kan ikke forlade
villi pga. tight junctions. Transporten af filtratet sker i stedet ved en aktiv proces, hvor natrium
pumpes ud i ventriklerne og filtratet flger efter igennem AQP-proteiner.
Funktionen af plexus choroideus er alts at danne ultrafiltrat af blodet, der kan bruges som
cerebrospinalvske i CNS.

2.2.3. Angiv de ndringer i hjernen, som en aflbshindring opstet under forsterudviklingen kan
forrsage.
En aflbshindring af CSF fra kraniet vil resulterer i hydrocephalus og et intrakranielt tryk.
Under forsterudvikligen, hvor kraniets suturer ikke er lukkede, vil hovedet p barnet vokse
abnormalt, fordi knoglerne giver efter for det gede tryk.
Dette kan resulterer i anomalier som bdkranie eller trnkranie, afhngigt af, hvile vkstlinjer i
kraniet, som knoglerne vokser efter.

Opgaver side 17

2.3 Blodforsyning:
31. januar 2014
11:50

2.3.1. Beskriv kort opbygning og funktion af blod-hjerne-barrieren.


Blod-hjerne-barrieren holder blodet adskilt fra hjernens ekstracellulre milj.
Hvis det ekstracellulre milj omkring hjernen blev ndret af solutter eller neuroaktive stoffer i
blodet ville det medfre ndringer i neuronexcitabilitet, hvilket ville vre problematisk for
neuronernes virke, derfor er blodet ndt til at vre adskilt fra hjernen.

Der er flere faktorer, der medvirker til at gre blod-hjerne-barrieren speciel:


Hjerne-kapillrer er meget mindre permeable end andre kapillrer over for sm molekyler dette skyldes meget ttte tight-junctions.
Der er kun f endocytotiske vesikler i cytoplasma af endothelcellerne i CNS.
Hjernens endothelceller er udstyrret med P-glycoprotein, der kan udskille lipid-oplselige
molekyler, der kommer igennem blod-hjerne-barrieren.
2.3.2. Gr kort rede for den arterielle karforsyning af medulla spinalis.
Medulla spinalis arterieforsyning udgres af tre arterier:
En medial a. spinalis anterior placeret ventralt.
To mere lateralt orienterede a. spinalis posterior placeret dorsalt p medulla.
2.3.3. Gr kort rede for den arterielle karforsyning af medulla oblongata.
Medulla oblongata forsynes fra tre arterier:
Fortil lateralt: a. vertebrales.
Fortil medialt og posteriort: a. spinalis anterior.
Posteriort og lateralt: a. cerebelli inferior posterior .
2.3.4. Gr kort rede for den arterielle karforsyning af pons.
Flere grene forsyner pons:

Medial del: Paramediane grene fra a. basilaris.


Anterolateral del: Korte circumferente grene fra a. basilaris.
Posterolateral del: Lange circumferente grene fra a. basilaris.
2.3.5. Gr kort rede for den arterielle karforsyning af mesencephalon.
Flere grene forsyner mesencephalon:
Medial del: Paramediane grene fra a. basilaris.
Anterolateral del: Korte cirkumferente grene fra a. basilaris.

Opgaver side 18

Posterolateralt: Lange cirkumferente grene fra a. basilaris, a. cerebri posterior og a.


cereballaris superior.
2.3.6. Forklar kort forskellen p det funktionelle udfald fremkaldt af okklusion af a. spinalis
anterior og en a. cerebellaris inferior posterior.
Frst betragtes en okklusion af a. spinalis anterior:
A. spinalis anterior forsyner den mediale, anteriore del af medulla spinalis og den mediale del af
medulla oblongata.
Beskadigelse af a. spinalis anterior vil give medialt medullrt syndrom. Dette kan medfre f.eks.
skader p n. hypoglossus (XII), hvilket vil medfre ipsilateral parese af tungens muskler. Test: At
rkke tunge. Tunge vil deviere til syg side.
Der vil kunne vre skader p pyramidebanen. Dette vil medfre kontralateral spastisk lammelse af
ekstremiteterne, hvis skaden er over krydsningen, der findes i den caudale del af medulla oblongata.
Under krydsningen lammelsen vre ipsilateral.
Der vil kunne vre skader p lemniscus medialis, hvilket vil kunne medfre kontralateralt tab af
diskriminationssans, flesans og vibrationssans - dvs. udfald svarende til BLM.
Nu betragtes en okklusion af a. cerebellaris inferior posterior:
A. cerebelli inferior posterior forsyner den posterolaterale del af medulla oblongata og de inferiore
dele af cerebellums hemisfrer.
Beskadigelse af a. cerebelli inferior posterior vil give lateralt medullrt syndrom. Dette betyder
skader p f.eks. tr. spinothalamicus (det anterolaterale system), hvilket vil medfrer kontralateral
tab af smerte og temperatursans p kroppen.
Der vil kunne vre skader p nc. trigeminale spinalis (sensorisk kerne for V), hvilket vil kunne
medfre ipsilateral tab af smerte og temperatursans i ansigtet.
Der vil kunne vre skade p nc. ambiguus, der indeholder de efferente kerner for n. vagus og n.
glossopharyngeus. Dette vil kunne give dysphagi, hshed, nedsat brkrefleks og lammelse af den
blde gane. Test: Se p del blde gane mens patienten siger "ah". Ganen vil ikke hve sig i syg side.
Der vil kunne vre skade p nc. vestibularis, hvilket vil kunne give vertigo og faldtildens ipsilateralt.
2.3.7. Redegr kort for hvilke arterier der indgr i circulus arteriosus Willisii, hvilke hjernehinder
de ligger imellem og hvilke betydning circulus arteriosus har.
Circulus arteriosus Willisii bestr af en ring af anastomoser p hjernens basis. Ringen ligger i
subarachnoidalrummet, dvs. mellem pia mater og arachnoidea mater.
De to aa. cerebri posteriores (fra a. basilaris) fr via a. communicans posterior kontakt til a. cerebri
media.
De to aa. cerebrales anteriores (fra a. carotis interna) vil kontakte hinanden via a. communicans
anterior.

Alts indgr i cirklen:


A. cerebri anterior sin et dxt
A. communicans anterior
A. caroticus interna sin et dxt
Opgaver side 19

A. caroticus interna sin et dxt


A. cerebri posterior sin et dxt
A. communicans posterior
Circulus arteriosus medvirker til at danne kollateraler i den cerebrale cirkulation, sdan s at hvis n
del af cirklen blokeres eller indsnvres kan de andre arterier forsyne de omrder der er blokeret
kompensatorisk.
2.3.8. Angiv, hvorfra hypothalamus modtager sin arterieforsyning.
Hypothalamus fr blod fra a. hypofysialis, som kommer fra a. carotis interna. Der gr ogs direkte
grene fra a. carotis interna.
2.3.9. Hvad er de kortikale og subkortikale forsyningsomrder for a. cerebri posterior?
De to a. cerebri posterior dannes nr a. basilaris deler sig i den rostrale pons.
Kortikale forsyningsomrder:
A. cerebri posterior danner a. temporooccipitalis og a. occipitalis interna:
A. temporooccipitalis forsyner de posteriorer 2/3 af temporallappen inferiort og medialt.
A. occipitalis interna forsyner det meste af occipitallappen, samt en del af parietallappen.
Subkortikale forsyningsomrder:
A. cerebri posterior forsyner den posterolaterale del af mesencephalon.
2.3.10. Angiv den typiske placering af Brocas sprogomrde og omrdets arterieforsyning.

Brocas omrde findes i gyrus frontalis inferior (area 44).


Omrdet forsynes af a. cerebri media.
2.3.11. De cerebrale arterier (aa. cerebri) afgiver en rkke dybe, centrale grene hvilke strukturer
forsyner disse grene?
Grenene forsyner de dybe dele af cerebrum, f.eks. nucleus caudatus.
2.3.12. Angiv arterieforsyningen til flgende strukturer:
Primr auditiv cortex: a. cerebri media.
Primr synscortex: a. cerebri posterior med anastomoser.
Globus pallidus: a. cerebri media.
Primr somatosensorisk cortex: Primrt af a. cerebri media, men omrdet svarende til
benet forsynes af a. cerebri anterior.

Underekstremitetsomrdet sv.t. primr somatosensorisk cortex: a. cerebri anterior.


Supplementr motorisk cortex: a. cerebri anterior.
Smagsomrdet i cortex: a. cerebri media.
Hypothalamus: Vasa hypothalmica, som bestr af grene fra a. carotis interna.
Opgaver side 20

Hypothalamus: Vasa hypothalmica, som bestr af grene fra a. carotis interna.


Pyramis og lemniscus medialis: a. spinalis anterior.
2.3.13. Hvordan kommer veneblodet fra plexus choroideus i lateralventriklerne til v. jugularis
interna?
Veneblodet lber i v. cerebri magna til sinus rectus.
Fra sinus rectus lber blodet over i sinus transversus, som fortstter som sinus sigmoideum.
Sinus sigmoideum ender i v. jugularis interna.
2.3.14. Angiv forlbet af venst blod fra den dorsale thalamus og 3. ventrikels og
lateralventriklernes plexus choroideus til v. subclavia.
Nej.
2.3.15. Medulla oblongata kan med hensyn til arterieforsyningen, og som flge af vaskulrt
betingede kliniske symptomer, opdeles i en medial og en lateral del. Nvn for hver af disse to
typer lsioner og en nerve og to nervebaner, som typisk beskadiges.
Medialt medullrt syndrom:
Skyldes okklusion af grene fra a. spinalis anterior. En beskadigelse kan fre til beskadigelse af n.
hypoglossus, pyramidebanen og af lemniscus medialis.
Lateralt medullrt syndrom (Wallenbergs syndrom):
Skyldes okklusion af a. cerebellaris inferior posterior. En beskadigelse kan medfre den sensoriske
trigeminuskerne og tractus spinothalamicus lateralis.

Der vil derfor ses tab af temperatur- og smertesans kontralateralt p truncus og ipsilateralt i
ansigtet.
2.3.16. Wallenbergs syndrom, der skyldes unilateral okklusion af a. cerebellaris posterior inferior,
resulterer i et karakteristisk st af symptomer. Angiv tre karakteristiske symptomer og redegr
for det neuronale grundlag for hvert af disse symptomer.
Wallenbergs syndrom kaldes ogs for lateral medullrt syndrom.
Ofte ses sensoriske udfald p truncus og p ekstremiteterne p den modsatte side af infarktet. Der
ses desuden sensoriske udfald i ansigtet og udfald af kranienerverne i samme side som infarktet.
Specifikt er der tab af smertesans og temperatursans kontralateralt p kroppen og ipsilateralt i
ansigtet. Fundet er karakteristisk for sygdommen.
Andre kliniske symptomer er:
Synkebesvr
Slrret tale
Svimmelhed
Dette skyldes skade p nc. ambiguus, der er kerne for n. vagus og n. glossopharyngeus.
Svimmelheden skyldes udfald af nc. vestibulocochlearis.
2.3.17. Okklusion af a. basilaris giver anledning til "locked-in" syndrom. Redegr kort for de
symptomer, der ses ved "locked-in" syndrom.
Opgaver side 21

symptomer, der ses ved "locked-in" syndrom.


Paralyse af nsten alle skeletmuskler udover jenmusklerne.
2.3.18. Beskriv kort de symptomer, som en okklusion af a. cereballaris posterior inferior sinister vil
medfre. Benvn de involverede symptomer.
Okklusion af a. cereballaris posterior inferior vil give Wallenbergs syndrom (ogs kendt som lateralt
medullrt syndrom).
Der vil vre sensoriske udfald kontralateralt p kroppen og ipsilateralt i ansigtet. Patienten vil alts
ikke kunne fle smerte eller temperatur i venstre side af ansigtet og vil ikke kunne fle smerte eller
temperatur i hjre side af kroppen.
Desuden vil der vre parese af de venstre tungemuskler og det venstre stemmebnd pga. skader p
nc. ambiguus.
2.3.19. Angiv udbredelsen af en bldning fra circulus Willisii ift. hjernehinder og angiv diagnostiske
symptomer for en sdan bldning.
Circulus Willisii ligger imellem pia mater og arachnoidea mater - dvs. i det subarachnoidelle rum.
En subarachnoidel bldning vil frst give hovedpine, der starter meget voldsomt. Desuden vil der ses
opkastning, forvirring, bevidsthedspvirkning og nogle gange kramper. Symptomerne skyldes, at ICP
stiger og at hjernestammen komprimeres.

Der vil ogs kunne mles get blodtryk og pulsen vil falde.
2.3.20. Angiv type og lokalisation af den type kar, der typisk ved en lsion giver subdurale
bldninger.

Det subdurale rum er rummet mellem dura mater og arachnoidea mater.


Bldninger i rummet skyldes ofte lsion af brovenerne, der er vener, der drner ind i sinuserne fra
det subarachnoidelle rum.
2.3.21. Beskriv kort forsyningsomrdet og de symptomer, der typisk ses efter en nsidig okklusion
af a. cerebri posterior.
A. cerebri posterior forsyner den posteriore del af hjernen. Arterien forsyner store dele af
occipitallappen, bl.a. det viesuelle cortex.
Desuden forsyner arterien den inferiore del af temporallappen, bl.a. de omrder, der indeholder
centre for hjere visuel association.
Okklusion vil f.eks. give visuelle forstyrrelser - f.eks. prosopagnosia hvor pt. mister evnen til at
genkende ansigter.
2.3.22. En subdural bldning skyldes ofte overrivning af en bestemt gruppe kar. Benvn disse og
angiv deres lokalisation.
Brovenerne, der drner blod fra det subarachnoidelle rum til sinusserne - f.eks. til sinus sagittalis
superior.
2.3.23. Angiv det funktionelle udfald, ved bortfald af arterieforsyningen til omrdet langs fissura
calcarinas verste kant p venstre side. Hvilken hovedarterie forsyner det pgldende omrde?
Fissura calcarina findes ved den caudale ende af hjernens mediale overflade. Sulcus calcarinus er
Opgaver side 22

Fissura calcarina findes ved den caudale ende af hjernens mediale overflade. Sulcus calcarinus er
stedet, hvor den primre visuelle cortex er beliggende.
Resultatet af manglende blodforsyning til dette omrde er manglende syn. Der er manglende syn i
hjre synsfelt.
Omrdet forsynes af a. cerebri posterior, som anastomoserer med a. cerebri media.
2.3.24. Hjernestammens karforsyning (essay):
a. Der nskes en beskrivelse af hjernestammens karforsyning omfattende de arterielle
hovedstammer, deres grene og disse grenes regionale forsyningsomrder.
Hjernestammen bestr af medulla oblongata, pons og mesencephalon.
Medulla oblongata forsynes fra:
Fortil lateralt: a. vertebrales.
Fortil medialt og posteriort: a. spinalis anterior.
Posteriort og lateralt: a. cerebelli inferior posterior.
Pons forsynes fra:
Medial del: Paramediane grene fra a. basilaris.
Anterolateral del: Korte circumferente grene fra a. basilaris.
Posterolateral del: Lange circumferente grene fra a. basilaris.

Mesencephalon forsynes fra:


Medial del: Paramediane grene fra a. basilaris.
Anterolateral del: Korte cirkumferente grene fra a. basilaris.
Posterolateralt: Lange cirkumferente grene fra a. basilaris, a. cerebri posterior og
a. cerebelli superior.
b. Med brug af en tegning af et relevant tvrsnit af medulla oblongata nskes virkningen
af hhv. en medial og en lateral vaskulr betinget lsion beskrevet i form af korte, men
prcise angivelser af:
i. De involverede kar.
ii. De involverede neurale strukturer.
iii. De resulterende symptomer, sammenholdt med funktionen af de neurale
strukturer.
Lsionerne kan opdeles i mediale og laterale lsioner:
Medialt medullrt syndrom:
Skyldes okklusion af grene fra a. spinalis anterior. En beskadigelse kan fre til
beskadigelse af n. hypoglossus, pyramidebanen og af lemniscus medialis.
Lateralt medullrt syndrom:
Skyldes okklusion af a. cerebellaris inferior posterior. En beskadigelse kan medfre skade
Opgaver side 23

Skyldes okklusion af a. cerebellaris inferior posterior. En beskadigelse kan medfre skade


p den sensoriske trigeminuskerne og tractus spinothalamicus lateralis.
Frst betragtes en okklusion af a. spinalis anterior:
A. spinalis anterior forsyner den mediale, anteriore del af medulla spinalis og den
mediale del af medulla oblongata.
Beskadigelse af a. spinalis anterior vil give medialt medullrt syndrom. Dette kan
medfre f.eks. skader p n. hypoglossus, hvilket vil medfre ipsilateral parese af tungens
muskler.
Der vil kunne vre skader p pyramidebanen. Dette vil medfre kontralateral spastisk
lammelse af ekstremiteterne, hvis skaden er over krydsningen. Under krydsningen
lammelsen vre ipsilateral.
Der vil kunne vre skader p lemniscus medialis, hvilket vil kunne medfre
kontralateralt tab af diskriminationssans, flesans og vibrationssans.
Nu betragtes en okklusion af a. cerebellaris inferior posterior:
A. cerebelli inferior posterior forsyner den posterolaterale del af medulla oblongata og
de inferiore dele af cerebellums hemisfrer.
Beskadigelse af a. cerebelli inferior posterior vil give lateralt medullrt syndrom. Dette
kan give skader p f.eks. tractus spinothalamicus, hvilket vil medfrer kontralateral tab
af smerte og temperatursans p kroppen. Der vil kunne vre skader p nc. trigeminale
spinalis, hvilket vil unne medfre ipsilateral tab af smerte og temperatursans i ansigtet.
Denne krydsning af de sensoriske udfald er karakteristisk for sygdommen!
Der vil kunne vre skade p nc. ambiguus, der indeholder de efferente kerner for n.
vagus og n. glossopharyngeus. Dette vil kunne give dysphagi, hshed, nedsat
brkrefleks og lammelse af den blde gane.
Der vil kunne vre skade p nc. vestibularis, hvilket vil kunne give vertigo og faldtildens
ipsilateralt.

Opgaver side 24

Kapitel 3: Rygmarven, reflekser og baner


6. februar 2014
18:02

Opgaver side 25

3.1. Rygmarven:
6. februar 2014
18:03

3.1.1. Benvn de med A-H markerede strukturer p figur 3.1.


A)
B)
C)
D)
E)
F)
G)
H)

Tractus spinocerebellaris posterior


Cornu dorsalis
Fasiculus cuneatus
Fasiculus gracilis
Funiculus lateralis
Canalis centralis
Fissura mediana ventralis
Tractus vestibulospinalis

3.1.2. Skitsr et tvrsnit af medulla spinalis p midtthoracalt niveau og benvn de makroskopisk


synlige strukturer.
Vigtige strukturer:

Sulcus medianus
Sulcus intermedius dorsalis
Sulcus dorsolateralis
Fissura mediana ventralis
Sulcus ventrolateralis
Cornu dorsale
Cornu ventrale

3.1.3. Navngiv fem kerneomrder/laminae i medulla spinalis med angivelse af funktion.


Her er en oversigt:
Laminae: Zona terminalis:
1

Smerte- og temperaturmodulation

Smerte- og temperaturmodulation

Kutane impulser og modulation heraf

Opgaver side 26

Kutane impulser og modulation heraf

Proprioception og sansereception

Proprioception og sansereception

Proprioception, sansereception, Clarks kerne,


visceromotoriske forbindelser

Motoriske interneuroner

Somatomotoriske neuroner

10

Centrale gr substans

3.1.4. Angiv tre eksempler p inhibitoriske interneuroner i medulla spinalis.


Renshaw-celler, enkefalinerge interneuroner og GABAerge interneuroner virker alle inhibitorisk.
Renshaw-celler: Renshaw-celler er inhibitoriske interneuroner. Alfa-neuroner i forhornet
hmmer sig selv ved at sende kollateraler til de inhibitoriske Renshaw-celler. Dette er med til
at regulere alfa-neuroners excitabilitet og flgelig musklens kontraktionsgrad.
Enkefalinerge interneuroner: Enkefalinerge interneuroner (interneuroner, der bruger enkefalin
(et opioid-peptid, der minder om endorfin) som transmitterstoffer) kan hmme
spinothalamiske neuroner svarende til lamina to. Serotonerge eller noradrenerge . Dette kan
modulere signaler fra nociceptorer.
GABAerge interneuroner: GABAerge interneuroner virker inhibitoriske p Ia-fibre fra
muskelspoler.
3.1.5. Giv to eksempler p intern topografisk fordeling af nervefibre i nervebaner i medulla
spinalis.
I bagstrengen er fibrene fordelt i funiculus gracilis og funiculus cuneatus. Funiculus gracilis ligger
mest medialt og sender sensoriske fibre fra alt under T6 til cortex via thalamus. Funiculus cuneatus
ligger mere lateralt og sender sensoriske fibre fra alt over T6 til cortex via thalamus.
I tr. corticospinalis lateralis er fibrene fordelt sledes, at fibrene til cervikaldelen ligger mest medialt
og fibrene til sacraldelen ligger mest lateralt. Herimellem er fibre til de andre spinalsegmenter
somatotopisk orienteret.

3.1.6. Angiv p et tvrsnit af medulla spinalis cervicalis beliggenhed, navn og funktion for to
motoriske og to sensoriske baner, samt to sensoriske kerneomrder.

Opgaver side 27

Der lber mange baner her. Eksempelvis de motoriske:


Tr. cortispinalis (pyramidebanen): Lber i den laterale funiculi.
Tr. vestibulospinalis: Lber i ventrale funiculi.
Og de sensoriske:
Tr. spinothalamicus (det anterolaterale system): Lber i de anterolaterale funiculi.
Bagstrengene (bagstrengs-lemniscus medialis systemet): Lber i de dorsale funiculi.
Og kerner:
Nc. proprius.
Nc. phrenici.
3.1.7. Angiv tre markante forskelle p udseendet af tvrsnit af medulla spinalis cervicalis (C4) og
thoracalis (T8).
Der er blandt andet flgende forskelle:
Rygmarven er tykkere p C4-niveau end p T8-niveau.
Cornu dorsalis er meget tykkere p C4-niveau end p T8-niveau.
Cornu ventralis er meget tykkere p C4-niveau end p T8-niveau.
Der er mere gr substans p C4-niveau end p T8-niveau.
Hvor fanden er Lateralhorn?

3.1.8. Tegn et tvrsnit af medulla spinalis svarende til midt-thoracalt niveau og angiv
beliggenheden af:
1.
2.
3.
4.
5.
6.
7.
8.
9.
10.
11.

Gr og hvid substans
aa. spinales posteriores og anterior
Substantia gelatinosa
Tr. spinocerebellaris dorsalis
Nc. thoracicus dorsalis Clarki
Tr. vestibulospinalis lateralis
Tr. reticulospinalis anterior/medialis
Tr. rubrospinalis
Tr. corticospinalis lateralis
Tr. spinothalamicus lateralis
Fasciculus gracilis

Opgaver side 28

3.1.9. Angiv p tvrsnit af medulla spinalis thoracalis beliggenheden af de descenderende


motriske baner af supraspinal oprindelse.
Se ovenfor.
3.1.10. Tegn to vellignende tvrsnit af medulla spinalis p hhv. C4- og T10-niveau, og angiv
placeringen af tre kerneomrder eller nervebaner, der findes p begge niveauer, og to
kerneomrder i gr substans, som kun findes p det thoracale niveau.
P begge niveauer findes ALS, BLM og tr. corticospinali
P thoracalt niveau findes Clarks kerne, den findes ikke p cervikalt niveau. Desuden findes der en
nc. motoris lateralis.
3.1.11. Angiv kort forlb og funktion(er) af tre vsentlige nervebaner, der beskadiges ved en
unilateral, halvsidig lsion af medulla spinalis.
En unilateral, halvsidig lsion af medulla spinalis vil give Brown-Sequard-syndrom. Man vil f delagt
flgende:
Det anterolaterale system (tr. spinothalamicus): Lber fra rygmarv, hvor det krydser
segmentrt, og lber til VPL og videre til SI og SII. Sender signaler fra termoreceptorer og
nociceptorer.
Bagstrengs-lemniscus medialis systemet (fasiculus gracilis og cuneatus): Sanseneuroner lber
ind i rygmarven og ascenderer i fasiculus gracilis og fasiculus cuneatus. Danner synapser i nc.
gracilis og nc. cuneatus. Herfra lber lemniscus medialis mod VPL. Den krydser i nedre medulla
oblongata. Fra VPL lber fibrene mod SI og SII. Sender signaler fra mekanoreceptorer og
proprioceptorer.
Tr. corticospinalis (pyramidebanen): Lber fra de motoriske cortex (isr MI) og mod
rygmarven som tr. pyramidalis. Krydser over midtlinjen i caudal medulla oblongata (decussatio
pyramidalis) og bliver til tr. corticospinalis. Danner synapser med motorneuroner (eller
interneuroner) i rygmarven. Medierer signaler om prcis bevgelser.
Man vil alts f sensoriske udfald under lsionen, sledes at berring, vibration, proprioception ikke
kan opfattes. Denne skade vil vre ipsilateral.
Man vil f tab af smerte- og temperatursans under lsionen, denne skade vil kontralateral.
Man vil f tab af prcise bevgelser under lsionen. Denne skade ville formentligt vre ipsilateral,
Opgaver side 29

Man vil f tab af prcise bevgelser under lsionen. Denne skade ville formentligt vre ipsilateral,
men det afhnger af, hvor skaden sidder.
3.1.12. Angiv de sensoriske og motoriske og eventuelt vaskulre symptomer man kan forvente at
finde distalt for en halvsidig tvrsnitslsion (hemisektion) af medulla spinalis svt. T8.
Patienten vil f Brown-Sequard syndrom. Der vil ses tab af smerte og temperatursans p
kontralaterale side. Det skyldes, at tractus spinothalamicus er afbrudt.
Der vil ses spasticitet ipsilateralt i underbenet, pga. en MN-skade p tr. corticospinalis.
Der vil ses manglende evne til at opfange mekaniske stimuli fra ipsilaterale UE, pga. ldering af
funiculus gracilis.
3.1.13. Angiv de symptomer, der udlses f.eks. svarende til en underekstremitet ved lsion af
MN, NMN og spinalnervernes dorsale rdder:
a. vre motorneuroner (MN): Spastisk paralyse/parese, Babinskis ttegn, clonus, hyperrefleksi,
hypertonicitet. Lidt atrofi?
b. Nedre motorneuroner (NMN): Slap paralyse/lammelse, hyporefleksi, hypotonicitet, meget
udtalt atrofi.
c. Spinalnervernes dorsale rdder: Sensoriske udfald - der vil ikke kunne sendes sensoriske
signaler overhovedet.

3.1.14. Forklar, hvordan en rumopfyldende proces i cervikaldelen af canalis vertebralis kan give
sensoriske udfald p kontralaterale underekstremitet.
Sensorisk information ascenderer i canalis vertebralis overordnet i to systemer:

Tr. spinotalamicus: Frer signaler om smerte og temperatur. Tr. spinotalamicus krydser p


segmentr niveau (det tager ca. 1-2 segmenter at krydse over) og derfor vil der ske udfald p
den kontralaterale UE.
Lemniscus medialis (bagstrengskernerne): Frer sensorisk information om tryk og berring,
stilling og bevgelse og topunkts-diskrimination samt vibration.
Lemniscus medialis krydser frst i decussation leminisorum i medulla oblongata, s derfor vil
skader give udfald p samme side som skaden.

Opgaver side 30

3.2. Strkkerefleksen:
6. februar 2014
18:12

3.2.1. Beskriv, gerne med en tegning, den neuronale grundlag for strkrefleksen.
En refleks bestr af flgende dele:

Receptor
Afferent led
Reflekscenter
Efferent led
Effektor

Ofte bruges patella-refleksen som eksempel p strkrefleksen. For patella-refleksen glder


flgende:
Receptor: Muskeltenen i m. quadriceps femoris.
Afferent led: I,a-fiber (sensorisk fiber, pseudounipolre med cellelegeme i spinalganglier).
Reflekscenter: Rygmarven. Fiberen lber ind i det dorsale horn til synapse i det ventrale horn
(lamina 9).
Efferent led: Alfa-motorneuron i lamina 9.
Effektor: Ekstrafusale fibre i m. quadriceps femoris.
Den mono-synaptiske strkrefleks er den vigtigste!
3.2.2. Hvilken rolle spiller strkrefleksen ved normal muskelaktivitet, f.eks. ved kontraktion af en
muskel mod pludselig strre eller mindre modstand?
Strkrefleksen beskytter muskler og sener, mod overstrk. Refleksen srger for, at musklens
lngde hele tiden holdes konstant.
3.2.3. Redegr kort for gamma-efferenternes funktion ifm. normal muskelaktivitet.
Gamma-motorneuronerne kontrahere de intrafusale fibre i musklen, dvs. muskeltenen, og
modulerer alts tenens tension. Tension i muskeltenen gr den mere sensitiv overfor strk og
sledes er gamma-efferenterne alts med til at modulere strkrefleksen.
Gamma-efferenterne inhiberes af vre muskelneuroner. Dette modulerer tenens flsomhed for
registrering af musklens lngde og for ndring af lngden. Dette gres for, at muskeltenen kan
informere om lngden i hele musklens arbejdsomrde, selv nr den er strkt forkortet.
Muskelspolens flsomhed afhnger af gamma-motorneuroner. Ved spastisk lammelse (skade p
MN) inhiberes gamma-efferenterne ikke, og derved ses hyperrefleksi.
3.2.4. Redegr kort for, hvordan en aktivering af spinale gamma-motorneuroner til en given
muskel i princippet kan udlse en kontraktion af musklens regulre muskelfibre.
Aktivering af gamma-efferenterne vil ge sandsynligheden for depolarisering af de sensoriske
afferente led. Dette kan medfre hyperrefleksi, da en strkrefleks kan udlses.
Sdan problematikker ses bl.a. ved skader p MN.
3.2.5. Hvordan forklares strkrefleksens ndring ved skaldt spastisk lammelse?
Opgaver side 31

3.2.5. Hvordan forklares strkrefleksens ndring ved skaldt spastisk lammelse?


Spasticitet skyldes ofte skade p descenderende motorpathways fra cortex cerebri til
motorneuroner. En sdan skade pvirker muligvis de spinale interneuroners excitabilitet.
Den gede modstand mod hurtige bevgelser skyldes formentligt en abnormt hurtig monosynaptisk
strkrefleks.
3.2.6. Redegr for begrebet spasticitet og den almindelige forklaring p dette neurologisk
symptom.
Spasticitet er patologisk get muskeltonus i skeletmuskulatur, som pvirker forskellige
muskelgrupper i forskellig grad. Dvs. en tilstand, hvor musklerne udviser get modstand mod strk.
Typisk ses spasticitet som get modstand mod den hurtige, passive bevgelse pga. hyperrefleksi.
Spasticitet skyldes ofte skade p descenderende motoriske pathways fra cortex cerebri til
motorneuroner. Den gede modstand mod hurtige bevgelser skyldes formentligt en abnormt
hurtig monosynaptisk strkrefleks.

Opgaver side 32

Kapitel 4: Det somatosensoriske system


6. februar 2014
18:15

Opgaver side 33

4.1. Somatosensoriske baner:


6. februar 2014
18:15

4.1.1. Hudens flesans kan opdeles i en rkke grundlggende sansekvaliteter. Nvn disse.
Sanseindtryk opdeles i modaliteter, som kan beskrives ud fra kvaliteter. Modaliteter kunne vre:

Berring
Varme
Tryk
Kulde
Smerte

Modaliteterne kan beskrives med kvaliteter, f.eks. kan en smerte vre brndende og tryk kan vre
mere eller mindre diffust osv.
4.1.2. Angiv den funktionelle inddeling af somatosensoriske receptorer i huden og deres adkvate
stimuli.
De overordnede receptortyper er:
Mechanoreceptorer: Reagerer p mekaniske stimuli af vv. Findes f.eks. i huden (som
Meissner-legemer, Merkelskiver osv.)
Chemoreceptorer: Reagerer p kemiske stimuli, findes f.eks. i karrene, hvor de sender signal
om blodets kemiske sammenstning.
Nocireceptorer: En slags high-threshold mechanoreceptorer, der sender signal ved
deformation af vv. Sender signal om smerte.
Termoreceptorer: Reagerer p temperaturmssige stimuli af huden. Sender signal om
temperaturndringer.
4.1.3. Regulering af perifert beliggende sensoriske receptorer gennem efferente nerveforbindelser
fra CNS forekommer relativt sjldent. Angiv to eksempler herp, og for hver af disse den
funktionelle effekt af den centrale regulering.
4.1.4. Benvn de seks endeomrder (i form af laminae, kerneomrder eller evt. neurontype) for
spinalnervernes primre afferente nervetrde og knyt en funktion til tre ad disse forbindelser.
Rexeds lamina er opdelt sledes:
Laminae: Zona terminalis:
1

Smerte- og temperaturmodulation

Smerte- og temperaturmodulation

Kutane impulser og modulation heraf

Kutane impulser og modulation heraf

Proprioception og sansereception

Proprioception og sansereception

Proprioception, sansereception, Clarks kerne,


visceromotoriske forbindelser

Motoriske interneuroner

Somatomotoriske neuroner
Opgaver side 34

Somatomotoriske neuroner

10

Centrale gr substans

Lamina 1-7 er mere eller mindre sensoriske.


4.1.5. De primre afferente nervetrde til rygmarvens baghorn udviser en rkke forskelle alt
efter, om de leder f.eks. smerteimpulser eller impulser vedr. 2-punkts-diskrimination . Angiv tre
forskelle mellem to nvnte fibertyper.

Der er forskel p fibertyperne.


Smerteimpulser ledes igennem tynde myelinerede A-delta fibre og igennem umyelinerede C-fibre.
4.1.6. Redegr kort for de somatosensoriske sansekvaliteter i bagstrengsbanerne og de perifere
receptorer knyttet hertil.
De sansekvaliteter, der ledes i bagstrengsbanerne, er:

Tryk og berring
Stillings- og bevgesans (i.e. proprioception)
Topunktsdiskrimination
Vibrationssans.

Signalerne ledes igennem lavtrskel mekanoreceptorer.


4.1.7. Redegr kort for, gerne hjulpet af en tegning, for den topografiske cortikale reprsentation
af bagstrengsbanernes sansekvaliteter, herunder for rsagen til den forskellige arealmssige
strrelse af de forskellige krops og legemesdeles reprsentation.
Bagstrengsbanen er somatotopisk organiseret - dvs. at fibre fra de samme steder lber sammen.

De kortikale omrder, der modtager information fra f.eks. hnder og fdder, er strre end andre
omrder.
4.1.8. Der nskes en redegrelse for bagstrengsbanen (lemniscus medialis systemet) efter
flgende disposition:
a. Funktion: Lemniscus medialis-systemet leder information om tryk, berring og ledsans til
cortex i tykke sensoriske fibre. Lemniscus medialis-systemet er vigtig for den diskriminerende
sensibilitet og for prcise bevgelser, der krver hurtig justering af muskelkraft p baggrund
af sensoriske inpu.
Bagstrengene, hhv. fasciculus gracilis et cuneatus, ender i hhv. nucleus gracilis et cuneatus i
medulla oblongata, hvor de har synapser. Impulser sendes videre til thalamus via lemniscen og
danner her synapser inden de nr cortex. Fibrene er ordnet somatotopisk, sledes at kaudale
fibre ligger medialt og kranielle fibre ligger mere lateralt.
b. Receptortype og beliggenhed: Igennem BLM formidles information om berring, tryk,
diskrimination, vibration og ledsans. Flgende receptorer indgr:
Lavtrskel mekanoreceptorer
Proprioceptorer.
c. Forbindelsen fra receptorer til cortex cerebri: Sensorisk afferent information ledes i nervefibre
(med trofisk centrum i spinalgangliet) og ender i dorsalhornet i medulla. Herfra lber fibrene
ud i bagstrengen og ascenderer ipsilateralt. Fibrene danner synapser i nc. cuneatus et gracilis
og krydser over til kontralateral side og bliver her til lemniscus medialis. De ascenderer videre
Opgaver side 35

og krydser over til kontralateral side og bliver her til lemniscus medialis. De ascenderer videre
og fr tilskud fra nc. trigeminus. Fibrene forstter til thalamus og danner synapser. Fibre fra
trigeminuskernen danner synapser i VPM, mens fibre fra selve bagstrengen danner syanpser
mere lateralt i VPL. Fra VPM og VPL vil tertire fibre forstte igennem capsula interna til
primrt somatosensorisk cortex i gyrus postcentralis.
I cortex vil der vre en somatotopisk opdeling, hvor fibre fra benet ligger lngst medialt og
fibre fra mere rostrale omrder ligger mere lateralt.
d. Den kortikale bearbejdning af den primre sensoriske information, herunder begrebet
stereognosi (evnen til somatosensorisk at genkende/bestemme berrte genstanses form,
strrelse og konsistens): Sterognosi er evnen til at genkende en genstand udelukkende baseret
p berring. Denne evne menes at vre lokaliseret til cortex cerebri.
BLM-systemet er hurtigt adapterende og er god til at formidle prcis information om et lille
receptorisk felt og derfor er det god til at formidle information om berring af forskellig art. I
fingrene er de receptoriske felter mindre, og vi fr mere prcis information, hvis vi bruger
dem.
4.1.9: Angiv forlbet af de smertefrerende nervebaner fra kroppen til cortex cerebri.
Smertefibre og signaler om temperatur fres via det anterolaterale system, hvor isr tr.
spinothalamicus er vigtig.

Der er tale om frie nerveender (nociceptorer og termoceptorer) fra hud, muskler og led.
Adelta og C-fiber frer smerte og temperatur, de har trofisk centrum i ganglion spinale. Fr
de fres ind i dorsalhornet deler de sig i en kort opstigende og en kortnedstigende gren som
gr en/to sekmenter, dorsalrdsfiberne danner et lille bundt lige forsalt for dorsalthornet =
lissauers bundt eller zona terminalis.
Grenene fra disse fiber gr ind gennem dorsalhornet og danner synapse med neuroner i
lamina I, II (C-fiberene) og V. Der dannes synapse med de spinothalamiske fibre. Dog ikke i
Lamina II, her dannes der synapse med internuroner som derefter danner synapse med de
spinothalamiske fiber i I, V, VII, VIII.
Strstedelen af 2. ordensnuronet krydser over i den ventrale hvide kommisur (det tager 1-2
segmenter at krydse). Neuronet lber op igennem det anteolaterale system i funiculus
lateralis. Ved hver niveau i medulla spinalis tilfres spinothalamiske fiber medialt,
efterhnden som banen ascendere. P vejen til thalamus afgive mange spinothalamiske
fiber kollateraler til kerner i hjernestammen som deltager i autonom kontrol. I
hjernestammen ligger banen lateralt og forholdsvis overfladisk.
I thalamus dannes der synapse i VPL og fra den spinale trigeminuskerne i VPM. Aktiveringen
af VPL krver mere summation end ved signal fra lavtrskelsreceptorene. Mange af fiberne
fra lamina I ender i VM. (Her udover ender enkelte spinothalamiske fiber i intralaminre
thalamuskerner som CL og MD).

Opgaver side 36

4.2. Smerte:
6. februar 2014
18:22

4.2.1. Forklar kort det neuronale grundlag for den centrale kontrol af smerteimpulser fra de
primre afferente smertetrde.
???
4.2.2. Hvilket neuropeptid spiller en rolle ved nociceptiv transmission i rygmarvens baghorn, og
hvordan udnyttes dette klinisk i behandling af smerte?

Endorphin eller enkefalin, muligvis? Morfin binder til de samme receptorer som endorphin og
modulerer smerterespons, ligesom endorphin gr det.
4.2.3. Forklar kort, gerne med en tegning, hvordan smerteimpulsernes videre transmission kan
modificeres allerede ved de primre afferente fibres frste synapser i rygmarven.
Det er mulig at hmme de afferente-somatosensoriske neuroner, fibre fra nociceptorer, som trder
ind i medulla spinalis baghorn og ender i lamina II. Dette sker ved at enkefalinerge (et opioid-peptid,
der minder om endorfin)interneuroner hmmer spinothalamiske neuroner svarende til lamina II.
Serotonerge eller noradrenerge neuroner menes at stimulere interneuronet til at frigive enkefalin
som igen hmmer spinothalamiske baner.
4.2.4. Redegr kort for fnomenet "gating" ifm. smerteopfattelsen.
Iflge gating-teorien leder bde tynde fibre (der leder smerte) og tykkere fibre (der leder tryk,
vibration osv.) information fra det sted, hvor smerten opfattes, og til to destinationer i rygmarven. Jo
mere de tykke fibre fyrer, jo mere vil de tynde fibre inhiberes.

Opgaver side 37

Kapitel 5: Det motoriske system


11. februar 2014
11:00

5.0.5. Definr begrebet vre og nedre motorneuron:


Motorneuroner inddeles i vre og nedre:
vre motorneuron (MN): Neuroner, der er lokaliseret i motoriske cortex eller i motoriske
centre i hjernen. MN bliver altid i CNS.
MN bruger glutamat som deres transmitterstof.
Nedre motorneuron (NMN): Neuroner, der danner synapse med muskler. Reprsenterer "final
common path" for signalerne. NMN gr altid ud af CNS.
NMN bruger acetylcholin som transmitterstof ift. aktivering af muskelceller.
NMN omfatter bde alfa- og gamma-motorneuroner!
5.0.6. Angiv og forklar symptomerne ved nedre og vre motorneuron-lsioner:
For lsioner i MN:
Spasticitet (get tonus, hyperaktive dybe reflekser, clonus): Skyldes manglende inhibition af
reflekser. get modstand mod hurtige, passive bevgelser. Dette skyldes formentligt en
overaktiv strkrefleks.
Babinskis tegn: Pyramidebanen skal normalt inhibere den refleksive kontraktion af extensormuskler i hallux nr plantar-refleksen stimuleres. Skade p pyramidebanen eller andre MN
giver Babinskis tegn, da extensormusklerne ikke inhiberes.
Tab af fine, voluntre bevgelser: Pyramidebanen medierer fine, voluntre bevgelser, der
krver koordination. Lsion af pyramidebanen eller relaterede MN vil give udfald her.
Clasp knife: Patienten udviser pludselig nedsat modstand mod passiv fleksion af led.
For lsioner i NMN:

Kraftnedsttelse/paralyse: Svag/ingen aktivering af motoriske enheder. Hvis n spinalrod


delgges vil musklen vre svkket, men have lidt funktion tilbage. delggelse af flere
segmenter vil medfre paralyse. Der er tale om slap parese/paralyse.
Nedsatte superficielle reflekser: Svag/ingen aktivering af motoriske enheder i den efferente del
af refleksbuen.
Nedsatte dybe reflekser: Svag/ingen aktivering af motoriske enheder i den efferente del af
refleksbuen, derfor svag aktivering af effektororgan (i dette tilflde muskel).
Nedsat muskeltonus: Svag/ingen aktivering af motoriske enheder, heller ikke nr musklen er i
hvile.
Fasciculationer: Spontan og lille depolarisering af nedre motorneuron. Skyldes spontan
frigivelse af Ach fra axonterminal, hvilket stimulerer til kontraktion.
Muskelatrofi: Svag/ingen aktivering af motoriske enheder medfrer henfald af muskelvv og
dermed atrofi.

Opgaver side 38

5.0.7. Redegr for hvilke motoriske symptomerne eller dysfunktioner, der kan forventes ved
bortfald af de tyktmyeliniserede axoner i spinalnervernes dorsale rod.
Nedsat signalering fra senetenen og muskeltenen, da I,a-fibre og I,b-fibre er degenerede. Desuden
nedsat signalering fra receptorer i led.
Derfor problemer med at kontrollere f.eks. hastighed af bevgelser. Manglende refleksiv ndring af
ledstilling og problemer med at holde balance, da proprioception (kroppens jne er vk).
5.0.8. Der nskes en beskrivelse af tr. corticospinalis, omfattende:
a. Oprindelse, forlb og termination:
Fibrene til pyramidebanen kommer fra primr motorisk cortex (MI), sekundr motorisk
cortex (MII), primrt og sekundrt sensorisk cortex og somatosensorisk associationscortex.
Fibrene fra cortex lber igennem crus posterior af capsula interna og kaldes herefter for tr.
corticospinalis (pyramidebanen).
Lber efter capsula interna i pendunculus cerebralis og herefter i pontine fibre. I medulla
oblongata lber trakten i pyramiderne og i den caudale del af medulla omblongata krydser
90% af fibrene midtlinjen via decussatio pyramidalis og lber i funiculus lateralis og kaldes nu
for tractus corticospinalis lateralis. Fibre der ikke krydser lber i tr. corticospinalis ventralis.
Disse innerverer rygmuskulatur og abdominalmuskulatur.

Tractus corticospinalis lateralis terminerer enten i interneuroner (Rexed lamina 8) eller i alfamotorneuroner (Rexed lamina 9)

Alts:
1)
2)
3)
4)
5)
6)
7)

MI, MII, SI, SII


Crus posterior af capsula interna
crus cerebri af mesencephalon (midterste 2/3)
pontine fibre
pyramiderne i medulla oblongata
overkrydsning ved decussatio pyramidalis
funiculus lateralis (og er nu tractus corticospinalis lateralis).

a. Funktion, herunder symptomer ved beskadigelser:


Pyramidebanen medierer prcise, voluntre bevgelser - isr finmotorik. Ved lsion ved
ses skader, som svarer til andre skader p MN - dvs. spasticitet, Babinskis tegn. Tab af fine
voluntre bevgelser.
b. Navn, oprindelse og placering i rygmarven af de vrige descenderende motoriske baner:
Det ekstrapyramidale system bestr af flere forskellige trakter:
Tr. rubrospinalis: Nc. ruber modtager fibre fra cortex. Fra Nc. ruber lber tr.
rubrospinalis mod medulla spinalis efter at vre krydset over midtlinjen i decussatio
segmentalis post. Tr. rubrospinalis sttter tr. corticospinalis og kontrollerer grovere
motorik. Nc. Ruber kommunikerer ogs med cerebellum.
Tr. reticulospinalis: Opretholder oprejst stilling, orienterer kroppen mod omgivelser,
kontrollerer autonom aktivitet (kredslb, GI-system, respiration, urinvejssystemet).
Lber fra cortex til formatio reticularis i pons.
Opgaver side 39

Lber fra cortex til formatio reticularis i pons.


Tr. vestibulospinalis: Integreret med det vestibulre system.
Tr. tectospinalis: Grundig integreret med synscortex. Giver mulighed for at fokusere p
noget visuelt i rummet. Sammenholder sensoriske indtryk med motorikken.
c. Hvorfor symptomerne ved en ren pyramidebanelsion og en lsion af f.eks. capsula interna er
forskellige.
I capsula interna lber der mange fibre - pyramidebanefibrene udgr kun en lille del af dem.
Der vil derfor ogs vre f.eks. sensoriske udfald. Ved lsion af pyramidebanen vil der kun ses
motoriske udfald.
5.0.9. Nvn de descenderende centrale motoriske baner til medulla spinalis og angiv for hver af
dem oprindelse, kendte afferente forbindelser, forlb og kendt eller sandsynlig funktion.
Tr. rubrospinalis: Nc. ruber modtager fibre fra cortex. Fra Nc. ruber lber tr. rubrospinalis mod
medulla spinalis efter at vre krydset over midtlinjen i decussatio segmentalis post. Tr.
rubrospinalis sttter tr. corticospinalis og kontrollerer grovere motorik. Nc. Ruber
kommunikerer ogs med cerebellum.
Tr. reticulospinalis: Opretholder oprejst stilling, orienterer kroppen mod omgivelser,
kontrollerer autonom aktivitet (kredslb, GI-system, respiration, urinvejssystemet). Lber fra
cortex til formatio reticularis i pons.
Tr. vestibulospinalis (medialis og lateralis): Opretholder balancen i oprejst stilling. Pvirker
strkreflekser og ekstensionsbevgelser. Refleksbevgelser af hovedet. Lber fra
vestibulum-cerebellum til rygmarven (primrt hoved og hals).
Tr. tectospinalis: Grundig integreret med synscortex. Giver mulighed for at fokusere p noget
visuelt i rummet. Sammenholder sensoriske indtryk med motorikken.
5.0.10. Angiv nc. rubers vigtigste afferente og efferente forbindelse, samt kerneomrdernes
funktion, baseret p disse forbindelser.
Nucleus ruber modtager afferente fibre fra MI og PMA (corticorubrale fibre). Nc. ruber
kommunikerer ogs med cerebellum.
Nc. ruber sender efferente forbindelser til rygmarven via tr. rubraspinalis.
Funktionen af tr. rubrospinalis er at sttte tr. corticospinalis.

Opgaver side 40

Kapitel 6: Hjernestammen og kranienerver


24. februar 2014
22:40

Opgaver side 41

6.1. Hjernestammen:
10. marts 2014
20:26

6.1.1. Forklar kort forskellen p effekten af en afbrydelse af de ascenderende forbindelser fra


formatio reticularis og afbrydelse af lemniscus medialis og tr. spinothalamicus.
Afbrydelse af forbindelser fra formatio reticularis: Koma? Bevidsthedstab?
Afbrydelse af lemniscus medialis: Tab af vibrationssans, berringssans, trykflsomhed og
proprioception kontralateralt eller ipsilateralt, afhngigt af, hvor lsionen sidder. Hvis
lsionen sidder over decussatio lemniscus medialis vil symptomerne vre kontralateralt og
hvis lsionen sidder under krydsningen vil symptomerne vre ipsilaterale.
Afbrydelse af tr. spinothalamicus: Vil medfre tab af smerte- og temperatursans kontralateralt
p kroppen.
6.1.2. Redegr kort for hvilke af hjernestammens tre dele, der udsttes for tryk ved forekomst af
rumopfyldende processer i kraniekavitetens supra- og infratentorielle rum.
I det supratentorielle rum:
Mesencephalon pvirkes af uncus fra temporallappen.
I det infratentorielle:
Medulla oblongata pvirkes af de cerebellare tonsiller, der presses ned gennem forarmen
magnum.
6.1.3. Redegr kort for de symptomer der optrder svarende til hovedet ved en unilateral lsion
af truncus sympaticus p halsen.
Der vil ses Horners syndrom, hvor der flgende symptomer:
Ptosis (jenlget falder ned, fordi m. tarsalis superior lammes) i den ramte side.
Miosis (fordi m. dillator pupillae lammes og parasympatikus-tonus til m. sphincter pupilla
stimuleres) i den ramte side.
Nedsat svedtendens i ansigtet i den ramte side
6.1.4. Angiv navn, beliggenhed og transmittertype for tre monoaminerge cellegrupper i
hjernestammen.
VTA og substantia nigra i mesencephalon: Benytter dopamin som transmitter.
Raphekernerne medialt i hjernestammen: Benytter serotonin som transmitter.
Locus caeruleus i pons: Benytter noradrenalin som transmitter.

6.1.5. I hjernestammen findes tre parvise st af pigmenterede nervecellekerner. Hvad hedder


kernerne og hvad er deres funktion?
De tre kerner er:
Substantia nigra i mesencephalon: Pars compacta sender dopaminerge fibre til basalganglier.
Nc. Ruber i mesencephalon: Sender fibre til medulla spinalis som tr. rubrospinalis.
Locus caeruleus i pons: Udgr en del af det cortikale aktiveringssystem, der kan skabe
opmrksomhedsreaktioner.
Opgaver side 42

6.1.11. Angiv hvilke overfladestrukturer man finder p mesencephalon.


Der er tre hjernenervekerner:
Den visceral-efferente oculomotoriuskerne (Erdinger-Westfall-kernen).
Den somatisk-efferente oculomotoriuskerne.
Den somatisk-efferente trochleariskerne.
6.1.12. Hvad er den funktionelle rolle af ponskernerne i pars basilaris af pons?
Ponskernerne modtager information fra cortex og sender det videre til cerebellums hemisfrer via
pedunculus cerebellum medius. I ponskernerne er der alts synapse mellem de corticopontine fibre
og de pontocerebellare fibre.
6.1.13. Angiv navn, beliggenhed og tilhrsforhold til kranienerver for de centrale,
sekretomotoriske kerneomrder for spytsekretion.
Kernerne for spytsekretion er:
Nc. salivatorius inferior: Giver visceral-efferente trde til n. glossopharyngeus, der innerverer
gl. parotidea. Nc. salivatorius inferior ligger caudalt og medialt i pons eller (meget) rostralt og
medialt i medulla oblongata.

Nc. salivatorius superior: Giver visceral-efferente trde til n. facialis, der innerverer
mundhulens spytkirtler. Nc. salivatorius superior ligger caudalt og medialt i pons, lige over nc.
salivatorius inferior.
6.1.15. Gr kort rede for den arterielle karforsyning af medulla oblongata:
Medulla oblongata forsynes fra tre arterier:
Anteriort: A. spinalis anterior.
Lateralt: A. vertebralis.
Posteriort: A. cerebelli inferior posterior
6.1.16. Angiv de symptomer, som en okklusion af a. cerebelli post. Inf. Dextra ville medfre, og
redegr for hvilke strukturer (nerver, nervebaner, kerneomrder), der er knyttet til de forskellige
symptomer:
Der er tale om Wallenbergs syndrom (lateralt medullrt syndrom). Der vil vre udfald her:
Nc. vestibulum: Giver faldtendens til den syge side og svimmelhed.
Nc. spinalis n. trigemini: Giver tab af smerte- og temperatursans ipsilateralt i ansigtet.*
Nc. ambiguus: Giver parese/paralyse af muskulatur i pharynx, larynx og gane. Der vil vre
hshed og dysphagi.
Tr. spinothalamicus: Giver tab af smerte- og temperatursans KONTRALATERALT p kroppen og
i ekstremiteterne.
Pedunculus cerebellaris inferior: Giver ataxi og dysdiadokinesi.
Descenderende sympatiske fibre: Giver ipsilateralt Horners syndrom.
6.1.17. Med brug af en tegning af et tvrsnit af vre, bne del af medulla oblongata nskes
virkningen af hhv. medial og lateral vaskulr betinget lsion beskrevet i prcise angivelser af:
a. De involverede kar:
For medialt medullrt syndrom: Skyldes okklusion a. spinalis anterior.
For lateralt medullrt syndrom (Wallenbergs syndrom): Skyldes okklusion. cerebelli
Opgaver side 43

For lateralt medullrt syndrom (Wallenbergs syndrom): Skyldes okklusion. cerebelli


inferior posterior (PICA).
b. De involverede neurale strukturer:
For medialt medullrt syndrom: Pyramidebanen, lemniscus medialis og fibre fra n.
hypoglossus.
For lateralt medullrt syndrom: Nc. ambiguus, nc. vestibularis, tr. spinothalamicus, nc.
trigeminus spinalis, sympatiske fibre og pedunculus cerebellaris inferior.
c. De resulterende symptomer sammenholdt med funktionen af de i pkt. b nvne strukturer.
For medialt medullrt syndrom: Spastisk lammelse kontralateralt for okklusionen, tab af
diskriminativ sans, proprioception og vibration kontralateralt for okklusionen og
deviation af tungen til den syge side.
For lateralt medullrt syndrom: Dysphagi og hshed, faldtendens og svimmelhed til syg
side, tab af smerte og temperatursans kontralateralt p kroppen og ipsilateralt i
ansigtet, Horners syndrom, ataxi og dysdiadokinesi.
6.1.18. Angiv hvilke kranienerver, der indeholder smagstrde og disse trdes perifere
innervationsomrder.
Tre kranienerver frer smagstrde til hjernen:
N. facialis: Frer smagstrde fra de forreste 2/3 af tungen (papilla fungiformis).
N. Glossopharyngeus: Frer smagstrde fra de bagerste 1/3 af tungen (papilla valleta og
papilla folliatae).
N. Vagus: Frer smagstrde fra et lille omrde omkring epiglottis (valleculae epiglottis).
6.1.19. Angiv den primre, sensoriske smagskerne.
Den primre smagskerne er nc. tractus solitarii i medulla oblongata. Den modtager fibre fra n.
facialis, n. glossopharyngeus og n. vagus.
6.1.20. Angiv denne kernes lokale og mere centralt kranielt rettede nerveforbindelser.
NTS bestr af en gruppe af kerner. Kernerne er ordnet sdan, at de mest rostrale kerner har
forbindelse til smag og de mere kaudale kerner har forbindelse til cardiorespiration.
Kernen har forbindelse til n. facialis, n. glossopharyngeus og n. vagus, som den modtager smagsfibre
fra.
Nc. tractus solitarii (NTS) sender fibre mange steder hen:
Fibre til nc. salivatorius superior et inferior
Fibre til nc. dorsalis vagii (som styrer maveskkens sekretion)
Fibre til hypothalamus (som styrer frigivelse af insulin og glucose)
Fibre til det primre smagscortex via VPM i thalamus
Fibre til amygdala
6.1.21. Hvilke funktioner har nc. tractus solitarius?
Modtager smagssignaler fra n. glossopharyngeus, n. vagus og n. facialis. Sender disse signaler videre
til f.eks. primrt smagscortex og nc. dorsalis vagii.
Modtager signaler fra chemo- og baroreceptorer via n. vagus og via n. glossopharyngeus.

NTS medierer opkastrefleksen, baroreceptorrefleksen, hosterefleksen, forskellige GI-reflekser og GImotilitet og GI-sekretion.

Opgaver side 44

6.1.22. Angiv lokale og ascenderende efferente forbindelser fra smagsdelen af nc. tractus
solitarius.
Der sendes fibre til:
Nc. salivatorius inferior og posterior
Nc. dorsalis vagii
Smagscortex via VPM i thalamus
Hypothalamus
Amygdala

Opgaver side 45

6.2. jensymptomer:
10. marts 2014
20:47

6.2.1. Redegr kort for, hvordan og hvorfor symptomerne ved en lsion af abducenskerneomrdet i pons adskiller sig fra en lsion af selve n. abducens.
I pons slynger n. fascialis sig rundt om abducenskernen.
En lsion i kerneomrdet for n. abducens vil alts formentligt ogs give perifer n. fascialis-parese
(parese af mimisk muskulatur i den ene side), hvilket vil give yderligere symptomer end bare parese
af m. rectus lateralis.
6.2.2. Benvn den anden hjernenerve, der hyppigt lderes samtidigt med n. facialis.
N. vestibulocochlearis og n. fascialis. N. facialis og n. vestibulocochlearis lber sammen igennem
meatus acusticus internus og kan pvirkes af f.eks. acusticus-neurinom.
6.2.3. Beskriv kort to reflekser, som n. facialis er involveret i.
To reflekser kunne vre:
Stapedius-refleksen: Den efferente del af refleksbuen er medieret af n. stapedius fra n. fascialis
Refleksbuen er:
Stimuli: Hj lydintensitet.
Receptor: Indre hrceller i det cortiske organ.
Afferent led: N. vestibulocochlearis.
Refleks-centrum: Retikulrsubstansen.
Efferent led: N. fascialis -> n. stapedius.
Effektor: M. stapedius.
Respons: M. stapedius trkker stapes fodplade vk fra det ovale vindue.
Cornea refleksen: Den efferente del af refleksbuen er medieret af n. fascialis.
Stimuli: Berring af cornea.
Receptor: Mekanoreceptorer i cornea.
Afferent led: N. opthalmicus fra n. trigeminus (n. V).
Refleks-centrum: Pons.
Efferent led: N. fascialis (n. VII).
Effektor: M. orbicularis oculi pars palpebralis.
Respons: jenlget trkkes ned og beskytter cornea.
6.2.4. Hvordan kan du ud fra kliniske symptomer placere en lsion af n. facialis langs forlbet af
nerven fra dens apparente udspring og til forarmen stylomastoideum.

N. fascialis afgiver n. intermedius og de skilles inden n. fascialis forlader kraniekaviteten.


N. intermedius forsyner trekirtler og de to spytkirtler i mundhulen med parasympatiske fibre. Tager
ogs smagsfibrene p de forreste 2/3 af tungen.
Hvis der er tab af tresekretion, smag og spytsekretion er n. fascialis formentigt lderet INDEN
afgangen ud igennem foramen stylomastoideum.
6.2.6. Angiv beliggenhed og navn af jenmuskelkernerne, nervernes apparente udsrping samt de
innerverede muskler og deres virkning p jenblet.
Tredje, fjerde og sjette hjernenerve innerverer jets muskler.
Opgaver side 46

Tredje, fjerde og sjette hjernenerve innerverer jets muskler.


N. oculomotorus (n. III): N. oculomotorius har to kerner. Der er en somatisk-efferent kerne (nc.
oculomotorius) og en visceral-efferent kerne (Edinger-Westphal-kernen). Begge kerner ligge
p mesencephalon-niveau tt op ad hinanden. Det apperante udspring for kernen er p
forsiden af mesencephalon.
De somatisk-efferente fibre innerverer:
M. rectus superior: Trkker jet opad og medialt og roterer det.
M. rectus inferior: Trkker jet nedad og medialt og roterer det.
M. rectus medialis: Trkker jet medialt.
M. obliquus inferior: Trkker jet opad og lateralt og roterer det (lidt).
M. levator palpebrae superioris: Lfter jenlget.
De visceral-efferente fibre innerver:
M. sphincter pupillae: Trkker pupillen sammen.
M. cillaris: Krummer linsen (gr den mere konveks) og medvirker til akkomodation.
N. trochlearis (n. IV): Trochleariskernen (nc. n. trochlearis) ligger foran akvdukten i
mesencephalon. Den eneste af hjernenerverne, der lber ud p bagsiden af hjernestammen.
De somatisk-efferente fibre innerverer:
M. obliquus superior: Trkket jet nedad og lateralt og roterer det lidt.
N. abducens (n. VI): Nc. abducens er den motoriske kerne. Den ligger p pons-niveau. Nerven
trder ud langs den nedre kant af pons.
De somatisk-efferente fibre innerverer:
M. rectus lateralis: Trkker jet lateralt.
6.2.7. Beskriv det forbindelsesmssige grundlag for ligevgtsorganets styring af jenbevgelser.
Ligevgtsorganet styrer den vestibulo-oculre refleks og den octo-kinetisk refleks.
Den vestibulo-occulre refleks: Primre afferenter fra buegangene lber til vestibulariskernen.
Neuronerne i vestibulariskernen sender neuroner til fasciculus longitudinalis medialis og de gr til
jenmuskelkernerne. Motorneuronerne i disse kerner sender axoner til jenmusklerne.
Refleksen bevirker, at blikket kan holdes fast p retina, selvom hovedet bevger sig.
6.2.8. Beskriv det forbindelsesmssige grundlag for den kortikale styring af jenbevgelserne.
Nedstigende forbindelser fra cortex til hjernestammen via capsula interna pvirker prmotoriske
neuroner.
De fleste omrder af cortex har relation til bde voluntre sakkadebevgelser og voluntre
flgebevgelser.
6.2.9. En person med fuldstdig lsion af n. oculomotorius sin. i fossa interpenduncularis bliver
bedt om at se til hjre. Redegr kort for de to jnes stilling og andre synlige symptomer.
Lsion af n. oculomotorius sin. vil lammelse af:
M. rectus medialis
M. rectus superior
Opgaver side 47

M. rectus superior
M. rectus inferior
M. obliquus inferior
M. cillare
M. sphincter pupillae
jet vil derfor, nr man kigger til hjre, vende lateralt og nedad. Der vil desuden vre mydriasis
(pga. lammelse af m. sphincter pupillae) og der vil vre manglende akkomodations.
6.2.10. Angiv den funktionelle betydning af konjugerede jenbevgelser og placeringen af de
neuronale centre i hjernestammen, der sikrer koordinationen af jenbevgelser.
Konjugering sikrer, at fikseringspunktet er ens p begge jne, sdan s vi undgr dobbeltsyn.
For konjugering er det ndvendigt, at de forskellige jenmuskler p begge sider arbejder sammen.
Dette sikres af blikcentrer, der forbinder de forskellige motoriske jenmuskel-kerner med hinanden.
De forskellige kerner kommunikerer med hinanden via fasciculus longitudinalis medialis.
Koordination af jenbevgelserne er forbundet med to omrder i hjernestammen:
Den paramediane pontine retikulrformation (PPRF): Ligger tt ved abducenskernen og
sender fibre til den. Kommunikerer ogs med oculomotoriuskernen p modsatte side.
Koordinerer alts horisontale bevgelser!
Rostral Interstitial Nucleus of the Medial Longitudinal Fasiculus (riMLF): Ligger i
mesencephalon tt ved oculomotoriuskernen. Kommunikerer med trochelearis-kernen og
oculomotorius-kernen. Styrer de vertikale bevgelse!
6.2.11. Hvordan klassificeres viljebestemte og reflektoriske jenbevgelser, og hvor udlses de
forskellige typer fra?
Der er flere forskellige slags jenbevgelser:
Sakkader: Sm hurtige bevgelser fra et punkt til et andet. Kan bde vre frivillige og
reflekstoriske.
Flgebevgelser: Langsomme bevgelser, der skal fastholde et objekt i bevgelse p samme
sted p retina.
Konvergensbevgelser: ndring af jets visuelle akse, nr et objekt kommer nrmere.
Opthokinetisk nystagmus: Blanding af sakkader og flgebevgelser. Ses nr omgivelserne
flytter sig ift. individet.
Vestibulr nygstagmus: Blanding af sakkader og flgebevgelser. Ses nr individet flytter sig
ift. omgivelserne. Skyldes impulser fra de vestibulre kerner.
6.2.12. Hvilke jensymptomer ses ved en mindre lokaliseret lsion af hjernestammens fasciculus
longitudinalis medialis p hjre side.
Ved ensidig afbrydelse vil der ses internuclear ophtalmoplegi med ophvelse af
adduktionsbevgelser (m. rectus medialis) af jet p samme side (her hjre side). Dette vil ske, da
der ikke er nogen kommunikation mellem occulumotoriuskernen og abducenskernen. Der vil vre
nystagmus p venstre je.
6.2.13. Angiv hvilke jensymptomerne, man vil forvente at finde hos en person med en mindre
Opgaver side 48

6.2.13. Angiv hvilke jensymptomerne, man vil forvente at finde hos en person med en mindre
lsion placeret i hjernestammens fasciculus longitudinalis medialis i den vre del af pons p
venstre side. Begrund kort svaret.
Ved ensidig afbrydelse vil der ses internuclear ophtalmoplegi med ophvelse af
adduktionsbevgelser (medieret af m. rectus medialis) af jet p samme side (her venstre side).
Dette vil ske, da der ikke er nogen kommunikation mellem occulumotoriuskernen og
abducenskernen. Der vil vre nystagmus p hjre je.
6.2.14. Redegr kort for den sandsynlige placering af en skade p nervesystemet ved flgende
observation: Belysning af svel hjre je som venstre je udlser kun lysrefleks p venstre, men
ikke p hjre je. Hvilke andre symptomer vil du forvente at finde hos den undersgte person?
Receptorne er intakte, de afferente refleksled er intakte og reflekscenteret er intakt. Det vestre
efferente led er intakt, men er ikke intakt p hjre side. Det efferente led er n. oculomotorius.
Manglende innervering af m. sphincter pupillae fra n. oculomotorius ville give samme symptomer
som her.
Et problem med oculomotoris ville ogs medfre mydriasis, ptosis (fordi m. levator palpebrae
superior mangler innervering), manglende akkomodation og jet vil vre stillet lateralt og nedad.
6.2.15. Redegr kort for den sandsynlige placering af en lsion i CNS, der medfrer, at en person
uden beskadigelse af n. oculomotorius ikke kan rette blikket (begge jne) mod venstre.
Kunne vre et problem med n. abducens eller nc. abducens for det venstre je. Dette ville medfre,
at hjre je ikke kunne abduceres og man ville ikke kunne se til venstre. Problemer i nc. abducenskernen ville dog muligvis medfre andre symptomer, da en fascialisskade tit ses sammen med en
skade p abducenskernen.
Kunne ogs vre et problem med fasiculus longitudinalis medialis, sdan s jnene ikke kan
koordineres til at lave en samlet bevgelse til venstre.
6.2.16. Redegr kort for funktionen af n. trochlearis, og hvilke symptomer (funktionsudfald), der
typisk ses ved en lsion af n. trochlearis sin.
N. trochlearis innerverer m. obliquus superior, der trkker jet inferiort og lateralt. En lsion af n.
trochlearis sin. vil p venstre je medfre svaghed, nr der skal kigges ned og lateralt. Desuden vil
jet stilles mere opadrettet.

Opgaver side 49

Kapitel 7: Synssansen
24. februar 2014
22:40

Opgaver side 50

7.1. Retina:
16. marts 2014
14:16

7.1.1. Beskriv retinas opbygning, herunder de strukturelle og funktionelle forskelle der er p


mellem centrale og perifere dele af retina:
Retina opdeles i flere lag:
1.
2.
3.
4.
5.
6.
7.
8.

Pigmentcellelaget
Ydre segment (stave og tappe)
Ydre kernelag (fotoreceptorernes cellelegemer)
Ydre synaptiske lag (synapser mellem fotoreceptorer og bipolre celler og horisontalceller)
Indre kernelag (bipolre neuroners cellelegemer)
Indre synaptisk lag (synapse mellem ganglion-celler og bipolr-celler og amakrin-celler)
Gangliecelle-laget
Axoner fra gangliecellerne.

P et fint billede:

Den centrale del af retina adskiller sig fra den perifere del:

Den centrale del af retina indeholder mange fotoceller, isr tappe. I retning af periferien falder
densiteten af tappene og der bliver flere stave. I fovea centralis er fotoreceptorene s tt p lyset s
muligt, da de andre cellelag er trngt vk, her er oplsningen skarpest.
Der er forskel p, hvordan tappene og stavene kontakter ganglie-celler:
Tap-celler: For tappene er der en lav grad af konvergens - hver tap-celle kontakter n eller
meget f ganglieceller.
Stav-celler: Stavene udviser strre konvergens hver ganglicelle kontaktes af mange
stavceller.

I papilla n. optici, hvor n. opticus lber ind i retina, er der ingen fotoreceptorer og alts en blind plet.
I hele retina gennem alle lag findes Mller celler, som er en speciel type glia.
EN vigtig pointe er desuden at aksonerne i retina ikke er myeliniserede, sledes lyset kan komme
igennem til photocellerne.
7.1.2. Redegr kort for, hvorfor det naturligt forekommende skotom svarende til papilla nervi
optici ikke opleves ved normalt binokulrt syn:
De to papilla n. optici ligger ikke korresponderende steder p retina og derfor vil lyset (under
normale omstndigheder) ikke falde p begge steder p n gang. Derfor vil man ikke opleve blinde
pletter.
Opgaver side 51

pletter.
7.1.3. Retina indeholder forskellige typer af ganglieceller. Hvad kaldes de forskellige typer, og
hvori bestr forskellene mellem dem?
Der er to typer:
M-celler: Registrerer bevgelser og kontraster i lysintensitet. Gr til det magnocellulre lag i
corpus geniculatum lateralis.

P-Celler: Registrerer farvesyn og er ansvarlige for synsskarpheden. Gr til det parvocellulre


lag i corpus geniculatum lateralis.
Begge typer kan vre ON- eller OFF-ganglie-celler.
7.1.4. Beskriv kort de srlige transmitterforhold mellem receptorceller og bipolre celler i retina.
Fotoceller er depolariserede i mrke. Dette skyldes at natriumkanaler str bne i mrke. Lys, der
rammer fotocellerne, frer til, at natriumkanalerne lukkes og dette medfrer, at cellen
hyperpolariserer.
Fotocellerne stopper med at udskille transmittersubstans nr de bliver hyperpolariseret (dvs. nr de
bliver ramt af lys).

Fotocellerne bruger glutamat som transmittersubstans.


Der er to typer af bipolr-celler:
OFF-bipolr: Hyperpolariseres af lys.
ON-bipolr: Depolariseres af lys.
P punktform:

1. Lyspvirkning medfrer hyperpolarisering af fotoceller ved lukning af natriumkanaler.


2. Hyperpolarisering medfrer reduceret transmitterfrigivelse fra fotoceller mere specifikt af
glutamat.
3. Den reducerede transmitterfrigivelse medfrer, at n type bipolrceller (ON-bipolarcelle)
disinhiberes, hvilket svarer til depolarisering.
4. Den disinhiberede/depolariserede bipolrcelle ger sin transmitterfrigivelse af glutamat til
gangliecellen.
5. Den anden bipolrcelle-type (OFF-bipolarcelle) hyperpolariseres og reducerer sin
transmitterfrigivelse af glutamat til gangliecellen.
6. Nr ON-bipolarcellen ger sin glutamat fristning til ON-gangliecellen, vil der ske en get
fyring af AP fra gangliecellen, hvilket signalerer om lys.
7. Signal fres i n. opticus via synsbanerne.
I retina findes to typer interneuroner, horisontalceller og amakrinceller.
Horisontalceller srger for lateral inhibition, ved at frigive GABA i synapsen mellem photocellerne og
bipolarcellerne. Lateral inhibering forbedrer kontrasten og skarpheden i synet.
Amakrinceller frigiver 7 forskellige transmittersubstanser i synapsen mellem bipolarcellen og
gangliecellen. Stavcellerne har udelukkende forbindelse til gangliecellerne via amakrincellerne.
Deres funktion er komplicerede.

Opgaver side 52

7.2. N. opticus:
16. marts 2014
14:16

7.2.1. Redegr kort for den cellulre opbygning af n. opticus og angiv hvorledes opbygningen
adskiller sig fra opbygningen af f.eks. n. vestibulocochlearis eller n. hypoglossus.
N. opticus er ikke en rigtig hjernenerve. Den er en fremskudt del af CNS og den har derfor ingen
egentlig kerne, ligesom f.eks. n. vestibulocochlearis eller n. hypoglossus.

Opgaver side 53

7.3. Synsbaner:
16. marts 2014
14:22

7.3.1. Redegr kort for forlbet af synsbanen fra de to retinae til synscortex med srlig fokus p
topografien af projektionen af de forskellige dele af synsfeltet og retinahalvdelene p synscortex,
inkl. begrebet "ocular dominance columns".
Den nasale del af synsfeltet projekteres til temporale del af retina, men temporale del af synsfeltet
projekteres til nasale del af synsfeltet.
Den nasale del af retina vil krydse over til modsattes sides tr. Opticus gennem chiasma opticum. Den
temporale del af retina krydser ikke, men fortstter til samme sides tr. Opticus.
Tr. Opticus fortstter til corpus geniculatum laterale i thalamus. P vej dertil afgiver den fibre til
prtektale kerner (ca. 10 %). Fibrene til prtektale kerner er vigtig for lysrefleksen og
akkomodationsrefleksen.
Fra corpus geniculatum laterale afgr radiatio optica, som inddeles i Meyers loop og Baums loop.
Meyers loop lber i temporallappen, mens Baums loop lber i paritallappen og occipitallappen.
Radiatio optica ender i area striata i occipitallappen (area 17), der ligger omkring sulcus calcarinus.

Retinotopisk ordning:
Ldering af n. opticus giver tab af syn p hele jet ipsilateralt.
Ldering af chiasma opticum giver tab af temporale del af synsfelt p begge jne (bitemporal
hemianopsi).
Ldering af tr. Opticus p giver tab af temporale del af synsfeltet p kontralaterale side og nasale
del af synsfeltet p ipsilaterale side.
Ldering af Meyers loop giver tab af synssans p kontralaterale, vre kvadrant p begge jne.
Ldering af Baums loop giver tab af synssans p kontralaterale nedre kvadrant p begge jne.

Ocular dominance columns:


Area striata modtager fibre fra korresponderende punkter p begge jne. Som regel sender det ene
je kraftigere signaler end det andet, hvilket bliver det dominerende je. Celler med dominans fra
det ene je danner bnd i area striata.
7.3.2. Angiv med udgangspunkt i synsbanernes topografi den mest sandsynlige lokalisation af en
lsion, der frer til blindhed svt. verste halvdel af venstre del af synsfeltet (venstre kvadrant
anopsi).
Hjre Meyers loop
7.3.3. Angiv med udgangspunkt i synsbanernes topografi for de mulige placeringer af en cerebral
lsion, der medfrer blndhed sv.t. nederste halvdel af den hjre del af synsfeltet (hjre nedre
kvadrantanopsi).
Venstre Baums loop
7.3.4. Redegr kort for den synsfeltsdefekt, der ses ved beskadigelse af de nederste fibre i capsula
internas retrolentikulre del (Meyers slynge) p hjre side.
Venstre, vre kvadrant anopsi.
Opgaver side 54

Venstre, vre kvadrant anopsi.


7.3.5. Redegr kort for hvilke synsforstyrrelser en hypofysetumor typisk kan forrsage.
Bitemporal hemianopsi dvs. tab af synet i temporal del af synsfeltet p begge jne.

Opgaver side 55

7.4. Synscortex:
16. marts 2014
14:22

7.4.1. Der nskes en beskrivelse af topografien i synsfeltets projektion p synscortex, herunder de


specielle forlb og endeomrder for tr. geniculocalcarinus (radiatio optica).
Fra corpus geniculatum laterale afgr radiatio optica, som inddeles i Meyers loop og Baums loop.
Meyers loop lber i temporallappen, mens Baums loop lber i paritallappen og occipitallappen.
Meyers loop p hjre side indeholder fibre fra venstre synsfelts vre kvadrant p begge jne.
Baums loop p hjre side indeholder fibre fra venstre synsfelts nedre kvadrant p begge jne.
Radiatio optica ender i area striata i occipitallappen (area 17), der ligger omkring sulcus calcarinus.
Her findes ligeledes retinotopisk opdeling. Korresponderende punkter p retina projekteres til
samme sted i korteks. Venstre synsfelt projekteres til hjre area striata og hjre synsfelt projekteres
til venstre area striata. Desuden vil vre del af synsfeltet ligge i nedre del af korteks, mens nedre del
af synsfeltet ligger verst p korteks.
Den centrale del af synsfeltet fylder mest i korteks i forhold til perifere dele af synsfeltet.

Nr man kigger p korteks udefra, kan man ikke se ret meget af synsomrdet, da de ligger nede i
selve sulcus calcarinus.
7.4.2. Redegr for beliggenhed og funktion af de forskellige visuelle associationsomrder,
herunder de cortikale omrder af betydning for perception og bevgelse, form og farve og
ansigtsgenkendelse.
De visuelle associationsomrder er area 18 og 19 i occipitallappen.
Ansigtsgenkendelse sker i et omrde i gyrus fusiformis p undersiden af temporallappen. Manglende
evne til at genkende kaldes agnosi. Manglende evne til at genkende ansigter kaldes derfor
ansigtsagnosi.
Farvegenkendelse sker i et omrde i gyrus lingualis og gyrus fusiformis, der ligger nedenfor sulcus
calcarinus. Manglende evne til at genkende farve, kaldes akromatopsi.
Genkendelse af bevgelse sker i et omrde p overgangen mellem temporallappen og
occipitallappen. Manglende evne til at genkende bevegelse kaldes akinetopsi.
7.4.3. Redegr kort for hvilke forstyrrelser i den cortikale bearbejdning af synsindtryk en lsion af
medialsiden af den forreste del af temporallappen vil medfre.
Hvis lsionen er meget medial vil man ldere tr. Opticus, hvilket vil fre til tab af nasale del af
synsfeltet p ipsilaterale je og tab af temporale del af synsfeltet p kontralaterale ge.

Hvis lsionen sidder medialt, men ikke helt ude ved 3. ventrikel, s vil Meyers loop lderes. Det vil
fre til tab af modsidig superior kvadrant anopsi.
7.4.4. Hvilket udfald i synsfeltet vil opst ved en selektiv beskadigelse af den del af area striata,
der ligger over den bagerste halvdel af fissura calcarina p hjre side?
Nedre del af synsfeltet p venstre side vil blive pvirket.
7.4.5. Forklar kort, hvilke symptomer der opstr efter en bilateral lsion af cortex omkring den
posteriore del af fissura calcarina.
Tab af syn centralt p begge jne. Den centrale del af retina projekteres netop til den posteriore del
af area striata.

Opgaver side 56

7.4.6. Redegr kort for hvilke synsforstyrrelser (funktionelle udfald)en tumor dybt beliggende
bagtil i temporallappen p venstre side typisk vil udlse.
Meyers loop lber i temporallappen. Derfor vil dette kunne beskades og vie modsidig vre
kvadrantanopsi.
7.4.7. Redegr kort for hvilke synsforstyrrelser en lsion af den bagerste del af nakkelappen
(polus occipitalis) typisk vil medfre).
Polus occiipitalis er nakkepolen, alts den allerbagerste del af baglappen, hvor area striata ligger. Er
skaden bilateral fs total blindhed, mens unilateral lsion vil fre til totalt tab afmodsidig del af
synsfeltet p begge jne.
7.4.8. Redegr kort for hvilke forstyrrelser i den cortikale bearbejdning af synsindtryk, der typisk
vil forrsages af en lsion placeret i hjernebarken p lateralsiden af hemisfren ca. midt mellem
den primre synscortex og Brodmanns area 5 og 7.
Der vil ske en pvirkning af associationsfibre mellem primr synskorteks og brodsmanns area 5 og 7,
som er af betydning for integration af visuel, auditiv og somatosensorisk information. Derved vil man
ikke kunne reagere p visuelle indtryk og omstte til motorisk handling. Dette kaldes apraksi.
7.4.9. Redegr for den arterielle blodforsyning af synscortex.

Synskorteks forsynes af a. cerebri posterior med anastomoser.


7.4.10. Redegr kort for den arterielle blodforsyning af synssansens associationsomrder p
medial- og undersiden af temporallappen.
Disse omrder forsynes af a. cerbri anterior.
7.4.11. Anfr det synsfeltudfald, der kan forventes ved okklusion af arteriegrenene til primr
synscortex beliggende under fissura calcarina p venstre side.
vre, hjre del af synsfeltet p begge.
7.4.12. Forklar kort, hvordan manglende synsindtryk under en kritisk periode af udviklingen skulle
kunne give varige problemer med f.eks. koordination af synsindtryk med motorik.
Brug af jnene tidligt i barndommen lgger grundlaget for at primr synskorteks kan arbejde
sammen med accosiationsomrderne. Synssystemet vil ikke virke hensigtsmssigt, hvis synet
blokeres. Dermed giver det problemer med motorisk koordination.
7.4.13. Definer, hvad du forstr ved simple, komplekse og hyperkomplekse neuroner i synscortex
og redegr kort for deres respektive funktioner.
Simple celler: kan registrere lysstriber eller lys/mrke overgange, hvis cellerne er orienteret i en
bestemt retning. De er alts stedsspecifikke
Komplekse celler: reagerer p stimuli inden for et strre omrde, s lnge striben var rigtigt
orienteret. De er vigtige for opfattelse af konturer.

Opgaver side 57

7.5. Synsreflekser:
16. marts 2014
14:43

7.5.1. Redegr kort for jets akkomodationsrefleks, herunder de involverede nerveforbindelser.


Receptor: Photoceller p retina
Afferent forbindelse: N. opticus
Reflekscentrum: prtektale kerner og Erdinger Westpahls kerner.
Efferent fobindelse: n. occulomotorius via. ganglion ciliare
Effektor: m. ciliare
Nr en genstand fres tt p jnene vil jnene akkomodere for at stille skarpt p genstanden. Dette
gres ved at linsen krummes.
7.5.2. Redegr kort for jets lysrefleks, herunder de involverede nerveforbindelser.
Receptor: Photoceller p retina

Afferent forbindelse: N. opticus


Reflekscentrum: prtektale kerner og Erdinger Westpahls kerner.
Efferent fobindelse: n. occulomotorius via. ganglion ciliare (der dannes synapse mellem pr- og
postsynaptiske fibre i ganglion ciliare)
Effektor: m. sphinchter pupillae

Hvis en person lyses i den ene je, vil pupillen i begge jne trkke sig sammen.

Opgaver side 58

7.6. jenmuskler:
16. marts 2014
14:45

7.6.1. En patient fremviser en total venstresidig facialisparese. Ved forsg p at se til venstre kan
venstre og hjre je ikke bevges, mens de vrige jenbevgelser i begge jne er normale.
Benvn de kranienerver, der er involveret i denne defekt, og redegr kort for hvordan en
lokaliseret skade kan give disse symptomer.
P venstre je: m. rectus lateralis der ikke virker (innerveret af. N. abducens)
P hjre je: m. retus medialis der ikke virker (innerveret af n. occulomotorius)
En skade kunne vre i PPRF (horisontal blikcenter) eller i abducenskernen.
Angiv hvilke andre symptomer man kunne forvente ogs ville optrde hos den pgldende
patient.
Pt. vil have hngende jenlg og mundvig p den ene side som flge af, at alle mimiske muskler er
lammet grundet total facialis parase. Patienten kan heller ikke rynke panden.
7.6.2. Angiv de symptomer, du vil forvente at finde ved en lsion bagtil/dorsalt i den caudale del
af pons p venstre side, som involverer nc. n. abducentis, og redegr for hvilke strukturer (nerver,
nervebaner, kerneomrder), der er knyttet til de forskellige symptomer.
Man vil have symptomer svarende til abducenskernen og facialis perifere forlb.

Manglende evne til at abducere venstre je pga. lammelse af venstre m. rectus lateralis innerveret n.
abducens.
Helsidig lammelse af mimiske muskler p venstre side (hngende mundvig og jenlg og manglende
evne til at rynke panden) pga. perifer facialis parese.
Nedsat eller manglende evne til spyt- og tresekretion pga. beskadigelse af n. facialis, som
indeholder n. intermedius.
7.6.3. Personen p fig. 7.1. forsger at se til venstre. Angiv hvilke(n) nerve(r), der er ramt, og
redegr kort for om lsionen ligger i nerven, nervekernen eller et andet sted i CNS.
Venstre m. rectus lateralis er lammet. Den er innerveret af. N. abducens, som s m vre beskadiget
7.6.4. Redegr kort for de symptomer (funktionelle udfald), der ses p overstende fotografier
(fig. 7.2) af en patient, der er bedt om at se lige frem. I redegrelsen br indg navn p
kranienerve, placering af nervekerner i CNS, det apparente udspring fra CNS, de innerverede
muskler og musklernes virkning.
Kvinden har en occulomotorius parese p hjre je. Der ses ptose (hngende jenlg), da m. levator
palpebra superior er lammet. Desuden er jet nedad og lateralt stillet, fordi der stadig er tonus i m.
rectus lateralis og m. obliquus superior (innerveret af n. abducens og n. trochearis hhv). Resten er
musklerne er lammet pga. lammelse af n. occulomotorius.
Occulomotorius har to kerner. En viceral efferent (Erdinger Westphals kerne) og en somatisk
efferent kerne. Begge kerner ligger i mesencephalon og n. occulomotorious har apperent udspring i
fosse interpeduncularis (p forsiden af mesencephalon).
7.6.5. Benvn de med A-F markerede strukturer p figur 7.3. og angiv funktionen af B, E og F.
A: retina
B: Corpus ciliare med m. ciliaris (ved kontraktion afslappes zonulatrdene, hvorved linsen krummes)
C: Zonulatrde
Opgaver side 59

C: Zonulatrde
D: Cornea
E: Iris (regulerer pupillens diameter, hvilket bestemmer mngden af lys, som rammer retina)
F: Processus ciliaris (producerer kammervske)

Opgaver side 60

Kapitel 8: Hresansen og ligevgtssansen


24. februar 2014
22:40

Opgaver side 61

8.1. Hreorganet:
10. marts 2014
19:55

8.1.1. Lav en vellignende, skematisk tegning af det cortiske organ og benvn de forskellige
strukturer. Benvn de to typer af receptorceller og angiv deres funktion.
Organet:

Receptorcellerne er:
Indre hrceller: Den egentligt receptive celle, der sender signaler om lyd til CNS via n.
cochlearis.

Ydre hrceller: Celler, der regulerer sensitiviteten for lyd.


Bevgelse af stereocilierne, p grund af bevgelser af membrane basilaris og perilymfe, medfrer
receptorpotentialer i de indre hrceller. Responset fra cellerne vil dog vre forskellige. De indre
hrceller vil frigive glutamat og de ydre hrceller vil kontrahere.
Punktvis:
1. Lyd kommer ind igennem det ydre re.
2. Lyd rammer trommehinden og stter den i svingning.
3. Malleus sttes i bevgelse sammen med trommehinden.
4. Vibrationerne overfres til incus.
5. Vibrationerne overfres fra incus til stapes.
6. Basis af stapes, der lukker det ovale vindue, sttes i svingninger.
7. Svingninger overfres til perilymfe i scala vestibuli.
Opgaver side 62

7. Svingninger overfres til perilymfe i scala vestibuli.


8. Svingninger overfres til endolymfen i scala media via membrana vestibularis.
9. Basilarmembranen presses nedad.
10. Basilarmembranens pres stter perilymfen i scala tympani i svingninger.
Alts: Basilarmembranen bevger sig i takt med stapes svingninger og flgelig bevger
endolymfen sig i takt med stapes svingninger.
11. En bevgelse af basilarmembranen og endolymfen medfrer en bevgelse af cilierne p de
indre hrceller, der er receptorcellerne.
12. Nr de indre hrcellers cilier bevger sig i det lngste cilies retning, trkker tip-links p
cilierne i mekaniske kation-kanaler, der bner.
13. Nr kanaler bner strmmer der kalium ind fra endolymfen. Dette medfrer en depolarisering.
14. Depolariseringen medfrer bning af spndingsstyrede calciumkanaler.
15. Den gede influx af calcium medfrer calciummedieret exocytose af glutamat til den
synaptiske klft, hvor det binder sig til AMPA-receptorer.
P denne mde dannes der aktionspotentialer, der ledes til hjernestammen.

8.1.2. Tegn et tvrsnit gennem en af cochleas vindinger og angiv beliggenheden af scala vestibuli,
scala tympani og ductus cochlearis. Indtegn desuden de vigtigste strukturer i ductus cochlearis.

8.1.3. Angiv den tonotopiske reprsentation i cochlea, og anfr en mulig forklaring herp.
Forskellige frekvenser registreres forskellige steder langs basalmembranen. Dette kaldes tonotopisk
lokalisering:
Toner med den hjeste frekvens registreres af hrcellerne tttest p det ovale vindue (dvs.
mest basalt i membrana basilaris).
Toner med den laveste frekvens registreres tt ved toppen af cochlea (dvs. mest apikalt i
membrana basilaris).
Den tonotopiske ordning skyldes formentligt basilarmembranens fysiske egenskaber. Membranen er
Opgaver side 63

Den tonotopiske ordning skyldes formentligt basilarmembranens fysiske egenskaber. Membranen er


smallest basal og bliver bredere apikalt.

Opgaver side 64

8.2. Beskyttelsesreflekser for hreorganet:


10. marts 2014
19:55

8.2.1. To muskler kan beskytte det cortiske organ mod f.eks. for hje lydtryk. Benvn de to
muskler og angiv deres innervation og deres virkemde.
De to muskler er:
M. stapedius: Musklen hfter p stapes og kontrahere ved j lydintensitet. Dette trkker
fodpladen ud af det ovale vindue.
Innerveres af n. stapedius fra n. facialis.
M. tensor tympani: Spnder ved hj lydintensitet trommehinden , sdan s den ikke kan
svinge s meget.
Den innerveres fra n. mandibularis.
8.2.2. Angiv hvad der udlser stapedius-refleksen. Hvad forsts ved stapediusrefleksen? Skitsr
refleksbuen fra receptor til effektor.
Stapediusrefleksen udlses af hj lydintensitet. Nr refleksen udlses vil stapes fodplade trkkes
vk fra det ovale vindue via m. stapedius, sdan s hrelsen beskyttes.
Refleksen er:
Stimuli: Hj lydintensitet.
Receptor: Indre hrceller i det cortiske organ.
Afferent led: N. vestibulocochlearis.
Refleks-centrum: Formatio reticularis.
Efferent led: N.facialis -> n. stapedius
Effektor: m. stapedius, som trkker stapes vk fra det ovale vindue.

Respons: Lydintensiteten i det cortiske organ snkes. Indre hrceller beskyttes.


8.2.3. Angiv to beskyttelsesreflekser for hreorganet og de dertil knyttede afferente og efferente
nerveforbindelser.

De to reflekser er:
Stapedius-refleksen (knyttet til m. stapedius): Den afferente forbindelse er n.
vestibulocochlearis og den efferente forbindelse er n. stapedius fra n. facialis.
Tensor tympani-refleksen (knyttet til m. tensor tympani): Den afferente forbindelse er n.
vestibulocochlearis og den efferente forbindelse er n. mandibularis fra n. trigeminus.

Opgaver side 65

8.3. Centrale hrebaner:


10. marts 2014
19:58

8.3.1. Beskriv forlbet af de centrale hrebaner, herunder placering af synapser og


krydsningsforhold, og benvn de involverede baner.
I begge side lber fibrene sdan her til colliculus inferior:
1. Hrcellerne danner synapse med n. cochlearis lige over membrana basilaris
2. N. vestibulocochlearis lber fra hrcellerne til nc. cochlearis dorsalis og nc. corchlearis ventralis
i hjernestammen. De har deres soma placeret i ganglion spirale.
3. Fibrene fra den ventrale kerne gr direkte til begge oliva superior, hvor de danner synapse.
4. Fra oliva superior lber fibre direkte til samsidige og kontralaterale colliculus inferior.
5. Nogle af de krydsede fibre fra nc. cochlearis ventralis lber udenom oliva superior og direkte
til den kontralaterale colliculus inferior.
6. Fibre til colliculus inferior, bde direkte og indirekte, lber i lemniscus lateralis i kontralateralt
for hreorganet.
7. Fibrene fra nc. cochlearis dorsalis lber direkte til modsatte colliculus inferior.
Nu er vi i colliculus inferior. Nu skal fibrene til auditivt cortex:
1. Fibre fra colliculus inferior lber til bde modsatte og samsidige corpus geniculatum mediale i
thalamus.
2. Fra corpus geniculatum mediale lber fibrene til samsidige primre auditive cortex via capsula
internas crus posterior
8.3.2. Angiv om en ensidig lsion af auditiv cortex giver anledning til dvhed.
Der vil ikke vre et tydeligt tab af hrelse, fordi fibrene krydser s mange gange p forskellige
niveauer. Men der vil vre problemer med at lokalisere, hvor lyd kommer fra.

8.3.3. Hvad er funktionen af nc. olivaris superior (oliva superior)?


Den laterale del af kernen spiller en rolle i forhold til lokalisering af lyd med grundlag i forskellen i
lydstyrken og den mediale del af kernen kan lokalisere lyd med baggrund i tidsforskellen mellem
lokaliseringen af lyden.
Olivacochleare fibre kan modulere de indre hrcellers aktivit ved at modulere ydre hrcellers
kontraktilitet.
Neuroner i nc. olivaris superior fungerer som interneuroner i stapediusrefleksen.
8.3.4. Hvordan bestemmes en lydkildes placering i rummet omkring os?
Den laterale del af nc. olivaris superior er med til at lokalisere lyden p baggrund af forskelle i
lydstyrke.
Den mediale del af nc. olivaris superior er med til at lokalisere lyd p baggrund af tidsforskellen i
opfangelsen af signalerne fra de to sider.

Opgaver side 66

Kernen modtager billaterale innervation fra nc. cochlearis.


Oliva superior er primrt ansvarlig for horisontal lydbestemmelse. Nc. cochlearis dorsalis er primrt
ansvarlig for vertikal lydbestemmelse.
8.3.5. Angiv placeringen af hrecortex.
Primrt hrecortex (A1) er lokaliseret i temporallappen omkring gyrus temporalis superior (sv.t.
area 41). Omkring area 41 er der et hreblte, der ogs er involveret i hrelsen.
8.3.6. Hvor i cortex er de dybe og hje tone reprsenteret?
Cortex er tonotopisk organiseret.
De lave frekvenser (dybe toner) findes mere rostralt i cortex end de hjere frekvenser (hjere toner),
der findes mere caudalt.
8.3.7. Angiv den arterielle karforsyning af hrecortex.
A. cerebri media.
8.3.8. Angiv det forventede funktionelle udfald ved en ensidig, venstresidig beskadigelse af
hrecortex.
Der lber bde kontralaterale fibre og ipsilaterale fibre til cortex og derfor vil der ved en
beskadigelse af venstre sides A1 ikke opst dvhed i en af siderne, da innervationen er bilateral.
Der vil dog vre problemer med at lokalisere lyd, der kommer fra modsatte side af beskadigelsen.

Opgaver side 67

8.4. Trommehinden:
10. marts 2014
20:03

8.4.1. En pusansamling i mellemret kan udtmmes gennem et lille snit i trommehinden


(paracentese) - sdvanligvis i nedre, bagerste kvadrent for at undg beskadigelse af en reknogle
og en nervegren. Benvn reknoglen og nervegrenen.
Man undgr at beskadige malleus hfte til trommehinden og chorda tympani.
Chorda tympani kommer fra n. facialis og lber fra bagvggen gennem den laterale del af
trommehulen ud for den verste del af trommehinden.
8.4.2. Benvn de nerver/nervetrde, der lber ind i eller igennem cavitas tympanica, og angiv for
hver af disse deres funktioner og perifert innerverede strukturer.
N. tympanicus (fra n. IX): Danner plexus tympanicus.
N. petrosus minor (fra n. IX): Gr til ganglion oticum, hvor den danner synapse med
postganglionre neuroner, som via n. auriculotemporalis innerverer gl. parotidea sekretorisk.
N. stapeidus (fra n. VII): Indeholder somatisk-efferente fibre til m. stapedius.
Chorda tympani (fra n. VII)i: Lber gennem mellemret og innerverer gl. submandibularis og
gl. sublingualis.

Opgaver side 68

8.5. Biofysik:
10. marts 2014
20:06

8.5.1. Benvn de lydledene strukturer fra meatus acusticus externa til det Cortiske organ.
I rkkeflge:
1.
2.
3.
4.
5.
6.
7.
8.

Trommehinden (membrana tympanica)


Malleus
Incus
Stapes
Det ovale vindue (fenestrae vestibuli)
Perilymfe i scala vestibuli
Perilymfe i scale tympani
Membrana basilaris sttes i svingninger, hvilet pvirker de indre hrceller.

8.5.2. Angiv enheden for lydstyrke og det normale frekvensomrde for menneskets hrelse.
Standarden for lydstyrken er decibell (dB). Decibel er defineret som logaritmen til det aktuelle
lydtryk divideret med et veldefineret referencetryk.
Det normale frekvensomrde er 20 Hz til 20.000 Hz.

Opgaver side 69

8.6. Ligevgtsorganet:
10. marts 2014
20:08

8.6.1. Beskriv kort, gerne suppleret med en tegning, balanceorganets strukturelle opbygning,
herunder type, placering og funktion af receptorceller..
Den vestibulre del af labyrinten bestr af:
Utriculus (en lille vesikel)
Sacculus (en lille vesikel)
Tre buegange, der er tilhftet utriculus.
Hver buegang har en ampulla tt p utriculus. De tre buegange er orienteret vinkelret p hinanden i
tre planer, hvoraf en buegang i neutral stilling str horisontalt.
Der findes sensorisk epithel med hrceller fem forskellige steder i den vestibulre labyrint:
De tre ampuller fra buegangene
Utricullus
Sacculus
Hrcellerne minder om dem i cochlea, men har ogs et lang kinocilie. Hrceller i ampullerne str p
en crista. Cilierne fra hrcellerne projicere ind i cupula, der er en geleagtig masse, der ligger over
hrcellerne p crista.
I utriculus og sacculus findes der ogs hrceller, disse celler findes i maculae sacculi og macuale
utriculi. De to maculae er ogs dkket cupula. Oven p cupula i maculae er der indlejret sm sten otolitter.
Utriculus optager horisontale signaler og sacculus optager signaler i det vertikale plan.

8.6.2. Hvilke omrder indgr i det vestibulre sanseorgan, og hvad er de adkvate sensoriske
stimuli.
De sensoriske omrder er:
De tre ampuller i buegangene
Sacculus
Utriculus

Opgaver side 70

Det er ikke alle rotationsbevgelser, men ndringer af rotationsbevgelsens hastighed (i.e.


accereltion eller decelleration) der er den adkvate stimuli for buegange - buegangene har alts
dynamisk flsomhed!
Utriculus monitorerer laterale vip og sacculus monitorerer horisontale vip (i.e. fleksion-ekstensionsbevgelser i hovedet). Utriculus og sacculus kan hele tiden sende information om hovedet stilling:
De har alts statisk sensitivitet, dog ger deres fyring ogs ved acceleration pga. otolitternes inerti.

Opgaver side 71

8.7. Centrale ligevgtsbaner:


10. marts 2014
20:10

8.7.1. Benvn de vigtigste afferente og efferente forbindelser til og fra det vestibulre
kernekompleks, og nvn mindst to reflekser i relation til navngivne forbindelser.
Afferente forbindelser:
Signaler fra buegangene og fra utriculus og sacculus konvergerer mod de vestibulre kerner og mod
cerebellum i n. vestibularis.

Nc. vestibularis ligger som en del af et vestibulrt kompleks, lige under fjerde ventrikel, der
indeholder flere kerner:

Nc. vestibularis superior


Nc. vestibularis lateralis
Nc. vestibularis inferior
Nc. vestibularis medialis

Nc. vestibularis modtager ogs signaler fra cortex. Primrt via formatio reticularis.
Efferenter forbindelser fra:
Nc. vestibularis virker p tre hovedregioner:
Motorneuroner i rygmarven
Motorneuroner i jenmuskelkernerne
Cerebellum
De fleste fibre til rygmarven kommer fra nc. vestibularis lateralis. De danner tr. vestibularis lateralis.
Tr. vestibularis lateralis virker kraftigt p muskler, der opretholder vores holdning og balance. Der er
ogs en mindre tr. vestibularis medialis, der stammer fra nc. vestibularis medialis. Banen virker
primrt p muskler, der bevger hovedet.
Fibre til jenmuskelkernerne stammer primrt fra de nc. vestibularis superior og nc. vestibularis
medialis. Fibrene lber i fasciculus longitudinalis medialis og ender i kernerne for n. abducens, n.
trochlearis og n. oculomotorius. Fibrene i fasiculus longitudinalis medialis er bde krydsede og
ukrydsede.
Vestibulo-cerebellare fibre kommer fra nc. vestibularis og ender i vestibulo-cerebellum. Disse fibre
kan justere sensitiviteten af den vestibulo-okulre refleks, fordi de gr tilbage og pvirker.

Opgaver side 72

8.8. Vestibulre reflekser:


10. marts 2014
20:13

8.8.1. Angiv to forskellige typer af vestibulre reflekser og angiv deres funktion beskrevet ud fra
den klassiske opbygning af en refleks.
Labyrintreflekserne: Justerer kroppens holdning. Medieres gennem de vestibulo-spinale baner
(tr. vestibulospinalis lateralis og medialis).
Opbygning:

Receptor: Labyrinten.
Afferent led: N. vestibulocochlearis.
Reflekscentrum: Primrt det vestibulre kernekompleks.
Efferent led: Tr. vestibulospinalis lateralis og medialis.
Effektor: Skeletmuskulatur, der enten kontrahere tonisk eller fasisk.
Vestibulo-oculre reflekser: Justerer jnenes stilling og bevgelser ift. hovedet. Srger for, at
billedet vi ser holdes stabilt p retina.
Opbygning:
Receptor: Buegangene.
Afferent led: N. vestibulocochlearis.
Reflekscentrum: Primrt det vestibulre kernekompleks, men det pontine blikcenter er ogs
involveret. Reflekscenteret er spredt ud i pons.
Efferent led: Fasciculus longtudinalis medialis til jenmuskelkerner og herfra hjernenerver til
jenmuskler.
Effektor: Ekstraoculre muskler.
8.8.2. Angiv de udlsende stimuli for labyrint-refleksen og virkningen heraf p
ekstremitetsmuskulaturen.
Refleksen udlses ved bevgelse eller rotation af hovedet. Information sendes til de mediale og
laterale vestibulariskerner, som formidler informationen videre til rygmarven via tr. vestibulospinalis
lateralis og medialis.
Dette medfrer tonisk eller fasisk kontraktion af forskellige muskler, sdan s balancen kan
genoprettes.
8.8.3. Beskriv kort, gerne suppleret med en tegning, de forskellige neurale strukturer i den
vestibulo-okulre refleks, og angiv refleksens funktion.

De vestibulo-okulre reflekser justerer jnenes stilling og bevgelser ift. hovedet. Srger for, at
billedet vi ser holdes stabilt p retina.
Opbygning:
Receptor: Buegangene.
Afferent led: N. vestibulocochlearis.
Reflekscentrum: Primrt det vestibulre kernekompleks, men det pontine blikcenter er ogs
involveret. Reflekscenteret er spredt ud i pons.
Efferent led: Fasciculus longtudinalis medialis til jenmuskelkerner og herfra hjernenerver til
jenmuskler.
Opgaver side 73

jenmuskler.
Effektor: Ekstraoculre muskler.

8.8.4. Anfr forskellen p en vestibulo-okulr refleks og en optokinetisk refleks og benvn deres


respektive receptorceller.
De vestibulo-okulre reflekser justerer jnenes stilling og bevgelser ift. hovedets bevgelser.
Receptoren er buegangene.
De vestibulo-okulre reflekser justerer jnenes stilling og bevgelser ift. omgivelserne. Receptoren
er retina.
Begge reflekser srger for, at et billede fastholdes p retina.
8.8.5. Redegr kort for de motoriske reaktioner, der kan observeres hos en stende person lige
efter stop af en ca. 2 minutter lang rotation mod hjre i stende stilling omkring personens
lngdeakse.
Patienten vil udvise post-rotatorisk nystagmus. Der vil vre hurtige sakkadebevgelser mod hjre
og langsommere flgebevgelser tilbage til venstre.
Der vil vre faldtendens.

Opgaver side 74

Kapitel 9: Cerebellum og basalganglier


24. februar 2014
22:06

Opgaver side 75

9.1. Cerebellum:
24. februar 2014
22:06

9.1.1. Beskriv cerebellums makroskopiske opbygning:


Cerebellum ligger i fossa cranii posterior dorsalt for hjernestammen og under cerebrum. Den er
forbundet til hjernestammen via de tre hjernestilke:
Pedunculus cerebelli superior
Pedunculus cerebelli medius
Pedunculus cerebelli inferior
Den midterste del af cerebellum kaldes for vermis. P hver side af vermis findes de cerebellare
hemisfrer. Hver hemisfrer er forbundet til flocculus, der ligger i forlngelse af vermis.

Cerebellum er opdelt i lobus fluccolonodularis og corpus cerebellum. Corpus cerebellum er opdelt i


en anterior og en posterior del. Cerebellum er strkt foldet og folderne er arrangeret transverst og
danner folia
Cerebellum er dkket cortex cerebelli, der bestr af gr substans og er delt i tre lag. Under den gr
sub substans findes der hvid substans. Dybt i den hvide substans findes der igen gr substans, der
udgr de dybe cerebellare kerner:
Nc. dentatus
Nc. emboliformis
Nc. globosus
Nc. fastigii
9.1.2. Angiv hvilken del af det primitive hjerneanlg cerebellum deriveres fra.
Cerebellum er udviklet fra kanterne af alarpladerne.
9.1.3. Beskriv den histologiske opbygning af de cerebellare cortex.
Cerebellums cortex er inddelt i tre cellelag. Fra yderst til inderst:
Det molekylre lag: Det yderste cellelag. Indeholder primrt dendritter og axoner fra de
dybere cellelag.
Purkinjecelle-laget: Det midterste lag. Bestr af et lag purkinjecelle-cellelegemer.
Purkinjecellerne har meget forgrenede dendritter og dendrittret er komprimeret i t plan,
der ligger vinkelret ift. de parallelle fibre fra de granulre celler. Purkinjecellerne er de eneste
celler, der sender deres axoner ud af cortex. De er dermed Cortex eneste efferente kanal!
Purkinjecellerne bruger GABA som neurotransmitter og virker inhibitorisk p de celler, som de
sender efferenter til.
Det granulre lag (korncelle-laget): Granulre celler (kornceller) sender deres axoner op
igennem purkinjecelle-laget og op i det molekylre lag. I det molekylre lag deler fibrene sig i
to grene, der danner synapser med dendritter fra purkinjecelle-laget med stor konvergens.
Cellerne er glutamerge og virker excitatoriske p purkinjecellerne.
9.1.4. Redegr kort for purkinjecellens placering i cerebellums neurale kredslb.
Purkinjecellerne findes i det midterste lag af cerebellums cortex (purkinjecelle-laget).
Purkinjecellerne modtager mange afferenter fra korncellerne (de parallelle fibre konvergerer mod
purkinje-dendritterne). Korncellerne er glutamerge og virker excitatoriske p purkinjecellerne.
Purkinjecellerne selv virker inhibitoriske p deres ml via GABA.

Purkinjecellerne sender deres axoner til kerner i cerebellum via de corticonuklre fibre. Fra
kernerne sendes axoner ud til cerebellums forskellige ml.
Opgaver side 76

kernerne sendes axoner ud til cerebellums forskellige ml.


9.1.5. Redegr kort for udlsning af complex spikes i Purkinjecellerne:
Hver klatrefiber danner mange synapser med en eller f purkinjeceller. Selvom det enkelte EPSP fra
en klatrerfiber-synapse er smt bliver den samlede virkning stor (spatial summation). Dvs. at et
aktionspotentaile i en klatrende fiber kan udlse bursts af potentialer i purkinjecellerne. Disse bursts
kaldes for complex spikes.
9.1.6. I cerebellum findes en speciel type astroglia. Benvn disse.
Bergmann-celle.
9.1.7. Angiv navnene p de dybe cerebellare kerner og de enkelte kernes afferente og efferente
projektionsomrder.

Kernerne er:
Nc. dentatus: Nogle fibre lber fra nc. dentatus til nc. ruber. Andre lber til thalamus, hvor
fibrene kan sendes videre til f.eks. motorcortex.
Nc. globosus: Nc. globosus modtager afferenter fra intermedirzonen og sender efferenter
fibre til modsatte sides thalamus og til nc. ruber. Derfra kan kernerne pvirke motorneuroner.
Nc. emboliformis: Nc. emboliformis modtager afferenter fra intermedirzonen og sender
efferenter fibre til modsatte sides thalamus og til nc. ruber. Derfra kan kernerne pvirke
motorneuroner. Nc. emboliformis virker alts p samme mde som nc. globosus.
Nc. fastigii: Sender fibre til nc. vestibularis og formatio reticularis. De kan pvirke
motorneuroner via reticulo- og vestibulospinale baner. (hvor modtager den fra?!)
9.1.8. Benvn cerebellums vsentlige eksterne afferente og efferente forbindelser med angivelse
af deres krydsningsforhold.
Cerebellum modtager eksterne afferenter fra:
Det vestibulre apparat / nc. vestibularis (via nedre lillehjerne-stilk).
Muskelspoler, senespoler og lavtrskels-mekanoreceptorer fra medulla spinalis (via vre og
nedre lillehjerne-stilk).
Fra interneuroner i medulla spinalis (via vre og nedre lillehjerne-stilk).
Pontine kerner, der er blevet kontaktet af cortex cerebri (lber ind via midterste lillehjernestilk).
De efferente forbindelser lber fra kernerne:
Nc. dentatus: Nogle fibre lber fra nc. dentatus til nc. ruber. Andre lber til thalamus, hvor
fibrene kan sendes videre til f.eks. motorcortex.
Nc. globosus: Nc. globosus modtager afferenter fra intermedirzonen og sender efferenter
fibre til modsatte sides thalamus og til nc. ruber. Derfra kan kernerne pvirke motorneuroner.
Nc. emboliformis: Nc. emboliformis modtager afferenter fra intermedirzonen og sender
efferenter fibre til modsatte sides thalamus og til nc. ruber. Derfra kan kernerne pvirke
motorneuroner. Nc. emboliformis virker alts p samme mde som nc. globosus.
Nc. fastigii: Sender fibre til nc. vestibularis og formatio reticularis. De kan pvirke
motorneuroner via reticulo- og vestibulospinale baner.
Opgaver side 77

9.1.9. Cortex cerebelli beskrives som funktionelt opdelt i vestibulocerebellum, spinocerebellum og


pontocerebellum. Angiv med navn, oprindelse og forlb de afferente forbindelser, der er
specifikke for hvert enkelt af disse omrder.
De tre opdelinger er:
Vestibulocerebellum: Vestibulocerebellum modtager primre fibre fra ligevgtsapparatet via
tr. vestibulocerebellaris (der lber i pedunculus cerebelli inferior). Der lber ogs sekundre
fibre fra. Nc. vestibularis til vestibulocerebellum.
Spinocerebellum: Spinocerebellum modtager direkte fibre fra medulla spinalis. Fibrene sender
information fra muskelspoler, senespoler og lavtrskels-mekanoreceptorer i huden til vermis
og den intermedire zone. Spinocerebllum modtager ogs direkte fibre fra spinale
interneuroner. Der er ogs indirekte trakter, der afbrydes synaptisk i oliva inferior. Fibrene fra
oliva inferior ender i den kontralaterale side af de cerebellare hemisfrer.
Cerebrocerebellum: De pontocerebellare fibre ender primrt i hemisfrerne. De pontine
kerner modtager afferenter fra cortex cerebri via den corticopontine trakt, der lber i capsula
interna. Fibrene kommer fra MI, SI, PMA og SMA. De fleste pontocerebellare fibre krydser i
pons. Den corticopontine bane er ukrydset.
9.1.10. Forklar udfra forskellige nerveforbindelser, hvorfor der er forskelle i symptomerne p en
lsion af hhv. cerebellums vermale/paravermale dele og de cerebellare hemisfrer.
De vermale og paravermale dele indeholder primrt spino-cerebellum. De cerebellare hemisfrer
indeholder primrt cerebro-cerebellum.
Ved skader p spino-cerebellum vil man se udfald svarende til proprioception (og balanceudfald, hvis
man lukker jnene). Dette skyldes, at cerebellum ikke lngere kan monitorere aktivitet fr og efter
motorneuronerne og at cerebellum derfor ikke lngere kan justere bevgelserne efter forholdene.
Dette vil give ataxi.
Ved skader p cerebro-cerebellum vil der ses udfald sv.t. planlgningen af bevgelser. Formentligt
vil der ses hypokinesi, dysmetri og dysdiadochokinesi.
9.1.11. Unilaterale lidelser i cerebellum medfrer unilaterale motoriske forstyrrelser. Anfr disse
forstyrrelser eller symptomer og de relevante forbindelse mellem de cerebellare hemisfrer og
det motoriske system.
Ved skader p vestibulo-cerebellum ses balanceproblemer.
Ved skader p hemisfrerne (cerebro-cerebellum) ses dysdiadochokinesi, dekomposition osv.

En skade p vermis (spino-cerebellum) leder til manglende prcis styring af den aksiale grove
muskulatur samt balancesans via de ekstrapyramidale baner. Dvs. ataxi.
9.1.12. Angiv mindst tre vsentlige typer af symptomer, som typisk optrder ved skade eller
sygdom i cerebellum.
Det afhnger af, hvor skaden sidder, men ofte vil man se:
Ataxi
Dekomposition
Balanceproblemer
Hypokinesi
Dysdiadochokinesi
9.1.13. Redegr kort for oprindelse, forlb, endeomrder og funktion af de spinocerebellare
baner.
Der er direkte og indirekte baner:
Opgaver side 78

Der er direkte og indirekte baner:


De direkte baner lber uafbrudt fra medulla spinalis til cerebellum. Der er to baner:
Tr. spinocerebellaris dorsalis: Sender signaler fra muskelspoler, senespoler og lavtrskelsmekanoreceptorer.
Tr. spinocerebellaris ventralis: Sender information om aktivitetsniveauet i den gr substans
(specielt interneuroner i lamina 7).

Fibrene fra de to baner ascenderer i funiculus lateralis og ender i spino-cerebellum sv.t. vermis og
paravermale omrder. Neuronerne ender ipsilateralt, da de enten slet ikke krydser eller krydser to
gange.
Den strste indirekte forbindelse afbrydes synaptisk i oliva inferior i medulla oblongata. Neuroner
fra alle niveau sender fibre til oliva i MODSATTE side. Fra oliva fres fibrene til MODSATTE side af
cerebellum. Fibrene krydser alts to gange og pathwayen sender derfor information til den
ipsilaterale del af cerebellum.
9.1.14. Redegr kort for oprindelse, forlb, termination samt funktion af tr. spinocerebellaris
dorsalis og tr. cuneocerebellaris.
Tr. spinocerebellaris dorsalis: Tr. spinocerebellaris dorsalis kommer fra de fibre, der danner
Clarkes sjler (der findes fra L2-T1). Trakten frer proprioceptive signaler fra
underekstremiteten og truncus til cerebellum. Fibrene ascenderer ipsilateralt og ender i
spinocerebellums benregioner via den nedre lilehjernestilk.
Tr. cuneocerebellaris: Sender proprioceptive signaler fra overekstremitet til cerebellum.
Fibrene lber til nc. cuneatus ext. I fasiculus cuneatus. Fibrene herfra lber i tr.
cuneocerebellaris og ender i armregionerne af spinocerebellum, via nedre lillehjernestilk.
Ender ipsilateralt.
9.1.15. Redegr kort for oprindelse, forlb, endeomrder og funktion af tr. spinocerebellaris
ventralis og tr. spinocerebellaris rostralis.
Tr. spinocerebellaris ventralis: Banen starter i lamina 7 i rygmarven. Den sender signaler om
interneuroners aktivitet. De fleste fibre krydser medulla spinalis p segmentrt niveau. De
krydser igen nr de nr cerebellum via vre lillehjernestilk. Banen sender kun signaler fra alt
UNDER midt-thoracalt niveau.
Tr. spinocerebellaris rostralis: Banen starter i lamina 7 i rygmarven. Den sender signaler om
interneuroners aktivitet. Banen sender kun signaler fra alt OVER midt-thoracalt niveau.
9.1.16. Redegr for oprindelse og funktion af tr. spinocerebellaris anterior og posterior.
Tr. spinocerebellaris anterior og posterior svarer til tr. spinocerebellaris ventralis og dorsalis.
9.1.17. Redegr kort for oprindelse, forlb og endeomrder, herunder type af synaptisk
termination, samt funktion af de cortico-ponto-cerebellare baner.
De fleste af banerne fra cortex gr via pons. Banerne har til forml at sende information fra cortex
cerebri til cortex cerebelli.

Banerne starter i SI, MI, PMA, SMA og fra bagerste paritalcortex og fra synsomrderne. Fibrene fra
alle disse omrder lber til kerner i pons via tr. corticopontinus.
Tr. corticopontinus lber i capsula interna og derefter i crus cerebri. Herfra lber banen til pons. Fra
kerner i ponskerner konvergerer fibrene mod cerebellums cortex i hemisfrene, hvorfra der
projiceres til nc. dentatus.
Via de cortico-ponto-cerebellare baner modtager cerebellum modtager information og
bevgelsesplanlgning og om motorkommandoer, som der sendes fra cortex. Som respons herp
Opgaver side 79

bevgelsesplanlgning og om motorkommandoer, som der sendes fra cortex. Som respons herp
kan cerebellum koordinere aktiviteten i motorisk cortex.
9.1.18. Beskriv kort mos- og klatrefibrenes oprindelse, synaptiske kontakter i cerebellum og
virkning p Purkinjecellerne.
Klatrefibrene kommer fra oliva inferior. Mosfibrene kommer fra alle andre steder end oliva inferior
(medulla spinalis, medulla oblongata og pons).
Mosfibrene kontakter kornceller i korncelle-laget med glutamat. Korncellerne sender fibre til det
molekylre lag, der deler sig, og bliver til parallelle fibre. De parallelle fibre kontakter
purkinjecellernes dendritter med glutamat.
Mosfibrene har lille effekt p hvert neuron de kontakter, men til gengld kontakter de mange
neuroner via parallelfibrene! Mosfibrene fr purkinjecellerne til at fyre i simple spikes. Mosfibrene
sender prcis information om planlagt motorik (styrke, hastighed, retning osv.).
Klatrefibre kontakter purkinjecellernes dendritter direkte med glutamat i forholdet n til n. n
purkinjeceller bliver alts kontaktet af n klatrefiber. En klatrefiber har stor effekt p det neuron,
som det kontakter og fr purkinjecellerne til at fyre i complex spikes. Klatrefibrer sender signaler om
fejl i udfrelsen af bevgelser dvs. nr en bevgelse ikke svare til det nskede. De er vigtige for
indlring.
9.1.19. Angiv nc. rubers vigtigste afferente og efferente forbindelser, samt kerneomrdernes
funktion, baseret p disse forbindelser.
Nc. ruber opdeles i en magnocellulr del og en parvocellulr del, der er funktionelt forskellige.
Den magnocellulre del: Modtager afferenter fra fibre fra motorisk cortex og sender
efferenter til medulla spinalis sv.t. tr. rubrospinalis.
Den parvocellulre del: Den parvocellulre del af nc. ruber er indskudt som en
forbindelsessljfle med cerebellum. Den modtager fibre fra nc. dentatus og sender fibre til
oliva inferior, der sender signaler tilbage til cerebellum. Samarbejdet med cerebellum kan via
nc. ruber pvirke de rubrospinale trakter.
9.1.21. Beskriv cerebellums arterielle blodforsyning, inklusive arteriernes oprindelse og
endeomrde.
Cerebellum forsynes fra:
A. cerebelli inferior posterior: PICA forsyner den nedre lillehjernestilk og den inferiore del af
cerebellum, inkl. Vermis. PICA kommer fra a. vertebralis.
A. cerebelli inferior anterior: Forsyner midterste lillehjernestilk, den caudale del af nc.
dentatus. AICA kommer fra a. basilaris.
A. cerebelli superior: Forsyner den superiorer overflade af cerebellum, de fleste cerebellare
kerner og den vre lillehjernestilk. A. cerebelli superior kommer fra a. basilaris.
9.1.22. Angiv det samlede st af symptomer, som en okklusion af a. cerebellaris post. inf. dextra
vil medfre, med samtidig benvnelse af de involverede kerner.

Der er tale om Wallenbergs syndrom:


Faldtendens til hjre side (fordi nc. vestibularis er pvirket).
Dysphagi og hshed (fordi nc. ambiguus er pvirket).
Tab af temperatur- og smertesans ipsilateralt i ansigtet (dvs. hjre) og kontralateralt p
kroppen (dvs. venstre). Skyldes at nc. spinalis n. trigeminus er pvirket og fordi ALS er pvirket.
Ataxi i hjre, idet pedunculus cerebellaris inferior er pvirket.
Horners syndrom (pga. udfald i de sympatiske fibre).
Opgaver side 80

Horners syndrom (pga. udfald i de sympatiske fibre).


9.1.23. Angiv navnene p de dybe cerebellare kerner og de enkelte kernes typiske afferente og
efferente projektionsomrder.
Kernerne er:
Nc. dentatus: Nogle fibre lber fra nc. dentatus til nc. ruber. Andre lber til thalamus, hvor
fibrene kan sendes videre til f.eks. motorcortex.

Nc. globosus: Nc. globosus modtager afferenter fra intermedirzonen (spino-cerebellum) og


sender efferenter fibre til modsatte sides thalamus og til nc. ruber. Derfra kan kernerne
pvirke motorneuroner.
Nc. emboliformis: Nc. emboliformis modtager afferenter fra intermedirzonen (spinocerebellum) og sender efferenter fibre til modsatte sides thalamus og til nc. ruber. Derfra kan
kernerne pvirke motorneuroner. Nc. emboliformis virker alts p samme mde som nc.
globosus.
Nc. fastigii: Sender fibre til nc. vestibularis og formatio reticularis. De kan pvirke
motorneuroner via reticulo- og vestibulospinale baner.
9.1.24. Redegr p baggrund af din viden om cerebellums relationer for symptomerne ved
herniering af de cerebellare tonsiller.
Tonsillerne vil presses ned gennem forarmen magnum, hvilket vil medfre komprimering af
hjernestammen og de cervikale dele af rygmarven. Symptomer:
Bevidsthedsniveauet falder (komprimering af formatio reticularis)
Blodtrykket bliver ustabilt (komprimering af formatio reticularis)
Nakkeledet bliver stift (fordi tonsillerne kommer i klemme i forarmen magnum)
Emisis (da der trykkes p brkcenteret i medulla oblongata)
9.1.25. Redegr kort for begrebet "motorisk adaption".
???
9.1.26. Et barn har en hurtigvoksende tumor i loftet af 4. ventrikel. Redegr for hvilke symptomer,
der forventes at optrde hos dette barn.
En rumopfyldene proces her ville vre en aflbshindring fra fjerde ventrikels CSF. Dette ville
medfre forstrrelse af ventriklerne, et get ICP og flgelig at barnets suturer ville give efter og
kraniet ville udvide sig til f.eks. et trnkranie eller et bdkranie.
9.1.27. Gang-ataksi kan typisk ses ved atrofi af lobus anterior. Anfr, hvorledes du vil pvise
ataksien, redegr for, hvorfor atrofi af lobus anterior forrsager gang-ataksi.
Ved delggelse af lobus anterior ses ndringer i de posturale reflekser og ndringer i det
vestibulre neuroner. Dette vil give gang-ataxi, primrt fordi spino-cerebellum vil pvirkes.
Cerebellum vil alts ikke kunne modtage proprioceptive signaler fra rygmarven og dette vil pvirke
gangen. Test om patienten kan g?

Opgaver side 81

9.2. Basalganglier:
24. februar 2014
22:07

9.2.1. Der nskes en beskrivelse af:


Hvilke strukturer, der indgr i basalganglierne, deres inddeling og deres topografiske relation til
hinanden og til capsula interna:
Basalganglierne bestr af striatum og globus pallidus. Strukturerne kan underinddeles:
Striatum bestr af nc. caudatus og af putamen. Striatum modtager fibre fra cortex.
Globus pallidus inddeles i en globus pallidus interna og en globus pallidus externa.
Funktionelt medregnes substantia nigra og nc. subthalamicus til basalganglierne.
Basalganglierne er placeret dybt i telencephalon og har relation til thalamus og capsula interna.
Nc. caudatus:
Nc. caudatus bestr af en cauda, en corpus og en caput. Nc. caudatus ligger medialt for capsula
interna og medialt for globus pallidus og putamen. Nc. caudatus ligger i hele sit forlb i vggen af
fjerde ventrikel. Udgr sammen med putamen striatum.
Putamen:
Ligger lateralt for globus pallidus. Udgr linsekernen sammen med globus pallidus. Udgr striatum
sammen med nc. caudatus.
Globus pallidus:
Ligger lateralt for capsula interna og nc. caudatus, men medialt for putamen. Inddeles i en ydre og
en indre del.
Substantia nigra:
Opdeles i en pars reticulata og en pars compacta. Pars compacta indeholder de dopaminerge
neuroner. Substantia nigra ligger i mesencephalon, kaudalt for thalamus og nc. subthalamicus.
Nc. subthalamicus:
Ligger i diencephalon mellem thalamus og substantia nigra.
Basalgangliernes afferente, interne og efferente nerveforbindelser, med angivelse af
neurotransmittere, hvor dette er kendt:
Afferente forbindelser lber til striatum. Det strste antal af fibre kommer direkte fra hjernebarken.
Fibre fra associationscortex nr nc. caudatus og fibre fra SI og MI nr putamen. Sammen udgr
fibrene de glutaminerge corticostriatale fibre. Putamen modtager mere r data og nc. caudatus
modtager mere bearbejdet information.
Der er ogs sekundre afferenter:
Fibre fra intralaminre thalamuskerner (via thalamo-striatale baner).
Dopaminerge fibre fra pars compacta af subtantia nigra (via en nigro-striatal baner).
Serotinerge fibre fra raphekerner (raphekerner har homeostatisk kontrol og/eller motorisk
kontrol som funktionsomrde).
De fleste forbindelser fra striatum projicerer til GP og substantia nigra med GABA-neuroner og
neuropeptider. Alts inhibitorisk virkning!

Der er efferente forbindelser til thalamus, PPN og colliculus superior.


Opgaver side 82

Der er efferente forbindelser til thalamus, PPN og colliculus superior.


Basalgangliernes funktion, gerne illustreret ved de symptomer, der opstr ved kendte
basalgangliesygdomme:
Striatiale neuroner sender fibre til GPi og GPe -> to signalveje ud af striatum. Dopaminerg signalering
fra striatum vil excitere neuroner i GPi og hmme neuroner i GPe. Fibre fra GPe gr til nc.
subthalamicus via en indirekte bane. Nc. subthalamicus gr tilbage til GPi og videre til thalamus.
Ved Parkinsons sygdom ses degenering af dopaminerge fibre i substantia nigras pars compacta.
9.2.2. Nvn de vigtigste afferente og efferente forbindelser for de thalamuskerner, der forbinder
sig med basalganglierne.
Flgende forbindelser er afferente (lber til thalamus):

De pallidothalamiske fibre: Der lber GABAerge pallidothalamiske fibre fra globus pallidus
internus (GI(in)) til thalamus (primrt VL og VA).
De nigrothalamiske fibre: Der lber GABAerge nigrothalamiske fibre fra substantia nigra til
thalamus (primrt til de intralaminre kerner og VA).
Flgende forbindelser er efferente (lber fra thalamus):
De thalamocortikale fibre: Der lber glutamerge thalamocortikale fibre fra thalamus til cortex
cerebri (primrt til det prfrontale cortex). Disse hrer strengt talt IKKE til basalganglierne,
men de er vigtige funktionelt.
De thalamostriatale fibre: Der lber glutamerge thalamostriatale fibre fra thalamus til
striatum.

9.2.3. Angiv skematisk det neuronale kredslb fra cortex cerebri gennem basalganglierne til cortex
cerebri, herunder ogs placeringen af to klinisk vigtige forbindelsessljfer involveret i hhv.
Parkinsons sygdom og hemiballismus.

Basalganglierne samarbejder alts med thalamus og er med til at danne to loops fra cortex cerebri
via thalamus og tilbage igen De to loops er hhv. det direkte og det indirekte loop.
Det direkte loop: Cortex cerebri -(glut)-> striatum -(GABA + substans P)-> GP(in) -(GABA)->
thalamus -(glut)-> cortex cerebri.
Det indirekte loop: Cortex cerebri -(glut)-> striatum -(GABA + enkefalin)-> GP(ex) -(GABA)-> nc.
Subthalamicus -(glut)-> GP(in) -(GABA)-> thalamus -(glut)-> cortex cerebri .
Det direkte loop stimulerer alts normalt motorcortex, mens det indirekte loop inhibere
motorcortex.

Bde det direkte og det indirekte loop er involveret i Parkinsons sygdom, der er karakteriseret ved
celledd af dopaminerge neuroner i substantia nigra pars compacta. Det indirekte loop er pvirket
ved hemiballismus, der er karakteriseret ved skader p nc. subthalamicus.
9.2.4. Redegr kort for dopamins betydning for nervetransmission i striatum og for udfrelsen af
viljestyrede bevgelser.
Dopamin kan i striatum binde til D1 og D2-receptorer. D1-receptorer virker excitatorisk og D2receptorer virker inhibitorisk. Det direkte loop udtrykker normalt D1-receptorer og exciteres derved
af dopamin. Det indirekte loop udtrykker normalt D2-receptorer og inhiberes derfor af dopamin.
Under normale omstndigheder, hvor der sendes dopamin til striatum fra substantia nigra,
inhiberes det inhiberende loop alts, mens det exciterende loop exciteres.
Dopamin fremmer alts bevgelse, under normale omstndigheder.
Dopamin kan desuden virke vkstfremmende p neuroner og har en beskyttende virkning.
Opgaver side 83

Dopamin kan desuden virke vkstfremmende p neuroner og har en beskyttende virkning.


9.2.5. For dopamin kendes isr to undertyper af receptorer, som findes i basalganglierne. Redegr
kort for, hvilke celler i basalganglierne, de to receptortyper findes p, og i hvilke neuronale
kredslb i basalganglierne, de hver isr indgr.
Der er to slags receptorer for dopamin i basalganglierne:
D1-receptorer: De virker excitatoriske. Findes isr i det direkte loop i striatum. Findes tt p
neuroner der bruger GABA og substans P som transmitterstoffer.
D2-receptorer: De virker inhibitoriske. Findes isr i det indirekte loop i striatum. Findes tt p
neuroner, der bruger GABA og enkefalin som transmitterstoffer.
Dopamin virker modulerende via metabotrope receptorer. De regulerer sandsynligheden for, at der
frigives GABA og neuropeptider.
9.2.6. Tab af dopaminerge nigrostriatale neuroner medfrer Parkinsons sygdom, som er
karakteriseret ved hypokinesi i form af besvr med initiering af bevgelse og rigiditet. Med
udgangspunkt i dopamins virkning p basalgangliernes interne og eksterne forbindelser nskes en
kort redegrelse for hvordan hypokinesien kan forklares.
Normalt inhiberer dopamin den inhiberende pathway og stimulerer den exciterende pathway. Ved
mangel p dopamin (som f.eks. ved Parkinsons) vil den exciterende stimuleres MINDRE og den
inhiberende bane vil disinhiberes (i.e. den vil GE sin inhibering pga. en hyperaktiv nc.
subthalamicus).
Resultat bliver, at motorcortex stimuleres MINDRE. Dette medfrer hypokinesi.
9.2.7. Der nskes en kort og fokuseret redegrelse for, hvordan den thalamiske aktivering af
motorisk/prmotorisk cortex er nedsat ved Parkinsons sygdom.
Ved Parkinsons sygdom mangler man dopamin i striatum.
Ved mangel p dopamin bliver den inhiberende bane hyperaktiv, hvilket leder til en hyperaktiv nc.
subthalamicus. Den hyperaktive nc. subthalamicus vil gre GP(in) hyperaktiv, hvilket vil gre
thalamus hypoaktiv pga. de GABAerge palladothalamiske neuroner. Dette vil betyde, at frre
excitatoriske signaler sendes fra thalamus til motorcortex.
9.2.8. Redegr kort for nc. subthalamicus' rolle for de motoriske symptomer ved Parkinsons
sygdom og Huntingtons chorea.
Ved Parkinsons bliver nc. subthalamicus hyperaktiv, hvilket vil lede til nedsat excitering af
motorcortex.
Der vil alts ses et NEDSAT excitatorisk output til de motorcortikale omrder.
Ved Huntingtons chorea bliver nc. subthalamicus hypoaktiv, hvilket vil gre thalamus hyperaktiv.
Dette vil lede til et get excitatorisk output til de motorcortikale omrder.
Der vil alts ses GET excitatorisk output til de motorcortikale omrder!
9.2.9. Hvad forsts ved supplementr motorisk cortex og hvad et dets funktion?
Det supplementre motoriske cortex er placeret lige anteriort for MI. Neuroner i SMA sender fibre
direkte til rygmarven og til formatio reticularis.
Funktionen af SMA er formentligt postural stabilisering af, koordinering af begge sider under
bimanuelle bevgelser og koordinering og planlgning af komplekse bevgelser.

9.2.10. Basalgangliernes direkte indirekte pathways:


Opgaver side 84

9.2.10. Basalgangliernes direkte indirekte pathways:


Angiv indholdet af neurotransmittere i de direkte og indirekte baner i basalganglierne samt i de
nigrostriatale og striatonigrale baner:
I den direkte bane er der flgende neurotransmittere:
De corticostriatale fibre: Glutamat.
Fibre fra striatum til GP(in): GABA og substans P.
De palladothalamiske fibre: GABA.
De thalamocortikale fibre: Glutamat.
I den indirekte bane:
De corticostriatale fibre: Glutamat.
Fibre fra striatum til GP(ex): GABA og enkefalin.
Fibre fra GP(ex) til nc. subthalamicus: GABA.
Fibre fra nc. subthalamicus til GP(in): Glutamat.
De palladothalamiske fibre: GABA.
De thalamocortikale fibre: Glutamat.
De andre baner:
De nigrostriatale fibre: Dopamin.
Striatonigrale fibre: GABA.
Redegr med udgangspunkt i direkte og indirekte baner for, hvordan dopaminfrigivelse i striatum
pvirker aktiviteten af de vre motorneuroner i cortex.

Dopaminfristning vil pvirke D1-receptorer i den direkte bane og D2-receptorer i den indirekte
bane.
Nr D1-receptorer stimuleres vil de virke excitatoriske p den direkte bane. Resultatet vil ge
exciteringen af motorcortex.
Nr D2-receptorer stimuleres vil de virke inhiberende p den indirekte bane. Resultatet vil vre
nedsat inhibering af motorcortex.

Opgaver side 85

Kapitel 10: Lugtesansen, smagssansen og hypothalamus:


16. marts 2014
14:17

Opgaver side 86

10.1. Lugtesansen:
16. marts 2014
15:04

10.1.1. Beskriv kort bulbus olfactorius celletyper.


I bulbus olfactorius findes mitralceller, som leder signal fra receptorcellerne til lugtebarken igennem
tr. Olfactorius. Deres transmittersubstans er glutamat. Flere receptorceller lber til en mitralcelle, og
synapsen danner en glumerulus, som anses for en funktionel enhed. En glomerulus modtager
aksoner fra receptorceller med samme specificitet.
Desuden findes et lag med kornceller, som formidler lateral inhibition p samme mde som
horisontalceller i retina. Transmitteren er GABA, som fristtes i dentrodendritiske synapser.
10.1.2. Beskriv kort opbygningen af de olfaktoriske forbindelser fra receptor til hjernebark.
Receptorcellerne er egentlig olfaktoriske epithelialceller, som har dendritter med cilier. CIlierne
ligger i det mukse lag i nsehulen og opfanger odoranter, som er oplst i slimet. Receptorcellerne
har stereospecificitet, hvilket betyder, at molekyler med samme strrelse og form opfanges af
samme type receptor.
Mellem epithelcellerne er der indskudt basalceller, som srger for fornyelse af receptorcellerne.
Desuden findes sttteceller som srger for isolation af aktionspotentialer.
De olfaktoriske celler er bipolre og sender deres aksoner op gennem lamina cribosa i kraniet, som
fila olfactoria. Fila olfaktoria tilsammen danner n. olfactorius.
I bulbus olfaktorius danner receptorcellerne synapse med mitralcellerne i funktionelle glomeruli.
Mitralcellernes axoner danner tr. Olfactorius, som sender signal til cortex og amygdala. I cortex er
det specielt area entorhinalis og uncus, som tilsammen danner den primre lugtebark.
Fra den primre lugtebark og amygdala sendes direkte fibre til hypothalamus, orbitofrontal cortex
og modsatte sides lugte bark (via commisura anterior). Fra orbitofrontal cortex sendes desuden
indirekte fibre til hypothalamus, som frer integreret information.
10.1.3. Angiv den cortikale reprsentation af lugtesansen.
Lugtesansen er reprsenteret p den mediale side af temporallappen. Mere specifikt er det area
entorhinalis og uncus.
10.1.4. Redegr kort for den biologiske konsekvens af lugesansens centrale forbindelse til
amygdala.
Olfactoriske celler til hypothalamus via amygdala, er vigtige for nringsoptag og seksualreflekser.
Amygdalas funktion er netop at vurdere respons p stimuli. Hvis vi lugter til noget mad, der er for
gammelt, vil det associeres med tidligere oplevelse af noget vmmeligt, og derfor vil man undg at
spise maden. Mht. seksualrefelkser er feromoner af betydning, da de fremkalder visse associationer,
nr bestemte odoranter udskilles fra det modsatte kn. Det vil medfre tiltrkning eller afstdning
afhngigt af associationerne.
10.1.5. I hvilke sygdomme er lugtesansen strkt nedsat.
Traumer i fossa cranii anterior, som kan ldere tr. Olfactorius.
Forklelse, hvor nsen er tilstoppet.
Udsat for pesticider og oplsningsmidler, som pvirker slimhinden i nsen.

Opgaver side 87

10.2. Smagssansen:
16. marts 2014
15:03

10.2.1. Redegr kort for smagssansens forbindelser fra de perifert beliggende receptorceller til
den cerebrale cortex.
Smagsimpulser kommer fra kemoreceptorer, der findes i epithelet p tungen. Mere specifikt findes
sansecellerne i smakspapiller, hvoraf der findes tre typer: Papilla fungiformes, Papilla valatae og
Papilla foliata. Papilla valatae er den strste. I furen omkring smagspapillen oplses smagsstoffer og
endeforgreninger er nerver sidder basalt ved sansecellen og registrere disse stoffer.
Den bagerste 1/3 af tungen er innerveret af n. glossopharyngeus, mens forreste 2/3 innerveres af n.
facialis (n. lingulis som gr over i chorda tympani og videre gennem facilis). De afferente fibre samler
sig i nucleus solitarius.

Fra nucleus solitarius i medulla oblongata lber fibre til VPM i thalamus, hvor den danner synapse.
Herfra lber der fibre til somatosensorisk cortex (SI) i det omrde, som reprsenterer ansigtet, samt
til insula. Tilsammen udgr disse to omrder den primre smagsbark.
10.2.2. Angiv de efferente forbindelser fra CNS, der indgr i en reflekstorisk, smagsudslst
spytsekretion.
Fibre fra nucleus solitarius gr til nucleus salivatorius superior, hvor den stter gang i spytsekretion i
mundhules spytkirtker via. n. intermedius (n. facialis). Desuden sendes fibre til nucleus salivatorius
inferior, som srger fr spytsekretion i gl. parotis via. n. glossopharyngeus. Dvs. at ved indtag af mad,
vil der reflekstorisk sttes gang i spytsekretion, som hjlper med fordjelsen.
Fra nucleus solitarius sendes ogs fibre til den motoriske vaguskerne, som indeholder viceral
efferente fibre. Ved madindtag sttes reflektorisk gang i sekretion af mavesaft i ventriklen.
10.2.3. Hvilke grundtyper af smagskvalitetens findes, og hvordan mener man, at de tilknyttede
receptorceller aktiveres
Der er fem forskellige smagsindtryk, som smagscellerne kan opfatte: surt, sdt, umani, bittert og
salt.
Surt og salt registres via ionkalnaler. Surt: Nr H+ koncentrationen rundt om cellen stiger, lukkes K+
kanaler, hvorved cellen depolariseres. Salt: Nr saltkoncentrationen stiger rundt om smagscellen, vil
Na+ g ind i cellen via passive ionkanaler, hvorved den ogs depolariseres.
Sd og umani registreres af G-protein-koblede receptorer, mere specifikt T1R.
Bittert registres af ligeledes af G-protein-koblede receptorer, mere specifikt T2R

Opgaver side 88

10.3. Hypothalamus:
16. marts 2014
15:03

10.3.1. Angiv p oversigtsform hypothalamus' opdeling i kerneomrder.


Overordnet kan hypothalamus inddeles i en medial del og en lateral del.
Den mediale del inddeles i en forreste (anterior), midtre (tuberal) og bagerste (mamillar)
kernegruppe.

Den forreste kernegruppe bestr af nucleus suprachiasmticus (ligger over chiasma opticus) og
nucleus paraventricularis (ligger i vggen af tredje ventrikle). Desuden bestr den af nucleus
supraopticus.
Den midterste kernegruppe bestr af nucleus ventromedialis og nucleus arcuatus.
Den bagerste kernegruppe bestr af nucelus posterior og nucleus mammilare
Den laterale del er diffus, men mange forbindelser gr igennem.
En anden mde at inddele p er zoner i hhv. ant-post. Retning og i med-lat retning.
I ant-post retning er der 4 zoner: en preoptisk, en supraoptisk, en tuberal og en mamilar zone.
I med-lat retning er der 3 zoner: en periventrikulr zone, en medial og en lateral

Opgaver side 89

Kapitel 11: Det limbiske system:


16. marts 2014
15:17

Opgaver side 90

11.0. Det limbiske system:


16. marts 2014
15:18

11.0.1. Angiv de strukturer, der typisk indgr i det limbiske system.


Amygdala
Gyrus cingulli
Hippocampusformationen
Septumkernerne
11.0.2. Angiv mindst tre overordnede funktioner af det limbiske system med samtidig angivelse af
de tilknyttede primre hjerneomrder.
Amygdala: Emotionel indlring (dvs. kobling af et stimuli til en emotionel vrdi).
Hippocampusformationen: Medvirker i dannelsen af den deklarative hukommelse.
Olfaktorisk cortex: Knyttet til overlevelses- og reproduktionsmekanismer.
11.0.3. Hvordan indgr corpora mamillaria forbindelsesmssigt i det limbiske system.
Corpus mamilaria modtager fibre fra hippocampusformationen og sender fibre til gyrus cinguli via
thalamus (tr. mamilothalamicus). Fra gyrus cinguli lber fibrene tilbage til hippocampusformationen.
Hippocampusformationen -> corpus mamilaria -> gyrus cinguli (via thalamus) ->
hippocampusformationen kaldes for Papez loop.
Corpus mamillaria er et vigtigt bindeled i overfrselen af signaler fra temporallappen til gyrus cinguli.
11.0.4. Angiv amygdalas vigtigste afferente og efferente forbindelser, samt de funktioner, der er
tilknyttet.
De corticomediale kerner (smerte) modtager afferenter fra:
Uspecifikke thalamuskerner
Lugtelappen
Hypothalamus
Septumkerner
Parabrachiale omrder
De basolaterale kerner (alle slags sanseindtryk) modtager afferenter fra:
Prfrontalt cortex
Temporale associationsomrder
Gyrus cinguli
Nogle thalamuskerner
Hippocampus
11.0.5. Giv tre eksempler p adfrdsndringer/neurologiske symptomer efter skade eller
stimulation af amygdala:
Der vil evt. ses:
Ingen frygtbetinget adfrd.
Social isolation.
Visuel agnosi isr kan man ikke genkende ansigtsudtryk.
Kan ikke skelne mellem farligt og ufarligt.
En skade p amygdala afbryder koblingen mellem emotioner og stimuli, og dermed ogs de
tilhrende relevante reaktioner p stimuli.

Opgaver side 91

Ved stimulering af den corticomediale del af amygdala vil der ses slikke og tyggebevgelser og
smasken.
Ved stimulering af de basolaterale del af amygdala vil der ses opmrksomhedsreaktion.

Opgaver side 92

11.1. Hukommelse:
16. marts 2014
15:18

11.1.1. Redegr kort for hippocampus' relation til hukommelse og indlring.


Hippocampusformationen udgres af:
Gyrus dentatus
Hippocapus
Subiculum
Area entorhinale
Selve hippocampus er inddelt i:
CA1
CA2
CA3
Der er en hovedvej for signaltrafik i hippocampus. Vejen gr fra:
Area entorhinales -> gyrus dentatus -> CA3 -> CA1 -> Subiculum -> Area entorhinales
Alle led er excitatoriske.
Korttidshukommelsen menes at involvere hippocampusformationen. Formentligt lagres information
i hippocampus i det frste stykke tid, inden minder distribueres ud til cortex. Det er kun deklarativ
hukommelse, der er hippocampusafhngig.
11.1.2. Redegr kort for, hvad man forstr ved "long term potentiation" (LTP).
LTP og LTD er former for synaptisk plasticitet.

LTP kan defineres som aktivitetsafhngig gning af synaptisk effektivitet, som der varer mere end
en time og som skyldes ndringer i synapsens struktur. LTP kan induceres af strk prsynaptisk
aktivitet eller af forskellige prsynaptiske pvirkninger, der falder samtidigt. Disse stimuli kan
betyde ndringer i mngden af transmittersubstans der frigives og/eller at den postsynaptiske celle
ndrer sit antal af receptorer eller deres flsomhed for neurotransmitterer.
Srlig kraftig og langvarig LTP kan fremkaldes ved en serie af prsynaptiske aktionspotentialer med
maksimal frekvens (en tetanisk stimulering). Dette fnomen kaldes for posttetanisk potensering og
kan bevirke get transmitterfristning, get receptorflsom osv. I dette tilflde vil den
prsynaptiske aktionsterminal huske, at den blev specielt strkt pvirket for en tid siden og ndre
sin adfrd p baggrund af dette. LTP reprsenterer lagring af information, der er vigtig for individet
eller som opfattes som vigtig.

Mange steder er LTP forbundet med aktivering af NMDA-receptorer. Disse receptorer virker ved
bning af calciumkanaler.
11.1.3. Hvilken funktioner tillgges LTP i bl.a. hippocampus-omrdet af hjernen.
LTP produceres, nr hippocampus pyramideceller stimuleres med excitatoriske input (f.eks. fra
Schaeffer-kollateraler (fibre der lber mellem CA3 og CA1)), mens de er exciterede i forvejen.
Det vil sige, at simultan synaptisk aktivering af cellen fra to forskellige kilder kan f den til at huske,
at den skal fyre mere, nste gang den stimuleres p samme mde. Meget tyder p, at LTP i
hippocampus er involveret i lring og hukommelse.
NMDA-receptoren menes at vre involveret i induktionen af LTP.
11.1.4. Hvorfor menes LTP at vre det synaptiske grundlag for lring og hukommelse?

Opgaver side 93

Fordi en NMDA-receptor-antagonist hindrer induktionen af LTP. Dette har vist sig at hindre
forsgsdyrs evne til at lre og huske.
11.1.5. Angiv den type (kvalitet) af lring og hukommelse, der er knyttet til hhv. amygdala og
hippocampus og pvirkes af lsioner af disse strukturer.
Amygdala er med til at farve minder emotionelt f.eks. hjlper den os med at huske, hvilken stimuli
der er gode for os og hvilken der er farlige. Ved lsioner ses ndret adfrd ift. til frygt, f.eks.
Desuden manglende evne til at genkende ansigtsmimik.
Hippocampus er involveret i korttidshukommelsen og i overfrelsen af information fra
kortidshukommelsen til langtidshukommelsen. Desuden mener man at hippocampus hjlper os
med rummelig orientering ved at lave et kort over vores omgivelser. Kortet hjlper os med at
huske, hvor vi har forskellige objekter ift. til os. Ved lsion i hippocampus ses som oftest anterograd
amnsi nogle gange med tilhrende retrograd amnesi.
11.1.6. Hvad forsts ved langtids- og korttidshukommelse, og hvilke strukturer menes at vre
involverede?
Korttidshukommelsen hrer til i hippocampus. Minder lagres kortvarigt i hippocampus, hvor de
konsolideres. Herfra kan de flyttes til cortex, hvor de etableres permanent i langtidshukommelsen i
forskellige distributive netvrk der er ikke noget egentligt hukommelsescenter i cortex (s vidt
vides).

Opgaver side 94

Kapitel 12: Cortex cerebri:


22. marts 2014
10:15

Opgaver side 95

12.1. Det cerebrale cortex:


22. marts 2014
10:18

12.1.1. Beskriv kort hjernebarkens laminre opbygning og forskellene i dens opbygning i gyrus
pr- og postcentralis.
Lagene i cortex er som flger:
1. Lamina 1: Lamina molecularis. Her er mange fibre, men f neuroner. Bare en masse
dendritter!
2. Lamina 2: Lamina granularis externa. Det indeholder tt pakkede, sm cellekroppe.
MODTAGER primrt associationsfibre.
3. Lamina 3: Lamina pyramidalis externa. Her findes mellemstore pyramideceller. Lamina 3
sender primrt fibre til andre steder af cortex (associationsfibre og kommisurale fibre). Dvs.
SENDER primrt associationsfibre.
4. Lamina 4: Lamina granularis interna. Lamina 4 MODTAGER affernenter fra thalamus. Lamina 4
er isr stort i de primre somatosensoriske omrder (gyrus postcentralis) og i area striatae.
5. Lamina 5: Lamina pyramidalis interna. Her findes store pyramideceller (Betz-celler).
Pyramidecellerne i lamina fem SENDER deres axoner primrt til kerner uden for cortex.
Lamina fem er srligt veludviklet i de motoriske omrder (gyrus prcentralis).
6. Lamina 6: Lamina multiformis. Her findes celle med ten-formede kroppe. Lamina 6 er primrt
EFFERENT og SENDER axoner til thalamus.
Lamina 2 og 4 er primrt modtagende. De er mest udviklede i
omrder (dvs. gyrus postcentralis, area striatae osv).
Lamina 3 og 5 er primrt efferente. De sender deres axoner ud af den del af cortex, hvor de findes.
Lamina 5 er isr veludviklet i gyrus prcentralis.
I sjlerne i cortex ligger neuroner, der har samme modaliteter og receptive omrder eller som gr
samme steder hen. Forskellige sjler kan kommunikere med hinanden via horisontale fibre og celler i
n sjle kan kommunikere med hinanden via vertikale fibre.
12.1.2. Beskriv kort den lysmikroskopisk synlige, histologiske opbygning af hjernebarken svarende
til primr synscortex.
Der vil vre de seks lag, der nvnes ovenfor. Desuden vil der vre et MEGET veludviklet lamina 4
(lamina granularis interna).

12.1.3. Nervebanerne i centralnervesystemets hvide substans kan principielt inddeles i 3 typer.


Benvn disse typer og angiv to nervebaner for hver type.
Subcorticale forbindelser: Forbindelser mellem cortex og andre dele af CNS. Kunne f.eks.
pyramidebanen, ALS eller BLM.
Corticocortikale forbindelser: Forbindelser mellem forskellige steder i cortex. Kunne vre f.eks.
associationsfibre til paritallappen (area 5 og 7) fra area striatae eller fra S1 til area 5.
Commisuralforbindelser: Forbindelser mellem de to hjernehalvdele. De fleste forbindelser
lber i corpus callosum, men der lber ogs nogle i comissura anterior (mest lugt og smerte).
12.1.4. Beskriv kort opbygning og funktion af corpus callosum og angiv to vigtige omrder af
hjernebarken, der ikke forbindes med callosale fibre.
Commisura anterior er et bundt af flade fibre, der lber under cortex. De forbinder de to hemisfrer
og faciliteter samarbejde mellem dem. Corpus callosum bestr af:
En forreste del (genu): Overfrer sensorisk information.
En midterste del (truncus): Overfrer sensorisk information.
En bagerste del (splenium): Overfrer synsinformation.
Opgaver side 96

En bagerste del (splenium): Overfrer synsinformation.


De motoriske omrder sv.t. hnderne er ikke forbundet med commisurale fibre. Hnderne har brug
for stor selvstndighed. Denne selvstndighed ville blive forstyret af commisurale fibre.
12.1.5. Hvad forsts ved kortikale sjler ("cortical columns")? I hvilke omrder er de isr
undersgt?
I lagene/laminae er neuronerne arrangeret i sjler - dvs. at sjlerne str vinkelret p overfladen af
cortex. I sjlerne i cortex ligger neuroner, der har samme modaliteter og receptive omrder eller
som gr samme steder hen dvs. fibre der p en eller anden mde har relation til hinanden.
Sjlerne kan kommunikere via interneuroner. Horisontale fibre forbinder forskellige sjler med
hinanden. Vertikale fibre forbinder forskellige lamina med hinanden.
Sjler er isr undersgt i somatosensorisk cortex.
12.1.6. Angiv hvad man udviklingsmssigt forstr ved sensitive (kritiske) perioder og redegr kort
for deres neurobiologiske grundlag.
En kritisk periode er en fase i udviklingen, hvor en organisme er srligt sensitiv overfor
udefrakommende stimuli. Hvis organismen IKKE modtager korrekte stimuli under den kritiske
periode kan det vre svrt/umuligt at udvikle visse funktioner senere i livet (f.eks. synet).

Hvis en speciel frdighed ikke lres i den kritiske periode vil de cortikale omrder, der normalt skal
bruges til denne frdighed, ikke udvikles rigtigt. Disse omrder vil s senere overtage andre
funktioner og kan sledes ikke udfylde sin oprindelige rolle efterflgende. Der er alts konkurrence
om de cortikale omrder.
12.1.7. Redegr kort for den topografiske reprsentation af vre motorneuroner i hjernebarken,
herunder forskellene i den areal-mssige strrelse af de forskellige krops-og legemsdeles
reprsentation.
Motorisk ligger i area 4 i gyrus prcentralis. Den stttes af prmotorisk cortex og
supplementrmotorisk cortex.
Det motoriske cortex er somatotopisk opdelt, s efferenter til benet ligger mest medialt og
efferenter til tunge og svlg ligger mest lateralt. Omrder som krver strst prcision fylder mest i
cortex, f.eks. fylder efferenter til hnden, foden og ansigtet en meget stor del af motorisk cortex
sammenlignet med benet og ryggen.
12.1.8. Angiv placeringen af supplementr motorisk cortex og omrdets funktion.
Supplementrmotorisk omrde (SMA) ligger foran motorisk cortex (MI) i area 6.
SMA modtager afferenter fra prfrontale associationsomrder, og sender frst og fremmest
efferenter til MI, men ogs til medulla spinalis og retikulrsubstansen.
Funktionen af SMA er organisering og planlgning af lidt mere komplekse bevgelser og bidrager
derved til motorik. Skader p SMA giver nemlig problemer med koordineret brug af begge hnder.
Desuden er SMA med til at formidle hensigtsmssige bevgelser som respons p sensorisk stimuli.
12.1.9. Redegr kort for den topografiske reprsentation af lemniscus
medialis/bagstrengsbanerne og lemniscus trigeminalis i hjernebarken, herunder for forskellene i
den areal-mssige strrelse af de forskellige kropsdeles reprsentation.
Lemniscus medialis og bagstrengsbanerne lber til somatosensorisk cortex i area 1,2 og 3 (gyrus
postcentralis).
Det er isr hnd og ansigt, som fylder meget arealmssigt, men foden har ogs et stort omrde
(prcis som i motorisk cortex).
12.1.10. Redegr kort for rsagen til, at en person med en lsion af f. eks. somatosensorisk cortex
Opgaver side 97

12.1.10. Redegr kort for rsagen til, at en person med en lsion af f. eks. somatosensorisk cortex
svarende til benet p modsatte side stadig kan fle smerte fra dette ben, samtidig med at der er
fuldstndigt tab af 2-punktsdiskrimination, vibrationssans og stillings- og bevgesans fra benet.
2-punktsdiskrimination, vibrationssans og stillings- og bevgesans fra benet fres i BLM til
somatosensorisk cortex (S I og SII). Hvis SSI og SII er skadet vil disse sanseindtryk vre fravrende.
Smerte fres ogs til SI og SII via tractus spinothalamicus (ALS), men desuden fres smerte ogs til
andre omrder som PAG og retikulrsubstansen. Derfor kan man stadig fle smerte p trods af
lsion af SI og SII. Dog vil lokalisationen af smerten vre fravrende
12.1.11. Begrebet med simple, komplekse og hyperkomplekse neuroner eller grupper af disse
(moduler) er specielt kendt fra synscortex, men noget tilsvarende findes ogs i den
somatosensoriske cortex. Giv et eksempel herp.
Simple celler opfanger en stimulus f.eks. tryk, berring, vibration osv. Disse informationer samles i
komplekse celler. Hyperkomplekse celler integrerer s informationer fra flere komplekse celler,
sledes man bliver i stand til at tolke de forskellige stimuli til en bevgelse eller erkende en form
mv.
12.1.12. Den del af hjernens parietallap, der ligger lige bag det primre somatosensoriske omrde
har en rkke integrative funktioner. Giv 3 eksempler herp (gerne med angivelse af de kliniske
udfaldssymptomer efter lsion).

Der er tale om bagre paritalcortex (area 5 og 7).


Area 5: Direkte sensorisk information fra S1. Sender efferenter til SMA.
Area 7: Modtager information fra area 5 og ogs fra det ekstrastriatale cortex, fra det
auditative cortex og det polymodale omrde. Modtager ogs afferenter fra det prfrontale
cortex og fra det prmotoriske omrde. Area 7 sender efferenter tilbage til de omrder, som
den modtager fra.
12.1.13. Hvilke funktioner tilskrives den verste posteriore del parietale cortex (Brodmanns areae
5 og 7)?
Se ovenfor.
12.1.15. Angiv de typiske funktioner af den prfrontale hjernebark.
Den prfrontale hjernebark (svt. den anteriore del af lobus frontalis, dvs. foran area 6 og area 8) har
med hjere kognitive funktioner at gre. Alle slags sansemodaliteter (samt information om
emotioner) nr det prfrontale cortex.
Frontallappen planlgger og igangstter mlrettet adfrd. Desuden hmmer den upassende
adfrd. Hjlper os til at rette opmrksomheden mod det vi nsker.
12.1.16. Redegr for begrebet lateralisation i forhold til de cerebrale hemisfrer, og angiv tre
eksempler herp baseret p normale funktioner eller "lateraliserede"typer af udfald ved cerebral
skade.
Lateralisation er det samme som hemisfrisk dominans dvs. at n hemisfrer dominerer over den
anden med hensyn til specielle funktioner. Eksempler:
Venstre cortex dominerer som oftest ift. sprog.
Hndmotorik kan vre lateraliseret (ift. f.eks. hjrehndet-hed (venstre cortex) vs.
Venstrehndet-hed (hjre cortex)).
Hjre hjernehalvdel er dominerende ift. flelser.
12.1.17. Hvor bearbejdes synsindtryk?
Det primre synsomrde er V1 i area striatae (area 17 i occipitallappen). Det visuelle cortex
omfatter ogs area 18 og 19.
Opgaver side 98

omfatter ogs area 18 og 19.


Information fra det visuelle cortex sendes til det bagre paritalcortex (area 7), hvor informationen
integreres. Der sendes ogs information til de visuelle associationsomrder i det inferotemporale
cortex.
12.1.18. Synsindtryk i hjre og venstre del af synsfeltet og somatosensoniske sanseindtryk fra
hjre og venstre del af kroppen gr til henholdsvis venstre og hjre hjernehalvdel, men nr en
genstand passerer fra den ene del af synsfeltet eller den ene korpsdel til den anden, oplever vi det
som et kontinuum uden "spring". Hvad sker der?
De kommisurale forbindelser tillader de to hjernehalvdele at kommunikere med hinanden og
samarbejde, sdan s vi ikke oplever sanseindtrykkene som bilaterale.
12.1.19. Angiv p en skematisk tegning den typiske placering af a) Broccas omrde, b) Wernickes
omrde og c) Brodmanns areae 5 og 7, og de symptomer, som en lsion af hvert af disse omrder
typisk vil medfre.

Brocas omrde: Ligger i area 44 og 45 i frontallappen (tt p temporallappen), svt. gyrus triangularis
og gyrus opercularis. Lsion vil ofte medfre ekspressiv afasi.
Wernickes omrde: Ligger i area 39 og 40 i temporallappen, svt. gyrus angularis, gyrus
supramarginalis og/eller gyrus temporalis (area 22). Lsion vil medfre impressiv afasi.
Brodmanns area 5 og 7: Bagre paritallap / det paritale associationsomrde, svt. lobulus paritalis
superior. Lsion vil medfre neglect, apraksi, agnosi.
12.1.20. Hvis dyslexi (ordblindhed) skyldes en forkert kobling af forbindelser mellem nerveceller
eller en lokal ubalance i transmitterstoffer og receptorer, hvor i hjernen ville du s lede efter
denne ndring? Giv en kort begrundelse.
Wernickes omrde som ligger i area 39 og area 40, nrmere bestemt gyrus angularis og gyrus
supramarginalis.
Skader p Wernickes omrde medfrer impressiv afassi, herunder aleksi.
12.1.21. Gr kort rede for begreberne sensorisk (impressiv) afasi og motorisk (expressiv) afasi og
den typiske placering af en beskadigelse ved hver af de to afasi-former.
Personer med impressiv afasi har problemer med at forst hvad der bliver sagt. Dette problem
skyldes ikke at sensoriske input mangler eller er ndrede, men at personen ikke er i stand til at f en
mening ud af det de sanser. Samtidig vil talen vre pvirket. De kan godt tale flydende, men de ord
de stter sammen, kan vre ren volapyk. Hvis en person har impressiv afasi, vil man lede efter en
beskadigelse i Wernickes omrde
Personer med ekspressiv afasi har problemer med at udtrykke sig. Deres tale er haltende og
grammatisk forkert. De kan dog godt forst, hvad der bliver sagt til dem og de kan ofte ogs opfatte,
at de taler forkert. Man vil lede efter skade p Broccas omrde.
12.1.22. Angiv navn og beliggenhed af hjernens sprogcentre hos en normal hjrehndet person, og
beskriv kort den funktionelle forskel p disse, gerne eksemplificeret ved de symptomer, der opstr
efter beskadigelse.
Se de tidligere sprgsml.
12.1.23. Angiv beliggenheden af Broccas talecenter og anfr den naturlige variation, der er
beliggenheden, samt hvilken strre arterie forsyner Broccas omrde?
Broccas omrde ligger i area 44 og 45 gyrus triangularis og gyrus obercularis.
Hos de fleste mennesker er Broccas omrde mest udviklet i venstre hemisfre (glder for 90 % af
hjrehndede og 70 % af venstrehndede).
Opgaver side 99

hjrehndede og 70 % af venstrehndede).
A. cerebri media forsyner omrdet.
12.1.24. Stereognosi er evnen til med f.eks. hnden at erkende genstandes 3-dimensionale
struktur. Angiv det neuronale grundlag for den stereognostiske sans og den typiske lokalisation af
en lsion, der medfrer astereognosi.
Den stereognostiske sans bygger p en intakt BLM, som sender sensoriske input til SI og SII. Herfra
sendes signaler til area 5 og 7, hvor man kan associere det med ting man kender.
En skade p bagerste paritalkorteks vil fre til astereognosi, da area 5 og 7 delgges.
12.1.25. Hvad forsts ved agnosi og apraxi, og hvorved adskiller disse tilstande sig fra alm.
sensorisk udfald og alm. lammelse?

Selve sansningen og motorikken er intakt, s personen har ikke problemer med at opfatte eller
udfre fysisk set. I stedet er der problemer med at integrere den information man modtager.
Agnosi: manglende evne til at genkende objekter (f.eks. personer, ting, farver, lyd mm)
Apraksi: Man har problemer med at udfrelse af en kommando (man fr en genstand i hnden men
ved ikke hvad man skal stille op med den, fordi man mangler en association mellem genstand og
kommando)
12.1.26. Betegnelsen apraxi, anvendes nr f.eks. evnen til at klde sig p eller bruge en hammer
er mere nedsat end svarende til en eventuel lammelse af de relevante muskler. Angiv den typiske
lokalisation af lsioner, der giver apraxi, og nvn de typiske afferente og efferente forbindelser
for omrdet.
Skader p bagerste paritalkorteks giver apraksi, da associationsomrder (area 5 og 7) bliver skadet.
Area 5 modtager afferenter fra SI og SII.
Area 7 modtager afferenter fra area 5, visuel cortex, auditiv associationscortex og prfrontal cortex.

12.1.27. Begrebet agnosi dkker over symptomer, hvor en person nok kan registrere primre
sanseindtryk, men ikke evner at tolke disse (nedsat eller manglende perceptionsevne). Nvn tre
eksempler herp og angiv den typiske placering af lsionerne for hver af dem.
Tre former:
Visuel agnosi: Kan ikke integrere visuel information. Skader p ekstrastriatale omrder (area
18 og 19).
Taktil agnosi: Kan ikke integrere berringensinformation. Skader p overgangen mellem
parital- og temporallappen.
Akustisk agnosi: Kan ikke integrere lydinformation. Skader p forbindelsen mellem auditive cortex og
bagre paritale cortex (area 5 og 7).

Opgaver side 100

12.2. Thalamus:
22. marts 2014
10:18

12.2.1. Nvn de sensoriske thalamuskerner, og angiv deres cortikale projektioner (forlb og


endeomrder):
Dorsale kernegrupper: Sender fibre til bagre paritalcortex (area 5 og 7).
Ventrale kernegrupper: Sender fibre til somatosensoriske omrder og motoriske omrder.
Pulvinar (m. corpus geniculatum lateralis og medialis): Pulvinar tager temporale
associationscortex. CGL projicerer til area striata (area 17) og CGM projicerer til auditivt
cortex.
Nc. medialis dorsalis: Sender fibre til prfrontalt cortex.
Nc. anterior: Sender fibre til gyrus cinguli.
12.2.2. De intralaminre thalamuskerner optrder i flere forbindelsesmssige og funktionelle
sammenhnge. Nvn to af disse kerner og angiv den forbindelsesmssige sammenhng, som de
indgr i.
Nc. centromedianum: Sender fibre til striatum. Nc. centromedianum integrerer mange typer af
sanseinformation og ger sin aktivitet ved overgangen fra rolighed vgenhed til
opmrksomhedskrvende opgaver.
Nc. centralis lateralis: Sender fibre til paritalcortex.
12.2.3. Angiv hhv. en afferent og en efferent forbindelse fra nc. anterius thalami.
Nc. anterius thalami modtager fibre fra hippocampus-regionen og sender fibre til gyrus cinguli.
12.2.4. Angiv med navn de thalamuskerner, der:
a. Indgr i forbindelsesmssige kredslb i hjernen af betydning for regulering af motorik.
Nc. centromedianum, der sender fibre til basalganglier.
b. Projicerer til de primre sensoriske omrder af cortex cerebri, og angiv disse omrders
njagtige placering.
Den ventrale del af den laterale kernegruppe (VPL og VPM). VPL sender fibre til den del af
somatosensorisk cortex, der svarer til kroppen og VPM sender fibre til den del af cortex, der
svarer til hovedet.
Indgr i forbindelsesmssige kredslb med det limbiske system og hypothalamus:
Nc. anterius thalami. Muligvis nc. medialis dorsalis?

Opgaver side 101

12.3. Capsula interna:


22. marts 2014
10:19

12.3.1. Beskriv kort capsula internas opbygning og funktion.


Capsula interna er en stor plade af hvid substans, der forbinder cortex med subcortikale omrder.
Der er tre dele:
Crus anterior: Frontopontine fibre, reticulocorticale fibre.
Genu: Corticobulbre fibre.
Crus posterior: Corticospinale fibre, fibre fra BLM og ALS. Opdeles i tre:
o Pars sublenticulare: Kun radiate acustica.
o Pars. Retrolenticulare: Temperopontine fibre, radiatio optica.
o Pars lenticulothalamicus: Pyramidebanen, BLM og ALS. Desuden corticopontine fibre.
Capsula interna forsynes af aa. lenticulostriatale.
12.3.2. Beskriv capsulas internas beliggenhed og opdeling ift. omkringliggende strukturer.
Capsula internas crus anterior ligger mellem nc. caudatus (medialt) og linsekernen (lateralt).
Capsula internas genu ligger mellem crus anterior og crus posterior.
Capsula internas crus posterior ligger mellem linsekernen (lateralt) og thalamus (medialt).
12.3.3. Beskriv nervebaner i capsula internas forskellige dele, idet banerne opdeles i
descenderende og ascenderende, og med angivelse af funktion.
Crus anterior: Frontopontine fibre (descenderer), reticulocorticale fibre (ascenderer).
Genu: Corticobulbre fibre (descenderer).
Crus posterior: Corticospinale fibre, fibre fra BLM og ALS. Opdeles i tre:
o Pars sublenticulare: Kun radiate acustica (ascenderer).
o Pars retrolenticulare: Temperopontine fibre (descenderer), radiatio optica (ascenderer).
o Pars lenticulothalamicus: Pyramidebanen (descenderer), BLM og ALS (ascenderer).
Desuden corticopontine fibre (descenderer).
12.3.4. Capsula internas crus posterius kan opdeles i tre dele, en del placeret mellem thalamus og
nc. lentiformis (pars lenticullothalamica), en del placeret bag nc. lentiformis (pars retolenticularis)
og en del, der passerer under nc. lentiformis (pars sublenticularis. Angiv hvilken nervebaner, der
forlber i hver af de tre dele, med samtidig angivelse af nervebanernes oprindelse, endeomrder
og funktioner:
Pars sublenticulare: Kun radiate acustica.
Pars retrolenticulare: Temperopontine fibre, radiatio optica.
Pars lenticulothalamicus: Pyramidebanen, BLM og ALS. Desuden corticopontine fibre (der lber
videre til lillehjernen).

Opgaver side 102

Kapitel 13: Formatio reticularis


22. marts 2014
10:16

13.0.1. Redegr kort for beliggenhed, generelle opbygning og de vsentligste funktioner af


formatio reticularis.
Formatio reticularis strkker sig fra den caudale ende af medulla oblongata til den vre del af
mesencephalon. Formatio reticularis udfylder de omrder, hvor der ikke ligger hjernenervekerner
eller andre distinkte kerner eller baner.
De retikulre neuroner har typisk meget lange og lige dendritter, s de kan pvirke meget vv.
Formatio reticularis underopdeles p baggrund af cellernes strrelse og form. De mediale 2/3 af
formatio reticularis bestr af store celler (kmpeceller). De laterale 1/3 bestr af mindre celler. Den
mediale del af formatio reticularis er efferent og den laterale del er afferent.
Formatio reticularis er vigtig for:
Homeostatisk kontrol af f.eks. vejrtrkning, hjerterytme og blodtryk
Justerer bevidsthedsniveau
Pvirker orientering af kroppen og opretholdelsen af de posturale reflekser.
Pvirker grovere bevgelser.
Spiller en rolle i smertemodulering.
13.0.2. Hvilken virkning har det retikulre system p den elektriske aktivitet i cortex cerebri.

Stimulering fra formatio reticularis pvirker bevidsthedsniveauet i cortex cerebri, hvilket igen
pvirker cortex elektriske aktivtet.
13.0.3. Afbrydelse af de ascenderende forbindelser fra formatio reticularis har vsentlig
betydning for funktionen af cortex cerebri. Hvorfor?
Den elektriske stimulering af cortex fra formatio reticularis vil frafalde, hvilket vil medfre et snket
bevidsthedsniveau i cortex og evt. koma.
13.0.4. Benvn de to vsentligste aminerge kerneomrder i formatio reticularis og angiv
kerneomrdernes lokalisation og deres indhold af neurotransmitter.
Det er:
Raphe-kernerne: Ligger i midtlinjen af medulla oblongata, pons og mesencephalon. Serotonin
er neurotransmitteren.
Nc. coeruleus: Ligger lige under gulvet af fjerde ventrikel i pons. Noradrenalin er
neurotransmitteren.

13.0.5. Hvad viser et EEG?


EEG er en forkortelse for elektroencefalografi. EEG registrerer elektriske impulser i cortex mlt via
huden p skalpen.
Et EEG viser alfa- og betablger:
Alfablger: Relativt langsomme og regelmssige blger.
Betablger: Relativt hurtige og med lav amplitude.
13.0.6. Nvn to situationer, hvor EEG-aktiviteten ndres.
Der er mange:
Opgaver side 103

Der er mange:
Ved fokusering af opmrksomhed.
Svn.
Eplepsi-anfald
Ved indtagelse af visse medikamenter.
13.0.7. Hvilken sammenhng er der mellem formatio reticularis aktivitet og den cortikale
neuronale aktivitet, mlt ved elektroencephalografi?

Nr formatio reticularis bliver mere aktiv bliver blgerne i cortex til mere usynkroniserede
betablger.
13.0.8. Hvad er den typiske ndring i den cortikale elektriske aktivitet ved arousal udlst via
aktivering af formation reticularis.
Arousal er overgangen fra afslappet til opmrksom tilstand.
Nr formatio reticularis bliver mere aktiv bliver blgerne i cortex til mere usynkroniserede
betablger.

Opgaver side 104

You might also like